STC Series 66 Final Exams 7, 8, 9, 10, 11

Pataasin ang iyong marka sa homework at exams ngayon gamit ang Quizwiz!

Several years ago, a person received a gift of 300 shares of stock that were originally purchased for $10 per share. After the gift, the person then inherited 700 shares of the stock when the price was $20 per share. This year, the person sold all of the shares for $40 per share. What are the tax consequences?

$23,000 long-term capital gain Capital gains are calculated by taking the sales proceeds (i.e., sale price of $40 in this question) and subtracting the cost basis. The cost basis for securities that are received as a gift is the lower of the donor's cost basis (i.e., $10) or the market value at the time of the gift, which was not provided in the question. As a result, the gain on the gifted securities is $30 per share ($40 - $10), or $9,000 total ($30 gain x 300 shares). Since the investor held the securities for more than one year, it's considered a long-term capital gain. For securities that are inherited, the cost basis is the market value at the time of previous owner's death (i.e., $20). As a result, the gain on the inherited securities is $20 per share ($40 - $20), or $14,000 total ($20 gain x 700 shares). On any securities that are inherited, gains and losses are always long-term, regardless of the decedent's or recipient's holding period.

What's the formula for calculating total return?

(Ending Value - Starting Value + Investment Income)/Starting Value Total return measures an investor's return over his entire holding period. Total return incorporates capital gains or losses, adds investment income (e.g., dividends or interest) and then divides by the original investment.

When the beta for a portfolio is 1.0, the portfolio return is 12%. What's the alpha if the portfolio now has a beta of 1.4 and the actual return is 18.8%?

+2.00 A portfolio's alpha is calculated by taking the actual return (which is 18.8% for this question) and subtracting the expected return. A portfolio's expected return is found by taking the beta and multiplying by the return on the market (i.e., the S&P 500). Although this question doesn't indicate the return on the market, it does indicate that the expected return is 12% when the beta is 1.0. Since the market has a beta of 1.0, it can be assumed that the return on the market is 12%. Therefore, the expected return on the portfolio is 16.8% (Beta of 1.4 x 12% return on the market). Ultimately, the Alpha is +2% (18.8% actual return - 16.8% expected return).

On January 1, an investor purchased 100 shares of ABC common stock for $30,000. On December 31 of that year, the shares are worth $34,000 and the investor also received $300 in dividends. What's the holding period rate of return?

14.33% Holding Period Return is also referred to as Total Return. The formula for calculating Total Return is: (Ending Value - Beginning Value) + Investment Income/Beginning Value. The investor generated a capital gain of $4,000 since the stock was purchased for $30,000 and subsequently sold for $34,000. The investor also received $300 in dividends; therefore, the holding period return is 14.33%. The calculation is done as follows: ($34,000 - $30,000) + $300/$30,000.

Ten years ago, a person invested $1,000. Today, the original investment is worth $4,000. What is the rate of the annual compounding interest?

14.4% The easiest way to determine the answer is by using the Rule of 72. The Rule of 72 can estimate the rate of annual compounding interest, also referred to as the compounded growth rate, but only if the number of years it takes for an investment to double is known. In this question, the investment has doubled twice, from $1,000 to $2,000, and then again from $2,000 to $4,000, in 10 years. We can assume that it took five years for the investment to double once. The next step is to take 72 and divide by the number of years it takes for the investment to double (in this question, five years). Therefore, 72 ÷ 5 = 14.4, or 14.4%. The Rule of 72 only provides an estimated rate. The actual compounding interest rate may be slightly lower.

What is an investor's internal rate of return if she invested $1,000 and it has grown to $4,000 in 10 years?

14.4% This question can be solved using the Rule of 72. In order to determine the internal rate of return using the Rule of 72, first determine how long it took for the investment to double in value. In this question, $1,000 doubles twice in 10 years (i.e., $1,000 x 2 = $2,000, then $2,000 x 2 = $4,000); which means it took five years for the investment to double once. Then, take 72 and divided by 5 to find the internal rate of return of 14.4% (72/5).

What's the current ratio of a company that has current assets of $10, fixed assets of $20, current liabilities of $5, long-term liabilities of $5, and shareholders' equity of $20?

2 to 1 To calculate current ratio, current assets are divided by current liabilities. In this question, the current ratio is 2-to-1 ($10 current assets/$5 current liabilities).

An investor is in the 20% marginal tax bracket and has a yield of 10% on a portfolio. If the CPI is 5%, what's the investor's after-tax inflation-adjusted return?

2.86% The first step is to adjust the investor's yield for taxes. The formula for finding After-Tax Yield is: Nominal Yield x (100% - Tax Rate%). In this case, the investor's after-tax yield equals 8% [10% x (100% - 20%)]. The next step is to adjust the after-tax yield of 8% for inflation. The formula for finding Inflation-Adjusted Return is: [(1 + After-Tax Yield) ÷ (1 + Inflation Rate)] - 1. For this question, the inflation-adjusted return equals 2.86% [(1 + .08) ÷ (1 + .05)] - 1. An alternative method for calculating the inflation-adjusted return is: After-Tax Return - Inflation Rate, which equals 3% (8% - 5%). Obviously, this method is not as precise, but it's a more simple calculation. For exam purposes, it may be helpful to be prepared to calculate it using both methods to definitively find the correct answer. Returns that are adjusted for inflation are also referred to as "real returns".

An investor buys a two-year U.S. Treasury note that has a 6% coupon. If the note is purchased at par and held to maturity, what is the real rate of return over the holding period, assuming the CPI is 3%?

3% The method used to calculate the note's real rate of return (or inflation adjusted return), is to subtract the rate of inflation as measured by the CPI, from the note's coupon. Therefore, the real rate of return is 3% (6% coupon less the CPI of 3%).

An investor in the 35% tax bracket is considering investing in a corporate bond, which has a 6% coupon. In order to earn an amount equal to her after-tax return from the corporate bond, she would need to invest in a tax-free bond that is yielding:

3.9% To determine the after-tax return, multiply the yield on the corporate bond by (1 - Tax Bracket). .06 x (1 - .35) = .039 3.9% is also known as the net yield, or the after-tax return. If the investor bought a tax-free bond (e.g., a municipal bond) yielding 3.9%, divide by (1 - .35) to obtain the taxable-equivalent yield, or 6% in this example.

If a corporation had annual earnings per share of $2.40, and $1.60 was retained by the corporation, the annual dividend payout ratio is what percentage of earnings?

33% To determine the annual dividend payout ratio, first determine the dividend, which is the earnings per share of $2.40, minus the retained earnings of $1.60 or $.80. Then divide the dividend of $.80 by the $2.40 of earnings per share to determine the percentage of earnings paid to the shareholders, 33%.

If a money market fund pays 1% per year for four years, what's its total return?

4% The formula for calculating Total Return = (Ending Value - Beginning Value) + Investment Income/Beginning Value. Since money market funds attempt to keep their value stable, investors will not have any gains or losses. Investors in money market funds will only receive the interest payments every year. In this question, investors will receive 1% of their investment each year for four years, which is approximately 4%. For example, a $10 investment will pay $0.10 per year and $0.40 over four years; therefore, the total return is 4% ($.40/$10).

Mark purchases an equity-indexed annuity contract that guarantees a 5% return with an 80% participation rate and a 12% interest-rate cap. The index to which the funds are tied rises in value by 10% this year. What return does Mark receive?

8% In an equity-indexed annuity, the owner receives a guaranteed minimum interest rate with potential upside based on the performance of the designated index. If the return on this index is less than the guaranteed rate, the owner receives the minimum. If the index return is greater than the guarantee, the owner receives the greater return up to the capped maximum. Many contracts only pay a portion of the index return. In this example, the client is entitled to 80% of the index return capped at a 12% maximum. The index increased by 10%, so the client's contract is credited with 80% of that amount, or 8%.

While examining a client's investment profile, an IAR determines that the client is able to tolerate a high degree of risk and does not anticipate the need to access invested funds for the next 25 years. What would be the best investment allocation for the client's portfolio?

95% equities and 5% money-market instruments An investor who has a long time horizon and is willing to tolerate high levels of risk may allocate a large percentage of her portfolio to stocks (equities). The only answer that has a more than 50% of the portfolio allocated to equities is the one that suggest 95% equities and 5% money-market instruments.

The Uniform Securities Act considers which of the following to be an investment adviser representative (IAR)?

A CPA with an office in a state who charges a fee for advice that is more than just incidental to his profession According to the Uniform Securities Act, an individual who solicits, sells, or negotiates investment advisory services is defined as an investment adviser representative (IAR). The rule excludes any person serving in a clerical or administrative position or any employee of a federal covered adviser who does not have an office in the state. Professionals (lawyers, accountants, teachers, and engineers) qualify for the exemption if their advice is incidental to their profession and no additional fee or charge is assessed for the advice.

Under the Investment Advisers Act of 1940, which of the following entities providing advisory services would be considered an investment adviser?

A bank holding company, which is an investment company Under the Investment Advisers Act, the following persons are exempt from the definition of investment adviser. A bank or any bank holding company that is not an investment company Any lawyer, engineer, accountant, or teacher whose advice is incidental to her profession Any broker-dealer whose advisory services are incidental to its business as a broker or dealer Any bona fide publisher Any person whose advice is limited to securities that are direct obligations of the U.S. government or guaranteed by the government A bank holding company that is an investment company is not exempt from the definition of an investment adviser.

An investment advisory firm is analyzing the market and building a portfolio for a client. The firm starts by identifying companies with strong financial performance and then creates forecasts for the entire sector based on its analysis. This is an example of:

A bottom-up approach When an investment adviser analyzes the market by first evaluating individual companies, it is considered a bottom-up approach. Conversely, when an adviser begins by analyzing the performance of a sector as a whole, it is considered a top-down approach.

Which of the following scenarios would be permitted in a soft-dollar arrangement between an adviser and a broker-dealer?

A broker-dealer pays for the cost of a conference concerning the future of the computer software industry An adviser is permitted to use a broker-dealer to execute transactions in exchange for certain services. The term is referred to as soft dollars and it is defined as a means of paying brokerage firms for their services through trade commissions. The key here is that the services that the adviser receives as part of a soft-dollar arrangement must benefit its clients. The broker-dealer is permitted to pay for the cost of the conference that an adviser attends concerning securities within an industry in which the adviser will be invested. Travel costs and any costs that should be paid by the adviser (e.g., salaries of the adviser's internal research staff) are not covered under a soft-dollar arrangement. Whereas the cost of the computer terminals could not be paid for with soft dollars, the cost of the data services would be covered by soft dollars.

A client will not have significant funds to invest for six months, but wants to be able to profit if a stock's price increases. What should she buy?

A call option Buying a call option is the best choice. A call option allows an investor to profit if the stock's price rises (i.e., it's a bullish position). Additionally, options are often less expensive than buying the shares directly. Buying puts and establishing a credit call spread are both bearish and are only profitable if the stock's price falls. Straddles are a way for investors to profit on a stock's price going up or down (i.e., volatile price movement).

A 70-year-old retiree is very risk-averse, but needs to generate investment income. She is not wealthy and is in a low tax bracket. Which of the following investments will BEST meet her needs?

A certificate of deposit Since the client is risk-averse, needs income, and is concerned about her principal fluctuating, the best choice is a certificate of deposit. All of the other choices are unsuitable because they are either too speculative or they are tax-free, which provides her with little benefit since she is in a low tax bracket.

A court has appointed a person to be the guardian for an incompetent individual. To open a guardianship account with a broker-dealer, which of the following court-issued documents is required?

A certificate of incumbency A certificate of incumbency is a court-issued document that provides the legal authority of a court-appointed guardian to act on behalf of another person. The certificate serves as evidence that the listed person is authorized to act as a fiduciary for another person (the account holder) or any unincorporated entity (i.e., business, club, association, or organization). On the other hand, a durable power of attorney authorizes a person to manage the affairs of an individual who is in good health and remains in force if the individual is declared incompetent or becomes incapacitated. It is important to note that a power attorney is not issued by a court; instead, it is issued by one person to another person.

The portfolio manager of a growth fund is analyzing potential common stocks. Generally, the manager will give which of the following the MOST consideration?

A company's year-to-year earnings momentum For growth investors, a key consideration is a company's earnings momentum (growth). A growth company will typically have a significant year-over-year increase in its earnings. On the other hand, value investors will look for stocks that have a low price-to-book value and high current yield.

Schedule 13D is required to be filed with the SEC by any person, or those acting together, who acquire more than 5% of the voting stock of a public corporation. Which of the following statements is NOT TRUE regarding Schedule 13D?

A copy must be given to the current shareholders All of the choices are true except the statement that a copy must be given to the shareholders.

An investment adviser representative sends a letter to a group of prospective clients that contains a coupon for a free financial plan. The investment adviser must keep:

A copy of the letter and the coupon When an investment adviser representative sends an advertisement to a prospective customer, it must be preapproved and kept on file for a minimum of five years. If any service is described as free, it must be free with no other purchase required.

Which of the following situations constitutes an assignment of an investment advisory contract and requires client approval?

A corporation acquires 60% of the assets of an investment adviser that's organized as a partnership An investment advisory contract may not be assigned without a client's consent. Any event that causes a change in the adviser's management or control is considered an assignment. If the adviser is organized as a partnership, the death or resignation of a minority of the partners does not constitute a change of control or management. A sole proprietorship hiring 40% more advisers doesn't signify a majority change; therefore, no assignment has occurred. However, a corporation acquiring 60% of the assets of an IA that's organized as a partnership's constitutes a change of control. Note, an investment adviser that's organized as a partnership MUST notify its clients if the composition of the partnership changes, but is NOT required to obtain client approval.

What formula is used to find the present value of an investment by using a future value that is decreased at a compound rate over time?

A discounting formula Present value is found by discounting the future value of an investment by an internal rate of return. The formula used to determine the present value is called the discounting formula.

Which of the following would most likely be registered with the state Administrator?

A distribution of an interest in a mining or real estate venture Interests in mining or real estate ventures are examples of partnership offerings. General and limited partnerships are often registered with the Administrator in the state in which they are offered. Municipal bonds are not subject to registration requirements since they are categorized as exempt securities under the Uniform Securities Act. Also, mutual fund shares and securities listed on the NYSE are federal covered securities, since these issues are only required to be registered with the SEC

Which of the following communications must be filed with the Administrator?

A final prospectus for a security that's registered using qualification Advertising and promotional materials that relate to exempt securities or exempt transactions are NOT required to be filed with the Administrator. Municipal bonds, U.S. Treasury bonds, and registered investment companies are all exempt under the Uniform Securities Act; therefore any advertisements related to these securities don't need to be filed with the Administrator. A prospectus that's used for a security being registered using qualification must be filed with the Administrator.

Which of the following investment advisers would most likely NOT be required to register with the Administrator?

A firm that provides advice on fixed annuities Investment advisers provide securities-related investment advice. A fixed annuity is not considered a security by state Administrators. Since the firm is not considered to be providing securities-related advice, it would be exempt from the definition of an investment adviser.

Which of the following firms would be required to register as an investment adviser in the state of Utah?

A firm with $90 million in assets under management and an office in Utah that deals exclusively with four small institutional customers Advisers are not required to register as investment advisers in a state if they have no office in the given state AND deal only with institutions in the state or send communications to five or fewer non-institutional customers in the state within a 12-month period. If a firm has a place of business in a state, but has assets under management (AUM) between $100 million and $110 million, it can choose to register with the either the state Administrator OR the SEC. If a firm has a place of business in a state and has AUM of less than $100 million, it must register with the state. Lastly, if a firm has AUM of more than $110 million, it must register with the SEC. These large advisers are referred to as federal covered investment advisers and are not required to register in any state a firm with 90 million in AUM and an office must reg in state regardless of number of clients

What is a mid-cap stock called if it has a P/E ratio of 29, while stocks of other similar companies are selling with a P/E ratio of only 19?

A growth stock Growth stocks tend to have higher P/E ratios due to the premium investors are willing to pay for the potential future growth in earnings. On the other hand, value companies tend to have lower P/E ratios than other similar companies.

Ten years ago, Tom bought 100 shares of ABC stock at $150 ($15,000 basis). Eight years later, ABC stock was trading at $250 and he gave his brother Vince 40 of his shares. Two years later, Tom passed away and Vince inherited Tom's remaining 60 shares when ABC stock was trading at $350. If Vince immediately disposes of all 100 shares at $400, the tax consequences are:

A long term capital gain of $13,000 Vince's cost basis for the 40 shares that are gifted to him is $150 (Tom's original cost). Therefore, when these 40 shares are ultimately sold at $400 (two years after receipt), the $250 gain per share x 40 shares equals a $10,000 long-term gain. Vince's cost basis for the remaining 60 shares is based on the market value at the time of death, which in this case is $350 per share. Since the remaining 60 shares are also being sold at $400, the gain is $3,000 (60 shares x $50 per share). Any securities that are received through inheritance are assigned a long-term holding period. Therefore, the $10,000 gain plus the $3,000 gain are combined to represent a $13,000 long-term capital gain.

A father makes a gift of XYZ stock to his daughter. Two years ago, the father purchased the stock for $5,000 and, at the time of the gift, the stock was worth $10,000. If the daughter sells the stock 10 months later for $12,000, what is the tax implication?

A long-term capital gain of $7,000 When a person receives a gift of stock, the recipient's cost basis is the donor's cost basis or the stock's current market value, whichever is less. The stock was originally purchased by the father for $5,000, but was then given as a gift to the daughter when its current market value was $10,000. Since the original cost basis ($5,000) was less than the current market value ($10,000), the original cost basis is used to determine the gain or loss when the stock is sold. In this question, the daughter subsequently sells the stock for $12,000; therefore, she has a resulting capital gain. To determine the ultimate tax implication, the daughter's holding period is based on the donor's holding period. Since the donor had held the stock for two years prior to the gift, the daughter's holding period is considered long-term. By using the original cost basis of $5,000 and comparing it to the proceeds of $12,000, the result is a long-term capital gain of $7,000 ($12,000 - $5,000). **maximizing cap gain tax***

When comparing value stocks to growth stocks, one difference is that value stocks have which of the following characteristics?

A low price-to-book value Value stocks are those that are considered undervalued in relation to their book value. Compared to growth stocks, value stocks often have a low P/E ratio and a higher dividend yield.

While gathering information from a married couple, an investment adviser asks them to disclose all of the investments that they currently own. Which of the following would be considered an investment?

A money-market account that is registered in their names at a bank Money-market accounts, money-market mutual funds, and CDs are all considered investments. Although term life insurance, health care plans, and automobiles are assets that have value, they are generally not considered investments.

Which of the following securities would NOT be considered federal covered, but would still be considered exempt under the Securities Act of 1933?

A municipal bond issued by a state and sold only in that state While a municipal bond is considered an exempt security under the Securities Act of 1933, it is NOT a federal covered security. Federal covered securities include those listed on a national exchange (NYSE, AMEX, or Nasdaq) and those issued by a registered investment company. There is no federal exemption for either variable annuities or trust certificates.

Which of the following may be included in an advertisement created by an investment adviser (IA)?

A no-strings-attached offer to furnish a list of all the recommendations made by the IA in the last two years Investment adviser advertisements may not make reference to past recommendations unless they offer to furnish all of the recommendations made over at least the last year. Also, specific investment advice and the use of testimonials are generally not permitted in IA advertising.

Which of the following transactions meets the definition of an exempt transaction under the Uniform Securities Act?

A nonissuer transaction of a security filed under the Investment Company Act Any nonissuer transaction of a security registered under the Securities Exchange Act, Investment Company Act, or an isolated nonissuer transaction would be considered exempt transactions. Any offer, but not sale, of a security filed with both the Administrator and SEC would be considered an exempt transaction.

Which of the following is the BEST description of a hedge fund?

A partnership of 55 high-net worth investors in which only one of the partners manages the fund for a fee

Joey, an IAR, has offered to sell a stock to his client Bruno, who is reluctant to make the purchase because he is concerned the stock may decline. To alleviate his fears, Joey promises to buy back the stock at today's price should it decline during the next six months. Joey has engaged in a prohibited practice known as:

A performance guarantee This practice is known as a performance guarantee, as Joey has guaranteed Bruno he cannot lose anything. This practice is prohibited under the Uniform Securities Act.

Which of the following is NOT defined as an IAR?

A person that manages portfolios and assets for its clients This is a challenging question, but three of the choices listed are clearly considered investment adviser representatives (IARs). A person who provides advice as an employee of an investment adviser, a person who solicits business for an investment adviser, and a person who manages other IARs for an investment adviser each meet the definition of an investment adviser representative. On the other hand, a person managing portfolios could be either an individual or a business. Keep in mind, if the person is a business (firm), it's considered an investment adviser (IA) and not an investment adviser representative (IAR).

According to the Uniform Securities Act, which of the following persons must register with the state Administrator?

A person who represents a non-exempt issuer in sales to the public Persons who represent exempt issuers are not defined as an agent so there is no need for registration. Persons who represent non-exempt issuers in sales to the public are defined as agents and must be registered whether or not they receive compensation. Persons who represent non-exempt issuers in sales to existing employees are only defined as agents (and must be registered) if they receive compensation related to the transaction(s). If a person represents a non-exempt issuer involved in an institutional transaction (e.g., investment banking transaction with a broker-dealer), he is not considered an agent and therefore not required to be registered.

The most appropriate buyer(s) for a variable life insurance policy is/are:

A person with an understanding of investments who can tolerate risk A person who is knowledgeable about investments is a candidate for variable life insurance because stocks and bonds are the foundation of the policy. As the market values of the securities fluctuate, cash value and death benefits change. Therefore, the insured must be able to tolerate risk.

A bond pays a 7% coupon. Later, a 9% coupon bond is issued. This bond is issued at:

A premium to the 7% bond A 9% bond would be more attractive to investors than one that pays 7%. Therefore, the price of the 7% bond would decline. This would result in the 9% bond being issued at a premium to the 7% bond.

A hedge fund is:

A private investment pool designed for wealthy, sophisticated investors Hedge funds are private investment pools designed for wealthy, sophisticated investors. These private investment pools tend to be less transparent, regulated, and liquid than investment company products. Hedge funds may also leverage, invest in derivatives, and engage in short selling. These hedge funds are typically sold as private placements.

A wrap fee program is best described as

A program in which a client pays a flat fee for both transaction costs and investment advice Wrap fee programs are advisory programs that bundle advisory services and trading costs together. By paying a single annual fee, which is typically less than 2.5% of the client's assets under management, the client receives various advisory services as well as the ability to execute trades without paying additional commissions or fees.

Mirage Adviser, a registered investment adviser, receives referrals from Ted, a retired businessman. Mirage gives Ted $500 for every client he refers who opens an account with Mirage. Which of the following documents must Mirage's personnel collect from any client Ted refers who opens an account with them?

A signed and dated acknowledgment that the client received Mirage's Brochure and a separate solicitor's disclosure document from Ted An unaffiliated solicitor who receives cash compensation for referring clients to an investment adviser must furnish these clients with both the investment adviser's Brochure and a separate written disclosure document. The separate disclosure document describes the terms of the solicitor's arrangement with the investment adviser including the compensation that the solicitor receives for referrals. The adviser must receive a signed and dated acknowledgment from the client confirming receipt of the adviser's Brochure and the solicitor's disclosure document. The acknowledgment must be collected before the client enters into a contract to retain the adviser's services or at the time this agreement is entered into.

An advisory client is interested in starting a small business. After getting details from the client, it is obvious that he will not be required to obtain a specific license for his business activities and that there will be very limited liabilities. If he is interested in the simplest business form, what is the best recommendation?

A sole proprietorship For a person interested in establishing a small company that will carry few liabilities, the simplest business form is a sole proprietorship. All of the other forms of business would require more extensive paperwork and administration. (67747)

Every investment advisory contract must be in writing and it must include which of the following provisions?

A statement that defines the length of time for which the services are contracted An investment adviser's contract must be in writing and clearly disclose the specific length of time that it is in force. Provided customer consent is obtained, advisory contracts may be assigned to another advisory firm. Advisory contracts may not include a clause which attempts to limit an adviser's liability. Also, an adviser is prohibited from sharing in a client's capital gains unless the client is qualified and meets specific financial criteria.

A sole proprietor desires to set up their business as a separate entity, but retain the same flow-through tax treatment with full ownership. Which of the following business entities would be BEST?

A subchapter S corporation By establishing a Subchapter S Corporation, the business is not taxed. Instead, income and losses flow through to the shareholders, which in this case is one person. While partnerships also provide flow-through of tax treatment, they require more than one owner or investor. A C Corporation is a taxable entity that lacks flow-through tax treatment.

After carefully considering a client's investment objectives, an investment adviser recommends a number of investment options to the client. In addition to seeking long-term appreciation, the client is also concerned with the risk of losing his entire investment. The adviser may recommend a portfolio that provides both growth potential and diversification. The adviser's recommendations for diversification would NOT include:

A technology index mutual fund An index fund of only tech stocks opens the client to sector risk. If technology sector performs poorly and the stocks decline as a whole, the client would lose money. The other three options are all means by which the client can achieve diversification, as his investment is reinvested in a portfolio of various securities from different sectors.

Under the Uniform Securities Act, which of the following is exempt from the definition of an investment adviser?

A trust company that provides investment advice to trust clients for a fee Under the Uniform Securities Act, a trust company is exempt from the definition of an investment adviser. The other persons that are exempt from the definition include: Banks and/or savings institutions Lawyers, accountants, teachers, and engineers (remember L,A,T,E) whose advice is incidental to their profession Broker-dealers whose advisory services are incidental to their business Bona fide publishers Federal covered advisers Any other person that is designated by the Administrator A firm that provides advice about securities (even if they are municipal bonds) for a fee is considered an investment adviser

Which trust is the most expensive for a trustee to manage?

A trust that has several young children as beneficiaries As it relates to trusts, when children are the beneficiaries, it can be assumed that the trust will need to be managed for a longer period and will incur more costs. The other choices will most likely be closed in a shorter period and be less expensive for the trustee to manage

Which investment provides the BEST hedge against inflation?

A variable annuity Compared to the other investment choices, variable annuities provide the best hedge against inflation because of the portfolio's potential to rise in a growing economy. Conversely, fixed-income investments (e.g., bonds and fixed annuities) have the highest inflation risk.

An agent of a broker-dealer previously recommended an investment to a group of customers. Later, the investment declined in value and she has now received a few customer complaints. Which of the following complaints require the firm to contact the customer and document the complaint?

A written customer complaint that alleges a sales practice violation Broker-dealers are always required to document, investigate, and respond to written customer complaints that make allegations of sales practice violations.

Tactical asset allocation is a type of:

Active asset allocation In tactical asset allocation, an investment adviser changes a portfolio's asset mix in an attempt to time the market. This is considered an active asset allocation strategy. Buy-and-hold and systematic rebalancing are passive asset allocation strategies.

An adviser would meet the business standard portion of the three-pronged test pursuant to SEC Release 1092 if:

Advice was given on a regular periodic basis According to SEC Release 1092, an adviser would meet the business standard test if advice was provided regularly. There is no particular percentage of time that must be devoted to giving advice.

When does a person become eligible for Social Security benefits?

After she's worked 40 quarters (think 40 hour work week) Social Security eligibility is based on credits that taxpayers earn by working. Individuals become eligible for Social Security benefits if they have earned 40 credits. Taxpayers can earn up to four credits for every year they work (i.e., one per quarter) and taxpayers become eligible by earning 40 credits. If an individual is earning the maximum credits per year, she can become eligible after 10 years (4 credits per year x 10 years = 40 credits). Be careful, even if a person has earned enough credits, she cannot take benefits until she turns age 62.

When a client purchases mutual fund shares from a broker-dealer, she receives a summary prospectus. When will the broker-dealer send the client a statutory (final) prospectus?

After the purchase has been completed, if requested According to the Investment Company Act of 1940, a client who purchases mutual fund shares must receive a statutory prospectus after the purchase has been completed. Therefore, if a summary prospectus is delivered first, a client must receive or be given access to the statutory prospectus, if requested.

Of the following factors, which one would be the most important to consider when analyzing the investment portfolio of a client who has retirement as her primary investment objective?

Age When analyzing a client's existing portfolio to determine how it affects recommendations you might make, it is important to consider a client's investment objectives and the length of time available to try to meet those objectives. When retirement is the primary objective, it is very important to know the client's age. The other items mentioned are also valuable for an RR to know, but they are not as critical as knowing the client's age.

According to the Uniform Securities Act, if the Administrator revokes, denies, or suspends a registration, it would NOT be required to provide which of the following?

All documents created by the Administrator's office regarding the action Prior to revoking a registration, an Administrator is required to provide a registrant with a written notice, written findings of fact, and an opportunity for a hearing. However, the Administrator is not required to release every document created in the course of the investigation.

One disadvantage of investing in a general partnership versus a limited partnership is that:

All general partners have unlimited liability All general partners have unlimited personal liability for debts of the partnership. They also participate in management decisions and may bind the partnership, e.g., with contracts. Limited partners, in a limited partnership, have no personal liability nor may they participate in management

According to the Investment Advisers Act of 1940, when advertising the performance of its managed accounts, an investment adviser must deduct which of the following?

All management fees and expenses It is a violation of the Investment Company Act of 1940 to advertise a fund's performance unless it deducts all of the associated fees and expenses. By not deducting the fees or expenses, returns would be exaggerated and the advertising would be considered misleading or unethical.

Which of the following statements describes a semi-strong form efficient market?

All public information, including historical data, is reflected in securities prices. The Efficient Market Hypothesis (EMH) explains three different forms -- strong, semi-strong, and weak form efficiency. In a weak form efficient market, all past prices and data are fully reflected in current prices. In a semi-strong form efficient market, all public data, including historical pricing, is reflected in current prices. In a strong form efficient market, both public and non-public (i.e., inside) information is reflected in current prices. The assumption that investors want to minimize risk and maximize returns is made in the Modern Portfolio Theory, not the Efficient Market Hypothesis.

Klearkettle Associates is a registered investment adviser in Delaware. Due to the retirement of the firm's senior partner and namesake, Horace Klearkettle, the firm decides to close it doors permanently, effective Thanksgiving weekend. What is the status of the registrations of Klearkettle's IA representatives upon the firm's closing?

All registrations will be ineffective upon the closing of the firm Investment adviser representative registrations only remain in force as long as the IA representative is affiliated with a firm that has a current registration. An agent is not permitted to act independently if her firm goes out of business.

Which of the following statements is TRUE regarding general partnerships?

All the general partners have the authority to bind the partnership Unless the partnership agreement specifies otherwise, all the partners in a general partnership have the authority to transact business on the partnership's behalf. A general partnership is formed by an agreement among the partners. No specific filing with the state is necessary. All the partners are personally responsible for the partnership's debts. Most partnerships qualify for flow-through taxation, which means that they are not taxed as individual entities unlike C Corporations, which are taxed as separate entities.

An investment adviser representative charges his clients both advisory fees and commissions when they buy and sell various investments. Under the Uniform Securities Act, this practice is:

Allowed if the clients receive the proper disclosures An investment adviser's fees may take many different forms. Fees, commissions, and soft-dollar arrangements are all allowed, provided the customer receives a brochure disclosing the nature of the fees and how they are calculated. A soft-dollar arrangement is one in which an adviser receives research or other services from a broker-dealer in exchange for executing its clients' trades through the firm. The arrangement is acceptable provided the compensation the adviser receives in return for the directed transactions benefits the advisory client. Under the Uniform Securities Act, there is no limit as to how large advisory fees may be. However, fees in excess of 2.5% of the assets under management would generally be considered excessive.

Which of the following styles of options can be exercised sooner than the others?

American-style options America-style options may be exercised at any time before expiration. European-style options may only be exercised on a certain day. Capped style options are automatically exercised at the cap price. Uncovered options are a type of option, not a style, which may only be exercised by the holder.

All of the following accounts would be considered institutional, EXCEPT:

An account with a total value in excess of $1 million Under the USA, there is no minimum size that defines an institutional account. The simple fact that an account value in excess of $1 million does not make it an institutional account.

Under the Investment Advisers Act of 1940, which of the following statements is TRUE regarding performance-based fees?

An adviser may charge a performance fee to qualified clients. Under the Investment Advisers Act of 1940, advisers may charge performance-based fees to qualified clients (those with at least $1 million under management with the adviser or a net worth of more than $2.1 million), non-U.S. residents, registered investment companies, and qualified purchasers (a person with not less than $5 million in investments). Suggesting that an adviser can "only" charge performance-based fees to investment companies or qualified purchasers is incorrect.

Which of the following choices is TRUE regarding performance-based fees under the Investment Advisers Act of 1940?

An adviser may charge a performance-based fee to qualified clients Under the Investment Advisers Act of 1940, advisers may charge a performance-based fee to ***"qualified clients."*** A qualifed client is one with at least ***$1 million under management with the adviser OR a net worth of more than $2.1 million.*** Performance-based fees MAY also be charged to non-U.S. residents, registered investment companies, and "qualified purchasers." A qualified purchaser is a person who owns not less than $5 million in investments.

Under the Investment Advisers Act, which of the following statements is TRUE?

An adviser who has investment discretion may not always have brokerage discretion The client can decide which firm will execute the trades, or the client can give the investment adviser brokerage discretion. If given the authority to select a broker, the investment adviser has the flexibility to select either a full-service broker or a discount broker. Information regarding these arrangements is found in Form ADV Part 2.

Under the Uniform Securities Act, which of the following advisers meets the definition of a federal covered adviser?

An adviser who is registered with the SEC under the Investment Advisers Act of 1940 If an investment adviser registers with the SEC, it is considered federal covered under the Uniform Securities Act. Based on the information provided, the other advisers would not necessarily be federal covered investment advisers. The Investment Advisers Act of 1940 specifically excludes U.S. government securities advisers from the definition of an investment adviser. Therefore, these advisers are required to register at the state level. (

Under NASAA's Statement of Policy on Unethical Business Practices, an adviser may NOT release unauthorized client information to:

An affiliated broker-dealer An investment advisory firm must keep all information concerning its clients confidential. It may release the information only if required to do so by law or with the client's approval. The SEC, FINRA, and the IRS are regulatory agencies that could obtain the information without the client's approval.

All of the following are paid compensation for giving investment advice. Under the Investment Advisers Act, which one is not provided with a general exemption from the definition of an investment adviser?

An agent for an athlete Any person in a profession that does not have an exclusion from the definition of an investment adviser and who meets the three-prong test of SEC Release 1092 would be considered an investment adviser. When determining status, the regulators look at a person's activities, not her title.

Under the Uniform Securities Act, which of the following statements is NOT TRUE concerning the state registration of an agent?

An agent's registration to sell securities in a given state expires at the end of the broker-dealer's fiscal year The licenses of all agent, broker-dealer, investment adviser, and investment adviser representatives expire on December 31 each year and must be renewed in order to be effective. Renewal is accomplished by the payment of a filing fee

Which of the following events would increase a partner's basis in a limited partnership?

An assessment A partner's basis for tax purposes is generally composed of all his contributions to the partnership (both cash and property). An assessment requires the partner to give additional cash to the partnership, which would increase his basis.

If used by a broker-dealer, which of the following advertisements must be filed with the Administrator?

An email sent to potential customers recommending an investment strategy for corporate bonds All advertisements, sales literature, and other written materials (prospectuses, pamphlets, circulars, form letters, etc.) that are intended for distribution to current or potential clients must be filed with the Administrator. For materials relating to exempt securities, exempt transactions, or federal covered securities, filing with the Administrator is not required. Since municipal bond, foreign government bonds, and securities sold under a Regulation D exemption are exempt from state registration, advertisements for those securities don't need to be filed with the Administrator.

Under the Uniform Securities Act, all of the following persons are considered investment adviser representatives, EXCEPT:

An employee of an advisory firm who provides administrative services that relates to portfolio selection The Uniform Securities Act defines an investment adviser representative (IAR) as an employee, partner, officer, or director of the adviser, who performs research, makes recommendations, manages portfolios, or solicits advisory services. However, employees who perform administrative, clerical, and/or ministerial functions are excluded from the definition of an IAR.

If an adviser has custody of customer funds and securities, the submission of Form ADV-E must be performed by:

An independent accountant within 120 days after the completion of an audit Submission of Form ADV-E with the SEC is required if the adviser has custody of client funds and securities. The form must be filed by an independent accountant, not the adviser, within 120 days after the completion of the audit

Under the USA, which of the following entities could be considered an investment adviser?

An insurance company Under the Uniform Securities Act, banks, trust companies, and saving institutions are specifically exempt from the definition of investment adviser. There is no specific exemption for insurance companies.

A federal covered investment adviser currently has five clients in State A. The IA would be required to make a notice filing with the Administrator in State A if it now enters into an advisory contract with which of the following?

An inter vivos trust account set up for a relative If a federal covered investment adviser has more than five individual clients in any one state, notice filing is required by the adviser. Adding the trust would bring the IA to six clients and would trigger the notice filing requirement. Financial institutions, regulated investment companies, and other investment advisers are excluded for the purposes of counting clients. Inter vivos is a legal term referring to a transfer or gift made during a person's lifetime, as opposed to a testamentary transfer (a gift that takes effect on death).

Which of the following investment advisers is subject to registration with the SEC?

An internet investment adviser If an investment adviser provides advice to clients exclusively through an interactive website based on information that has been submitted by its clients, the adviser is considered an Internet investment adviser and is subject to SEC registration. Additionally, the IA must maintain a record which demonstrates that the advice was exclusively provided through the interactive website. Advisers that are required to register in fewer than 15 states and hedge fund advisers that manage less than $150 million in assets are required to register with the Administrator, not the SEC. A newly formed investment adviser is permitted to register with the SEC as long as it meets the eligibility requirements within 120 days of its formation, not within 120 days of its second anniversary.

According to NASAA's Statement of Policy on Unethical Business Practices, which of the following business practices would NOT be considered unethical?

An investment adviser disclosing to potential clients that all of the employees have MBAs from an Ivy League School If all the employees of an investment adviser have MBAs from an Ivy League school, this may be disclosed to potential clients. An investment adviser may not: Misrepresent to a prospective client the qualifications of the investment adviser or any employee of the investment adviser Borrow money or securities from a client unless the client is a broker-dealer or a financial institution engaged in the business of lending funds Lend money to a client unless the investment adviser is a financial institution (e.g., a bank) normally engaged in that business Send research reports prepared by a third party without proper disclosure to clients

According to the Uniform Securities Act, which of the following investment advisory practices is prohibited?

An investment advisory firm is purchased by a large broker-dealer and all client contracts are automatically amended to reflect the broker-dealer ownership Investment advisory contracts must provide that: The adviser will not be compensated on the basis of a share of the capital appreciation of the account. The adviser may not assign client contracts without the consent of the client. If the adviser is a partnership, clients will be notified of changes in the partnership within a reasonable period. It is perfectly acceptable to refund advisory fees if an advisory contract is terminated and to charge a fee based on the total value of the account ($150,000). There is no requirement to notify clients if three new portfolio managers (who are not partners or owners) are hired by the firm. (62576)

Regarding risk tolerance and suitability, Modern Portfolio Theory contends that:

An investment that is risky on its own might be less risky in a portfolio if it behaves independently from the other investments in that portfolio An investment that might not be suitable on its own, such as a hedge fund, might be appropriate as part of a larger portfolio. According to Modern Portfolio Theory, an investment that is risky but relatively uncorrelated with other investments in the portfolio, could reduce the risk of the portfolio while maintaining or even increasing the portfolio's expected return.

An investment adviser has recently published an advertisement that is directed to accredited investors and states that it is a fee-only adviser. Which of the following fees received by the adviser will cause the advertisement to be misleading?

Any 12b-1 fees on mutual fund share sales Under the Investment Company Act of 1940, 12b-1 fees are asset based charges that are collected by broker-dealers to compensate their registered representatives who are involved in mutual fund shares sales. Collecting 12b-1 fees will cause a conflict with the advertisement since these fees are not assessed for creating a plan or providing advice. All of the other fees are acceptable according to the advertisement. (89092)

Which of the following statements is TRUE of using a UGMA or UTMA account to save for college tuition compared with using a 529 plan?

Assets in the account are considered the property of the minor. Uniform Gift to Minors Act (UGMA) and Uniform Transfer to Minors Act (UTMA) accounts are a way for minors to own securities. Any money or investment directed to an UGMA or UTMA account is the property of the minor; however, UGMA or UTMA accounts are not provided with tax breaks. Since qualified withdrawals from 529 plans are not taxed, they're typically a better way to save for college. In a 529 plan, assets are owned by the donor (i.e., parent or grandparent who funded the account), not by the minor. Large gifts that are made to UGMA accounts, UTMA accounts, and 529 plans are subject to the gift tax.

Which of the following statements is TRUE regarding the receipt of Social Security benefits?

At age 62, a recipient receives a lower initial benefit, but he will receive benefits for more years. At age 62, eligible workers are permitted to start receiving benefits at a reduced level. The reduced level is partially offset by the fact that they will receive benefits for a longer period. If a person waits until his full retirement age (either 66 or 67), he will start with a higher payout. However, at age 70, a person must start receiving benefits. Keep in mind, no recipient forgoes cost of living increases.

The advantages of a living (inter vivos) trust include:

Avoiding probate A living (inter vivos) trust is a trust created while the grantor is still alive. One of the main advantages of a trust is that it allows the estate to avoid probate. Trusts do not allow people to avoid estate taxes, but they can be used to reduce them in certain circumstances. (Most living trusts are revocable which means the grantor may rescind them at any time. The assets of a revocable trust must be included in the estate when calculating the estate taxes due.)

According to NASAA provisions, an investment adviser that maintains custody of a client's funds must:

Be subject to a surprise audit by an independent accountant If an investment adviser maintains custody of its clients' funds, it must provide prompt written notification as to the location of where the funds are being held as well as whether the location is changed. The notification of taking custody of client funds must be provided in written form, not verbal. Any audit of the records must be performed by an independent accountant, not by the Administrator.

Which of the following positions would expose a customer to the greatest possible loss?

Being long 200 shares of ABC stock and selling 3 ABC calls If a customer is long 200 shares of stock, but short 3 calls, she has both covered and uncovered calls. The customer is covered on two of the call options that she sold, but uncovered on the remaining short call option. For that reason, if the underlying stock rises, she has an unlimited potential loss. When long 200 shares of stock and short 2 calls, it is considered a covered call. The position is covered since, if she is exercised against, she can deliver the 200 shares that she's long. A covered call position has limited risk. A position in which an investor is long 1,000 shares and long 10 puts is referred to as a protective put, which is an effective hedge against downside risk. When long 10 calls and long 10 puts on the same stock, the investor has created a long straddle. With a long straddle, the investor wants the stock's price to be volatile (either rise or fall dramatically). For a long straddle, the maximum loss is limited to the total premiums paid

A client has invested $20,000 in a variable annuity. After 10 years, the annuity is valued at $45,000. If the client withdrew $20,000 at age 59, he is subject to:

Being taxed on the distribution as ordinary income, plus a 10% penalty on the amount withdrawn If an individual purchases a variable annuity, it is not considered a tax-qualified plan. The contribution is not taxed upon withdrawal; however, any earnings withdrawn prior to age 59 1/2 are subject to a 10% penalty and ordinary income tax.

All of the following statements about Benchmark Portfolio Management are TRUE, EXCEPT:

Benchmarks should be reweighted at least quarterly to improve the relative performance of the portfolio When portfolio managers construct a portfolio, they usually start with the securities in the benchmark as a beginning point. Then the decision as to what additional positions to take is made in an effort to add value. The benchmark indicates not only the kinds of securities that should be included in the portfolio, but also the types that should not be. For example, choosing a government bond index as the benchmark makes it clear that the portfolio should not include a large percentage of securities with a high degree of risk. Reweighting a benchmark on a quarterly basis would not have any effect on the actual performance of the portfolio. If it were done to make the portfolio appear to perform better, that would be considered, at best, unethical and, more likely, fraudulent.

Which of the following statements best describes an Administrator's right to examine the books and records of a broker-dealer?

Books and records of any broker-dealer registered with the state Administrator may be inspected at any time by the state administrator A state Administrator may conduct a surprise inspection at any time. Providing one day's verbal notice would not be a surprise inspection. It is not necessary to obtain a court order, or to notify the office of the Administrator in the state in which the broker-dealer is located.

An IAR who is registered in State A decides to conduct a series of investment seminars in State B. One of the clients who attends the seminar in State B invests with the IAR, and loses a significant part of his investment. If the client wanted to alert the regulators, which Administrator would have the ability to initiate an investigation and take action?

Both Administrators may investigate the IAR The Uniform Securities Act provides the Administrator with the ability to investigate investor complaints within the state or from outside the state if the investor has done business with an individual or business within the state. Since the customer resides within State B, the Administrator in that state has jurisdiction. The Administrator in State A also has jurisdiction since the IAR is located within his state. The Administrator's role is to protect potential investors within its state and to prevent professionals within its state from acting in a way that could harm other potential investors.

An investor is evaluating two different bonds-one that matures in three years and another that matures in 25 years. Both bonds have a high credit rating. The 3-year bond has a 4% coupon and the 25-year bond has a 7% coupon. If interest rates are expected to decrease, how will the bonds' prices be affected?

Both bonds will gain value The movement of existing bond prices is inversely related to the changes in market interest rates. Since interest rates are expected to fall, prices will then rise. In this case, both bonds will gain value. However, long-term bonds are more volatile and will fluctuate more than short-term bonds.

A new investment advisory firm registers with the Administrator on June 1. Just before its one- year anniversary on June 1 of the following year, the firm renews its registration and the registrations of each of its investment adviser representatives. Which of the following statements is TRUE?

Both the firm's and its IARs' registrations lapsed on December 31 of the prior year Under the Uniform Securities Act, all registrations expire on December 31 and then must be renewed. In this situation, both the firm and the investment adviser representatives have allowed their registrations to lapse.

An approach to investing that selects stocks based on a company's financial health, management team, product offerings, and then compares the market price of the stock to similar companies to determine if the company is undervalued is generally referred to as:

Bottom-up investing The bottom-up analysis method examines stocks based on a variety of factors, including fiscal health, management team, competitive market position, and current and prospective future product offerings. The market price of the stock is then compared to similar companies to ascertain if the company is undervalued in the investor's mind.

A company's stock experiences wild fluctuations due to unsubstantiated rumors about its products. After further analysis, the company's earnings and sales are better than average. The wild price swings may be described as:

Business risk Since the rumors are limited to the products of one specific company, this is an example of business risk. If the rumors affected the market as a whole, it would be an example of market risk. Additionally, an investor has the ability to diversify away from the risk these rumors present by purchasing stocks of several companies. The ability to diversify is a key component in handling business or unsystematic risk.

An investment adviser rebalances a client's portfolio by liquidating a single stock position and investing the proceeds into an index fund. Which of the following risks did the adviser reduce the MOST?

Business risk The best answer is business risk. By diversifying the client's assets from one company's stock into a basket of stocks (index), the risk associated with that single company's profits declining has been reduced.

What's the best way to hedge a long stock position?

Buy a put option The most effective way to hedge a long stock position is to buy a put on that stock. If the price of the shares fall, the long put option gives the owner the ability to sell the shares at the predetermined strike price. A short call option doesn't give the investor control over when the option is exercised and is not an effective hedge. Buying a call option is a bullish position and gives the investor the ability to buy more shares, which doesn't provide any protection if the price of the stock falls. Taking a long futures position is also bullish and doesn't provide protection if the price of the stock falls.

Which of the following option strategies is the BEST hedging strategy if a client is long 1,000 shares of common stock?

Buy puts If the investor wants to hedge against downside moves in a stock, he should buy put options. Purchasing put options is an efficient hedging strategy. Buying calls on stock owned is considered a bullish strategy. Selling or shorting call options is an income strategy and only provides limited downside protection to the extent of the premium received.

Which statement best summarizes the benefits of a buy-and-hold compared with an active management strategy?

Buy-and-hold investing gives investors both lower transaction costs and lower tax liabilities. Buy-and-hold investing is a passive investment style that's appropriate for investors who believe that markets are efficient. This strategy will reduce transaction costs (i.e., no commissions), since investors will buy securities and hold them until they retire. Since investors are not going to be actively selling their securities, they don't incur any capital gains taxes until they retire. On the other hand, active management attempts to beat the market (e.g., S&P 500) by taking advantage of inefficiencies in stock market prices. Sector rotation (a type of active strategy) attempts to profit as the economy moves through the business cycle. Value investors buy stocks that have low P/E ratios and high dividend payments.

A client portfolio is invested in a well-diversified group of large-cap stocks. Which of the following strategies would minimize the market risk connected with this portfolio?

Buying puts on a large-cap index Buying puts is a way to hedge a stock portfolio against market downturns. By buying puts on a large-cap index, the investor could minimize her losses in the event of a downturn in the large-cap market. (

A client is invested in a large number of stocks in different industries. If the client is concerned that they may fall in value, an adviser may recommend a hedging strategy to the client, such as;

Buying puts on the S&P 500 Index In buying puts on the S&P 500 Index, the client has the right to exercise the option at its strike price and receive in cash the intrinsic value of the contract. Should the market decline, the puts would increase in value as they move into the money, thus offsetting losses on the stock portfolio.

Which of the following entities pays federal income taxes?

C Corporation In the U.S., a C Corporation is taxed separately from the owners of the business. After the C Corporation has paid income taxes, it can pay their shareholders a dividend; however, this payment is then taxable to the shareholder (i.e., double taxation). Both S Corporations and sole proprietorships are not taxable entities. In other words, all of their income passes through and is taxed to the owners (i.e., single taxation). Taxation of trusts is more complicated, but in some instances, income from a trust is only taxable to the beneficiary. Since C Corporations are always taxed, it's the best answer to this question.

All of the following entities provide for the direct flow-through of income to its owners, EXCEPT:

C Corporations S Corporations, limited partnerships, and limited liability companies each directly distribute income to their owners. For that reason, these types of investments are considered tax efficient. On the other hand, C Corporations are taxable entities (tax inefficient). The income that is earned by a C Corporation is taxed at the corporate level and, if paid to investors in the form of a dividend, it is taxed again.

Which of the following factors is a disadvantage of a buy/hold strategy?

Changing portfolio risk levels While a buy/hold strategy has the advantage of minimizing capital gains taxes and transaction costs, the mix of assets can drift substantially from the original asset allocation, changing the risk levels of the portfolio. The risk level may eventually be outside the client's risk tolerance

Which of the following advisory fees is prohibited?

Charging a fee of 5% on the highest value of the account each month Advisory fees must be appropriate for the service being provided. Charging a flat fee, regardless of how much management has been provided by the adviser, is prohibited. Investment advisers are allowed to charge flat fees for creating financial plans, as well as hourly fees or fees that are based on assets under management, provided they're not excessive for the service being provided. Charging a 1% fee based on assets under management is roughly the industry average; however, charging 5% is most likely prohibited under both state and federal law.

Philip is considering the use of sector rotation as a strategy for his portfolio. Which of the following statements best describes how Philip's portfolio will be managed after implementing this strategy?

Choice of sectors for the portfolio is mainly based on economic indicators Shifting investments from one business sector to another is referred to as sector rotation. As the economic cycle changes, different business sectors will be affected. Some sectors will outperform due to the change in economic activity, while others will not. The strategy is used in an attempt to profit through timing from a particular economic cycle. Based on the reading of the economy and its future direction, a portfolio will be shifted into different sectors in anticipation of a particular sector outperforming the others at that point in the business cycle. Fundamental analysis is not used to determine which sector to rotate into, though it could be used to determine which companies to buy once the sector has been chosen. The current performance of a sector will not determine the choice in which to invest or not invest, since this strategy is used to pick where to go next.

Under the Uniform Securities Act, an IA would MOST LIKELY need to be registered in a state if it has no place of business in the state but its:

Clients are all considered accredited investors If an IA has no place of business in a state, it would only be required to register in that state if it provides advisory services or directs business communications to more than five noninstitutional (individual) clients there. If an advisory firm is state-regulated and has a large number of accredited investors in a state, it will most likely be required to register in that state. Remember, the Uniform Securities Act does not recognize accredited investors as institutional investors.

All of the following records must be retained by an investment adviser and broker-dealers, EXCEPT:

Clients' tax returns used to obtain their annual income Any investment adviser subject to the registration provisions of the Advisers Act is required to keep certain records. Ledgers are used by the firm for accounting purposes. E-mail is used as a means of communication between the firm and clients. Trade tickets are used to record all orders for the purchase and sale of securities. There is no requirement that an investment adviser or broker-dealer retain a copy of a client's tax return.

Which one of the following investments trade independently from its net asset value (NAV)?

Closed-end fund Mutual funds (open-end funds), unit investment trusts, and variable annuities are priced based on their net asset values. A closed-end investment company share may sell at, above, or below its net asset value since it trades on the stock exchange

All of the following financial instruments are derivatives, EXCEPT:

Closed-end fund shares A derivative security is a financial product that is valued based on the worth of another (underlying) security. The value of an option contract is determined by the underlying stock (or other financial product). A LEAP is a type of option contract with an expiration date that's longer than nine months. The value of a futures contract is also based on the financial instrument or commodity that is referenced in the contract. On the other hand, the value of a closed-end fund's shares is simply their market price.

Which of the following can time-weighted return be used to evaluate?

Comparing the performance of portfolio managers Time-weighted return is used to evaluate the performance of money managers. Time weighted return minimizes the impact of investor deposits and withdrawals, which cannot be controlled by the manager.

The Administrator of the state of California has begun an investigation of your investment advisory firm. You work at the home office in Ohio and are registered as an agent in both California and Ohio. You have received a subpoena to testify at a hearing in Los Angeles. Your supervisor tells you that if you testify, you will not have a job when you get back. What do you do in this situation?

Comply with the subpoena and testify According the Uniform Securities Act, no person is excused from attending and testifying in response to a subpoena, and no individual can be prosecuted or subjected to any penalty for testifying

Which of the following terms BEST describes the process for calculating future value?

Compounding To calculate future value, cash flows are compounded to determine the expected value at a future date. The process of compounding involves periodically reinvesting earnings on the principal. Amortization is an accounting term that's used to describe how a company recognizes certain costs/expenses over multiple years. Annualizing is a situation in which a return for a certain period is projected out over the year. Since interest is not always compounded on an annual basis, compounding is the best answer.

Currently, many economic indicators are pointing to an imminent decline in the market. If an investor chooses to ignore the negative sentiment and increases her investment positions, she is using what style of investing?

Contrarian An investor who uses the contrarian style of investing is going against market trends and conventional wisdom. Many contrarians search for stocks that are out of favor and have low P/Es.

All of the following details regarding a Coverdell Education Savings Account are TRUE, EXCEPT:

Contributions are always tax-deductible Contributions to a Coverdell Education Savings Account (CESA) are limited to $2,000 annually and are never tax-deductible. Additionally, contributions are not allowed if the contributor's income exceeds a certain amount. Withdrawals from the account are tax-free if used for qualified educational expenses.

Which of the following is NOT a precious metal?

Copper Precious metals are metals that are relatively rare and more difficult to find than other types of metals. The rarity of precious metals is what allows them to command a higher price. Precious metals include gold, silver, platinum, and palladium.

What's the formula for calculating working capital?

Current assets - current liabilities A company calculates its working capital by taking Current Assets and subtracting Current Liabilities. Working capital measures liquidity, which is the short-term financial health of a company.

Which of the following doesn't appear on a bond confirmation?

Current yield When a customer buys a bond, a broker-dealer must send a trade confirmation which indicates its par value, as well as the lower of the yield-to-call or yield-to-maturity. The bond's current yield is not required to be disclosed on the confirmation.

Which ratio measures the amount of leverage that a company maintains?

Debt-to-Equity Leverage is another term for borrowing or debt. The debt-to-equity ratio measures the leverage of a firm. The current and quick asset ratios measure liquidity or, put another way, the short-term financial health of a company. The price-to earnings (PE) ratio measures the value of a company (i.e., its stock price) relative to the profitability (i.e., earnings per share [EPS]).

If an analyst wants to measure the degree to which a company or partnership is leveraged, he would calculate the:

Debt-to-total capital ratio, which is debt / total capital The debt-to-capital and debt-to-equity ratios both measure the amount of a company's capital that is financed with debt (i.e., its degree of leverage). The quick asset test and current ratio both measure a company's liquidity or short-term financial health.

A 6% coupon bond is selling at a basis of 6.20. If interest rates in the market decline below 6%, the bond's basis will:

Decline A bond's basis is synonymous with its yield-to-maturity. Interest rates and yield-to-maturity on a bond will move in the same direction. If market interest rates decline, it means that yields (including the yield-to-maturity) will also decline.

Z Best, a broker-dealer, has filed an application for registration with the Administrator of New York. Three years ago, the firm was suspended from trading on the London Stock Exchange, due to the activities of a rogue trader. The firm's clients were not affected in any way. Upon receipt of the application, the Administrator may:

Deny registration, as any action by a foreign securities regulator may result in a denial by the Administrator Under Sec. 204(H.1) of the USA, the Administrator may, by order, deny, suspend, revoke, cancel, or withdraw the registration of any registrant (i.e., broker-dealer, agent, investment adviser, investment adviser representative) that has been the subject of a foreign securities regulator's disciplinary action, within the past five years. The fact that foreign governments have no jurisdiction or that evidence may be inadmissible is irrelevant.

The amount of the bond that a broker-dealer is required to post is:

Determined by the Administrator The Administrator determines the amount of the bond that a broker-dealer must post. The Administrator is allowed to waive the bond requirement if the broker-dealer has a minimum amount of net capital, which is also determined by the Administrator.

The Simon Morris Insurance Company is a large sponsor of variable annuities and is located in State I. The company also owns a broker-dealer with offices in States W and I, and is registered in States X and Y. An agent of the broker-dealer selling the company's variable annuity products would be required to take which of the following actions?

Disclose the relationship to clients in States W, I, X, and Y Failing to disclose that a broker-dealer is affiliated with or controlled by an issuer of securities is considered a dishonest and/or unethical business practice. The agent would need to disclose the affiliation before entering into any contract with any client that will be purchasing the variable annuities. The disclosure may be made verbally prior to the trade if written disclosure is made at or before the completion of the transaction (usually the settlement date). The broker-dealer or insurance company is not required to refund any fees charged to clients.

If an IA or any advisory affiliate pleads nolo contendere to a felony that was committed in a foreign jurisdiction, this action is:

Disclosed to clients on Form ADV Part 2 An investment adviser is required to provide clients with disclosure of certain disciplinary events on Form ADV Part 2 and then must complete a Disclosure Reporting Page (DRP) to provide details regarding the event. DRPs provide information to the public and/or to regulators about specific criminal, regulatory, and civil actions. Any felony or misdemeanor charges or convictions against an IA must be reported on a Criminal Action DRP. For any actions that are taken by the SEC, state, or a foreign financial regulatory authority, reporting is done on a Regulatory Action DRP. For proceedings in a civil court, reporting is done on a Civil Judicial DRP. Although advisers must also include DRPs in Form ADV Part 1, that disclosure is for the applicable regulator (i.e., the SEC or state Administrator), but not for individual clients

According to the Securities Exchange Act of 1934, which of the following is NOT required to be included on the confirmation for a bond trade?

Disclosure of the bond's rating Although a bond's rating is not required to be included on a confirmation, if the bond is unrated, this fact must be disclosed. A bond's price and its yield-to-maturity at the time of the trade must be disclosed on a confirmation.

ABC Farms, a regional farm, is seeking to expand its product line through the acquisition of another regional farm and has targeted XYZ Farm, which is listed on Nasdaq. XYZ Farms has established a powerful West Coast presence over the last two decades. ABC Farms has not yet contacted XYZ Farms and is in the process of formulating an initial price range for its offer. What methodology is ABC Farms most likely use to formulate its bid?

Discounted cash flow analysis Discounted cash flow analysis is used to determine a company's value based on its projected future cash flows. Duration measures a bond's price sensitivity to changes in interest rates. The future value formula determines the amount of money that a specific dollar amount being invested today will be worth in the future based on a given the number of compounding periods and an internal rate of return. The efficient market hypothesis is a theory which states that market prices reflect all relevant information and that investors cannot beat the market.

An investment adviser is attempting to determine whether a fixed-income security is priced attractively relative to a client's desired annual interest rate. Which of the following methods would BEST determine a bond's fair value?

Discounted cash flows Discounted cash flow evaluates each coupon payment and the repayment of a bond's principal at a present value, based on a rate of return. This makes it possible to evaluate a bond's value against the investor's desired rate of return. The sum of each of the discounted cash flows, plus the present value of the bond's principal, determine the total value of the bond. By comparing this value to the current price of the bond, the adviser will be able to determine if the bond is an attractive investment for her client

In reference to storing customer books and records, an adviser is permitted to store records on:

Disks, provided the information cannot be altered Books and records must be maintained in an easily accessible place for five years. During the first two years, the records must be maintained in an appropriate office of the investment adviser. Records may be preserved on microfilm, microfiche, or any similar device. They may also be kept on various electronic storage media such as CD-ROMs, provided the disks are tamper-evident (write once read many). This means that any attempt to alter the records would become obvious and easily determined upon examination. These files do not need to be password-protected, but the adviser must be able to limit access to the records to authorized personnel and regulators. (

Which of the following statements is TRUE regarding a 403(b) plan?

Distributions from the plan will be subject to taxation at ordinary income tax rates because of the zero cost basis Contributions to a 403(b) plan are made on a pretax basis, resulting in a zero cost basis. Therefore, all distributions are taxed as ordinary income.

A client of a broker-dealer calls his agent and informs him that he is interested in buying a specific stock when the price is right. The client has just left town on vacation and cannot be reached when news breaks that the agent believes will drive the price of the stock significantly higher. What should the agent do in this situation?

Do nothing since he is unable to reach the client Trading in a customer's account without his prior written authorization is considered unethical. In this situation, since the client did not consent to the trade by granting written authorization, the agent may do nothing. The compliance department may not approve an unauthorized trade. It is also unethical and a violation of securities regulations to place the trade in one brokerage account, with the intent to subsequently transfer the shares into the client's account upon his return. (67643)

One of your clients, John Smith, would like to buy one share of ToyKids Inc. for each of his 12 grandchildren. The average transaction cost on these trades would be 16% based on your firm's minimum commission schedule. What action should you take?

Do the trades, provided you have already informed the client of the higher-than-normal commissions Anytime a client will be subject to higher-than-normal charges, he should be informed of this fact prior to execution. Opening a wrap account for each child is not practical since these accounts typically have minimum asset requirements.

Zack is employed at Indiana Trust Company, a federally chartered bank. The CFO of the company asks Zack to help sell the bank's securities to some potential institutional clients as well as some select few retail investors who do not have accounts with the bank. Under the USA, Zack:

Does not meet the definition of an agent under any circumstances Zack is not considered an agent under any circumstances since he is representing an issuer and is selling securities that are exempt from registration (bank-issued securities). Regardless of whether the securities are being sold to institutional and/or retail investors, Zack does not meet the definition of agent.

When recommending a leveraged ETF to a client, an agent should disclose that:

Due to the daily resetting of the portfolio, a leveraged ETF's performance does not provide true tracking of the underlying index over long periods Since an ETF's underlying portfolio is reset daily, price changes are based on a percentage value for one day only. Therefore, a leveraged ETF's performance does not provide true tracking of the underlying index over longer periods. Keep in mind, purchasing a leveraged ETF on margin increases leveraging; it does not decrease it. Due to the daily resetting feature, leveraged ETFs are not considered to be suitable long-term investments

Which of the following statements is TRUE about ETNs?

ETNs may lose value even if the underlying index remains stable. Unlike an ETF which is backed by an independent pool of securities, an ETN is an unsecured bond that's issued by a financial institution. That company promises to pay ETN holders the return on some index over a certain period and return the principal of the investment at maturity. However, if something happens to the issuing company (e.g., bankruptcy) and it's unable to make good on its promise to pay, ETN holders could be left with a worthless investment.

A client who is willing to accept market risk should be advised to accept which of the following investment recommendations?

Eliminating the unsystematic or diversifiable risk in his portfolio Since the customer is willing to assume market risk, the best course of action is to eliminate unsystematic risk. All of the other choices are inappropriate since they either avoid or eliminate the market risk that the customer is willing to accept. Beta is the measure of the volatility of an asset/portfolio as compared to the volatility of the market. Generally, assets with a beta higher than 1 are more volatile than the market, while assets with a beta lower than 1 are less volatile than the market.

Which of the following statements is NOT TRUE regarding a SEP-IRA?

Employees are permitted to make contributions to the account. A simplified employee pension plan (SEP-IRA) does not allow employees to make contributions. Instead, SEPs are funded by employer contributions only and these contributions are elective (discretionary). (72038)

Who is eligible to enroll in a private 457 plan?

Employees who are members of a union Private 457 plans are retirement plans for non-governmental employers and there are restrictions regarding participant eligibility. Participants in a private 457 plan may include members of a union, hospital workers, and employees of charitable organizations.

Which of the following is a characteristic of a Money Purchase Plan?

Employers must make mandatory contributions A Money Purchase Plan is a type of defined contribution plan to which an employer makes mandatory contributions, regardless of the company's profitability. For tax purposes, the employer is able to deduct the contributions.

According to the Uniform Securities Act, all of the following are considered securities, EXCEPT:

Endowment policies Endowment policies are insurance products and are not considered securities. The Uniform Securities Act often refers to various unregistered securities with odd names that promise unrealistic returns and have been the subject of enforcement actions by the Administrator, such as preorganization certificates and whiskey warehouse receipts.

A project manager needs an internal rate of return of 8%. Her evaluation of a proposed project indicates that it has a net present value (NPV) of zero. Based on this information, the project's estimated internal rate of return (IRR) must be:

Equal to 8% For the project manager to determine whether a project is a viable investment based on a required minimum internal rate of return (IRR), he should calculate the net present value (NPV). NPV is calculated by comparing the present value of the future cash flows (using the present value formula) to the cost of the project. By comparing the projects inflows (in today's dollars) to the project's cost, the manager can determine if the NPV is positive, negative, or equal to zero. If the NPV is positive, then the rate of return earned on the project is greater than the required IRR. On the other hand, if the NPV is negative, then the rate of return earned on the project is less than the required IRR. In this question, since the NPV is zero, the rate of return is equal to the required IRR.

Mary and John are both investment adviser representatives with Everyman Investment Advisers. Mary leaves Everyman to join another firm and her accounts are reassigned to John. What is Everyman required to do under the Investment Advisers Act?

Everyman does not need to do anything Everyman does not need to do anything in this situation. According to the Investment Advisers Act, an advisory contract may not be unilaterally assigned to another investment adviser without the client's consent. However, the contract is between the client and the investment advisory firm (Everyman in this case), not the individual investment adviser representative. If the management or control of Everyman changed hands, such as if it were bought by another firm, then Everyman would need to notify its clients and seek their consent to have their contracts reassigned.

An income statement of an individual would contain:

Expenses The income statement of an individual contains all income (salary and investment income) and expenses that a person has had during a given period. It also contains any gains or losses on sales of assets by the individual as well as any interest paid on borrowed funds. The balance sheet includes the assets owned, the liabilities owed, and the person's net worth, which is assets minus liabilities.

Mr. Lear is a widower who owns rental properties that he wants to leave to his three daughters when he dies. His goals are to minimize estate taxes while also retaining control of the properties during his lifetime. Furthermore, he thinks that his daughter Cordelia is a spendthrift and wants to protect the assets from claims by her creditors. Based on these facts, Mr. Lear should consider forming a:

Family limited partnership A family limited partnership (FLP) is the best choice since it will satisfy all of Mr. Lear's goals. If Mr. Lear set up an FLP with himself as the general partner and gave limited partnership interests to his daughters, he could take advantage of certain IRS rules regarding the valuation of minority and illiquid interests to minimize estate and gift taxes. As the general partner, he also retains control of the assets. Furthermore, it would be difficult for his daughter's creditors to attach the partnership property. On the other hand, a general partnership would not allow him to retain full control of the property since all general partners have an equal say in managing the partnership's assets. A general partnership would also not provide the same kind of estate tax benefits.

Under Section 404(c) of ERISA, which of the following statements regarding fiduciaries is TRUE?

Fiduciaries are not liable for investment losses if certain requirements are met. If a retirement plan meets the requirements of ERISA section 404(c), the plan fiduciary will not be held liable for any loss that is the direct result of a participant's exercise of control over the investment of her plan account. However, plan fiduciaries are responsible for the selection or retention of particular investment options and for investments required by the plan or directed by the plan sponsor.

In order to form a limited partnership, two or more people must:

File a certificate with the appropriate state or local official The only way to create a limited partnership is by filing a certificate (or other document) with a state or local agency. A general partnership, in contrast, is created whenever two or more people agree to form a partnership. The agreement does not even need to be in writing.

According to the Uniform Securities Act, an Administrator is NOT allowed to demand that broker-dealers do which of the following?

File their advertising related to federal covered securities with their state Administrator prior to use According to the Uniform Securities Act, all advertising used by a broker-dealer must be supervised and checked for errors. They are also required to keep a file that is subject to audit by the Administrator. However, the Administrator does not regulate advertising that addresses only federal covered securities.

If an individual forms a broker-dealer as a sole proprietorship, what information needs to be provided to the Administrator?

Financial statements The USA requires a broker-dealer to file its financial statements with the Administrator annually and, in some cases, quarterly.

The Uniform Securities Act requires investment advisory firms to keep their books and records for:

Five years Investment advisory firms are required to keep books and records for a minimum of five years, according to the Uniform Securities Act

According to the NASAA Recordkeeping Requirements for Investment Advisers Model Rule, an investment adviser is required to maintain records for:

Five years, with the first two years in the principal office Investment advisers must keep books and records for five years total, but for the first two years they must be in their principal office. On the other hand, broker-dealers keep most books and records for only three years.

For a new issuance of securities, when is a prospectus required to be delivered to purchasers?

For primary and potentially secondary market transactions When investors buy securities in the primary market, they're always required to be provided with a prospectus. However, if investors buy stock in the secondary market shortly after a primary market issuance (e.g., 25 days following an IPO), they must also receive a prospectus. Summary prospectuses are permitted in some mutual fund transactions, but not for stock and bond purchases in the primary market.

A publicly traded corporation has 20,000,000 shares of common stock outstanding and an investor buys 1,400,000 of the shares in the open market. Which of the following forms is the investor required to file with the SEC

Form 13D Any investor that acquires more than 5% of the common stock of a reporting company is required to file Form 13D with the SEC. Since the client has acquired 7% of the 20,000,000 outstanding common shares ($1.4 million ÷ $20 million), he is subject to the filing requirement. Form 13F is filed by institutional investment managers that exercise investment discretion over $100 million or more in equity securities. Form 144 is filed when an investor intends to sell restricted (private placement) stock or when an insider intends to sell control stock.

The portfolio manager of an open-end investment company has investment discretion for amounts of more than $125 million in equity securities. What form must the manager file with the SEC?

Form 13F If a portfolio manager has investment discretion for more than $100 million of equity securities, the manager must file Form 13F with the SEC within 45 days of the end of quarter.

What form is required when registering as a broker-dealer?

Form BD Form BD is the Uniform Application for Broker-Dealer Registration, which is filed with the SEC, SROs that have jurisdiction, and the appropriate states, through the Central Registration Depository (CRD) operated by FINRA.

An advisory firm has 30 individuals who are registered as IARs. What record is the investment adviser required to keep in regard to its IARs?

Form U4 for each IAR Under the NASAA Recordkeeping Requirements for Investment Advisers Model Rule, an IA is required to maintain all documents which are filed with the state and/or federal regulators that relate to the firm and its IARs (e.g., Form U4, amendments, and renewal filings). IARs are not required to obtain fingerprints from law enforcement.

After occurrence of the event for which a filing is required, when must a Form 8-K be filed?

Four days Unless otherwise specified, Form 8-K must be filed or furnished within four business days after the occurrence of the event for which the filing is made. Form 8-K is required by the SEC to announce certain significant changes in a public company, such as a merger or acquisition, a name or address change, bankruptcy, change of auditor or accountant, or any other information that may be reasonable for a potential investor to know.

After buying a large block of stock for herself, an agent is overheard telling another employee that after her purchase she recommended the same stock to a number of her clients who also purchased it. After completing the clients' orders, the agent sold her stock for a large profit. The agent's prohibited action is best described as:

Front-running Front-running is the prohibited practice of a broker-dealer or agent buying or selling stock before the public release of proprietary information concerning a large block order. In this question, another form of front-running is exemplified by the agent buying a stock and then recommending that her clients buy the same stock. After the recommendation to her clients, the agent would likely benefit from the potential increase in the stock's price. Even if the client recommendations are suitable, it is unethical for the agent to put herself before her clients.

To determine the fair market value of a bond, an adviser would calculate the

Future cash flows discounted to their present value Discounted Cash Flow (DCF) can be used to determine the fair value of a bond. The future cash flows (interest payments and principal) are discounted to their present value and then converted into a single net present value.

An investor wants to know how much money he will have in 10 years if he invests $100,000 in a variable annuity today, assuming an annual average return of 6%. This investor needs to calculate the:

Future value of money The investor is trying to determine the future value of money. He wants to know how much he will have in 10 years if he invests $100,000 today with an annual return of 6%.

A disadvantage of investing in a general partnership versus a limited partnership is:

General partners have unlimited liability All general partners have unlimited personal liability for debts of the partnership. They also participate in management decisions and may bind the partnership, e.g., through contracts. Limited partners have no personal liability nor may they participate in management.

What's a benefit of establishing a general partnership compared to establishing an S-corporation?

General partnerships don't require incorporation with the state An advantage to establishing a general partnership compared to an S-corporation is that general partnerships are not required to file articles of incorporation. This filing is time consuming and expensive. Both types of business are tax efficient, since only the owners are required to pay taxes on the income (i.e., both are flow-through investments).

According to the Capital Asset Pricing Model (CAPM), all of the following statements are TRUE, EXCEPT:

Given a choice of two investments that offer the same return, investors will choose the investment with greater variability. CAPM makes the assumption that all investors are risk-averse. Therefore, when given a choice between two investments that offer the same return, a risk-averse investor will choose the one with less variability (less risk). Likewise, when given a choice between two investments with the same variability (risk), risk-averse investors will choose the one with the higher return.

A stock that is traded on the NYSE has a high P/E ratio, a low dividend payout ratio, and a high level of retained earnings? What type of stock is it?

Growth Growth stocks are characterized by high P/E ratios, low dividend payout ratios and high levels of retained earnings. Therefore, the stock described in the question is most likely a growth stock. Value stocks, are characterized by low P/E ratios, high dividend payout ratios, and low price-to-book ratios. Many growth stocks are also small cap-stocks which are issued by companies with market capitalizations between $300 million and $2 billion or micro-cap stocks which are issued by companies with market capitalizations of less than $300 million. However, the characteristics of the stock being described fit the definition of a growth stock much more exactly.

An individual who primarily invests in securities that have a low dividend payout ratio, a high level of retained earnings, and a high price-to-earnings ratio would be described as a:

Growth investor Stocks with high retained earnings and low dividend payout ratios are considered growth- oriented investments. A value-oriented investor would seek investment in stocks that have low price-to-earnings ratios, or below what is perceived to be the stocks' intrinsic value. A conservative investor invests in low-risk investments that have stable cash flows. This would be consistent with stocks that have a high dividend payout ratio. A contrarian invests in stocks that are currently out of favor with investors.

Under the Uniform Securities Act, an agent's registration will expire if:

He fails to renew his registration by December 31 An agent must renew his registration by December 31, each year. If he is no longer employed by a broker-dealer, his registration is terminated by the state Administrator. It does not expire.

Which statement is FALSE regarding the use of futures contracts for hedging purposes?

Hedging is only used by speculators Hedging strategies are typically used by those who produce or consume a particular commodity. Speculators have no underlying interest in the commodity price and simply take positions for potential profits. While speculators may hedge, this is a much more common practice for the producers and users of the commodity.

The potential loss for a limited partner in a real estate limited partnership is limited to

Her initial investment plus any unpaid amounts to which she had committed to pay In a limited partnership, the liability for a limited partner may not exceed her initial investment amount plus any unpaid amounts to which she has agreed to pay. Generally, limited partners are not liable for the debts that are incurred by the partnership. However, when a limited partner cosigns a loan for the partnership and agrees to be responsible for a portion of the loan, the limited partner's liability is increased.

Which of the following statements is FALSE regarding discounted cash flow methods used to evaluate an investment?

Holding period return is a discounted cash flow method Holding period return does not discount or compound cash flows. Holding period return is calculated by adding any income, plus capital gains, minus capital losses, and dividing by the value of the initial investment.

An investor purchases a bond that was quoted in terms of its yield-to-call. The best outcome for the investor is:

Holding the bond until it matures Bond yields are quoted on the basis of yield-to-call or yield-to-maturity, whichever is lower (i.e., yield-to-worst). Since the yield-to-call is quoted, the yield-to-maturity must be higher and will represent a greater rate of return than if the bond is called.

According to the Uniform Securities Act, an entity can avoid meeting the definition of a broker-dealer if it: Has no office in the state Only deals with institutional clients Does not hold customer funds or securities

I and II If a broker-dealer has no office in a state and effects transactions only with institutional clients, it would be exempt from the definition of a broker-dealer. An institutional broker-dealer cannot qualify for this exemption if it has an office in the state, even if it avoids holding customer funds and securities. (62990)

A registered investment adviser is normally required to post a surety bond to satisfy the Uniform Securities Act requirements. Under which TWO of the following conditions could this requirement be waived? The investment adviser does not maintain custody of client assets. The investment adviser has a net worth of $35,000. The investment adviser is a subsidiary of a broker-dealer. The investment adviser maintains custody of customer assets.

I and II If an investment adviser does not maintain custody of client assets A N D has a minimum net worth of $35,000, the surety bonding requirement could be waived by the Administrator.

Which of the following activities are considered illegal or fraudulent, rather than being simply unethical? An investment adviser representative describes himself to clients as an expert in senior citizen retirement planning and, although no such designation exists, he displays a certificate in his office from a fictitious institution conferring the designation. An agent intentionally solicits sales in securities that are non-exempt and the issuer has not filed a registration statement in the state. An IAR fails to notify his superior of a letter that he received from a client which alleges fraudulent activities on the part of the IAR. An agent of a broker-dealer knowingly provides material, public information to her largest client, with the client ultimately acting on the information.

I and II only Fraud offenses always include some sort of false statement, misrepresentation, or deceitful conduct. Deceit is defined as willful or reckless misrepresentation or concealment of material facts with the intent to mislead. For choice (I), the intentional misrepresentation of a registered person's qualification with the intent to deceive constitutes fraud. For choice (II), an agent soliciting trades in securities that are non-exempt and unregistered is an illegal activity. For choice (III), the failure to notify a superior of a written customer complaint is an unethical business practice, rather than an illegal activity. This is true despite the fact that the allegation in the complaint is of an illegal activity. And lastly, for choice (IV), the action by the agent of the broker-dealer who provided material, public information to a client is not considered to be insider trading since the material information was public. (

All of the following choices are maintained for five years by an IA, EXCEPT: Partnership agreements Articles of incorporation A copy of audited financial statements Written complaints A copy of a solicitor's written disclosure documents

I and II only Partnership agreements, articles of incorporation, minute books from board meetings, and stock certificate books are preserved for the life of the firm and for at least three years after termination of the firm. All other records are retained for five years.

Which of the following choices represent(s) reasons why a limited partnership is the business form often used in direct participation programs? Partnerships are not considered a separate entity for tax purposes. Partnerships pass through the results of their operations for treatment on the individual's tax return. Partners are never liable for tax when they invest through this type of structure.

I and II only Partnerships are not considered a separate entity, as are corporations, for tax purposes. Instead, income or loss from the partnership's operations is passed through to the partners for tax treatment. (III) is not a correct statement because taxes need to be paid by the partners if the business reports a profit.

Which of TWO of the following actions will increase an investor's basis in a limited partnership? The investor contributes additional capital to the partnership. The investor provides managerial services to the partnership. The investor contributes property to the partnership. The partnership passes on investment losses to the partners.

I and III An investor's basis in a partnership is increased whenever she contributes additional cash or property to the partnership. The contribution of services will not increase the investor's basis. Cash distributions and losses that are passed through to the partners will actually decrease the investor's basis.

Which TWO of the following types of risk cannot be mitigated by diversifying into various sectors? Systematic risk Nonsystematic risk Nondiversifiable risk Business risk

I and III Systematic or nondiversifiable risk (market risk) is the risk associated with the entire market. The assumption is that, regardless of how many different securities an investor may hold, if the overall market declines, no amount of diversification will reduce the risk of loss. Business risk is the risk that circumstances or factors may have a negative impact on the operation or profitability of a given company. For example, a company may suffer due to increased competition, a decrease in demand for its goods or services, or adverse economic conditions.

Which TWO of the following statements are TRUE regarding the registration of securities under the USA? The registration expires one year from the effective date. The registration expires on December 31 following the effective date. The issuer is subject to quarterly state reporting requirements until the registration expires. The issuer is subject to state reporting requirements only until the securities distribution is completed.

I and III Under the Uniform Securities Act, registrations for securities expire one year from the effective date of the registration. Registration of broker-dealers, agents, investment advisers, and investment adviser representatives expire on December 31 of each calendar year. In addition, issuers that register securities under the Blue Sky laws must file quarterly reports with the Administrator for as long as the registration is effective (i.e., one year after the effective date).

If an investment advisory firm decides to increase its fees for all new business, which of the following steps must it take to properly disclose its new fees? Change its brochure to reflect the new fee schedule Deliver a copy of its brochure with the new fee schedule to all current clients Deliver a copy of its brochure with the new fee schedule to all new clients Continue to use the current brochure while it awaits the Administrator's approval

I and III only If an investment adviser changes its fee schedule, it is considered a material change and it must update its Form ADV Part 2 to reflect the new fees. Many IAs use Form ADV Part 2 as the disclosure document that they provide to their clients. After the update is made, the revised brochure must be provided to all new clients; however, the firm is not required to deliver the brochure to current clients. (Note: investment advisers must offer to deliver their disclosure brochures to all of their clients annually.) If an IA changes its fee schedule, it is considered a material change and the firm must amend its Form ADV Part 2.

For federal tax purposes, which TWO of the following are NOT included in gross income? Municipal bond interest Salary Bonuses Alimony payments received

I and IV Municipal bond interest and alimony payments received are excluded from a person's gross income for federal taxes. Alimony is non-deductible for the payer and non-taxable for the recipient. However, both bonuses and salaries are included in gross income and are taxable

Which TWO of the following statements are TRUE regarding the investment risk of a life insurance policy? Whole life policy writers bear the risk that the general account return will not meet the guaranteed rate. Whole life policy owners bear the risk that the general account return will not meet the guaranteed rate. Variable life policy writers bear the risk of poor separate account returns. Variable life policy owners bear the risk of poor separate account returns.

I and IV Since the writer of a whole life policy guarantees the return, the insurer bears the risk that general account returns will not meet the guaranteed rate. In a variable account, the writer does not guarantee increases in death benefits and cash value. They are dependent on the returns in the separate account

Which TWO of the following statements are TRUE regarding Subchapter S Corporations? Their status is terminated if there are more than 100 shareholders. They have a federal charter. Shareholders have unlimited liability. Income and losses flow through to shareholders.

I and IV Subchapter S status is revoked if there are more than 100 shareholders. As with limited partners, shareholders have limited liability, and income and losses flow through to shareholders. Subchapter S Corporations have state charters

Which TWO of the following statements are TRUE regarding the buyer and writer of a straddle? The buyer of a straddle expects the market to fluctuate. The writer of a straddle expects the market to fluctuate. The buyer of a straddle expects the market to remain stable. The writer of a straddle expects the market to remain stable.

I and IV The writer (seller) of a straddle (call and put) believes the stock's price will remain stable. The buyer of a straddle expects that the market price of the underlying stock will be volatile.

Which TWO of the following statements are NOT TRUE regarding TIPS? During a period of inflation, the interest rate is adjusted upward. During a period of deflation, the principal is adjusted downward. During a period of inflation, the principal is adjusted upward. During a period of deflation, the interest rate is adjusted downward.

I and IV Treasury Inflation-Protected Securities (TIPS) pay a fixed rate of interest, based on inflation-adjusted principal. During an inflationary period, the principal of the TIPS increases. During a deflationary period, the principal decreases. When the security matures, the amount paid will be the greater of the original principal or the adjusted principal.

Which TWO of the following statements are TRUE regarding investment advisers that intend to charge a performance-based fee which is based on the monthly capital appreciation in their clients' accounts? This compensation arrangement is allowed for contracts with qualified clients. This compensation arrangement is available to all advisory clients for the term of their contracts. This compensation arrangement is always prohibited under the Uniform Securities Act and NASAA rules. This compensation arrangement may be permitted under the Investment Advisers Act of 1940.

I and IV Under the Investment Advisers Act of 1940, IAs are permitted to enter into performance-based fee arrangements with qualified clients. A qualified client is a person with at least $1 million of assets under management with the adviser OR more than $2.1 million in net worth. At the state level, the Uniform Securities Act makes no mention of performance-based fees; however, NASAA has created a Performance-Based Compensation Exemption for Investment Advisers Model Rule which allows the arrangement to be entered into with only specific clients—one of which is qualified clients.

Gary and Mary were recently divorced. Mary has been ordered to pay child support to Gary, who has custody of their two children. Which of the following statements is TRUE regarding child support payments? Child support is not tax-deductible for the payer. Child support is tax-deductible for the payer. Child support is taxable for the receiver. Child support is not taxable to the receiver.

I and IV only Child support payments are not tax-deductible for the payer and not taxable for the recipient.

Which of the following statements is/are TRUE of an investment adviser who takes custody of cash and securities belonging to its customers? The funds must be deposited into one or more separate accounts that only contain customer funds. The adviser may combine customer cash with its own if proper disclosure is made. The adviser must also be registered as a broker-dealer with the Administrator. The adviser must disclose to its clients both the location and manner in which the securities are held.

I and IV only Firms that maintain custody of customer assets must satisfy these guidelines: (1) Customer funds must be segregated, (2) The location of assets must be disclosed in writing and, (3) The records must be audited annually. When a firm has custody of customer cash and securities, it is required to be a qualified custodian. Broker-dealers, banks, and trust companies may be recognized as qualified custodians and may, therefore, hold cash and securities belonging to customers. Since these different entities may serve as qualified custodians, it is incorrect to state, as in choice (III) that the adviser may only maintain custody if it is also registered as a broker-dealer.

Global Investments is a broker-dealer based in State A. It is currently registered as a broker-dealer is States B, C, and D. The firm is looking to expand its business to State E. Under the Uniform Securities Act, in which of the following situations would Global Investments NOT be required to register as a broker-dealer in State E? The firm transacts business only with other broker-dealers. The firm's sole business in State E is with four or fewer noninstitutional customers within a 12-month period. The firm's sole business in State E is limited to transactions in federal covered securities. The firm transacts business only with institutional investors.

I and IV only If a firm meets the definition of a broker-dealer, it is generally required to register in any state in which it effects transactions in securities. The term broker-dealer, however, does NOT include: Agents Issuers Banks, savings institutions, and trust companies (though banks, holding companies, or bank subsidiaries are not excluded from the definition) A person that has no place of business in the state AND only transacts business with issuers, other broker-dealers, financial institutions, or institutional buyers A person that has no place of business in the state AND is licensed where the person maintains its place of business and sells only to existing clients who are not residents of the state There is no de minimis exemption for broker-dealers with no place of business in a state and a limited number of noninstitutional clients. There is also no exemption based on the type of securities in which the broker-dealer transacts business in a state.

An Administrator from State A has jurisdiction over an offer made: On a radio or television program that originated in State A On a radio or television program broadcast in State A where the communication originated outside the state In a newspaper circulated in State A, but published in State B In a newspaper published in State A, but with more than two-thirds of its circulation outside the state in the last year

I only Generally, an Administrator will have jurisdiction over any offer if it originated in that state, was directed in that state, or was accepted in the Administrator's state. However, if the communication originated outside the state and was in the form of a radio or television broadcast, the Administrator would not have jurisdiction. The Administrator has jurisdiction if the newspaper is published in that state and more than one-third of the circulation is inside that state. (62498)

Which of the following would be considered a sale of securities? A car dealership gives each customer a gift of one free share of stock for buying a new car. An investor converts a bond into 50 shares of common stock. A minor is named as a beneficiary of a trust containing common stock. An individual inherits a security from the estate of a deceased parent.

I only If an individual is required to pay for an item in order to receive a free security, then the security investment is not actually free. The required payment would constitute a sale of the security. The other examples involve either the gift or transfer of securities and would not be considered a sale.

According to the Uniform Securities Act, an IAR may legally make which of the following statements to a client? "I am registered to conduct business within the state." "The Administrator has verified my registration information." "Since I passed an examination, I have been deemed competent and registered."

I only The Administrator does not verify, endorse, or rule on a registrant's competence. A registered person may legally state that she is registered to conduct business within the state.

Triangle Advisers is a very small investment adviser that is planning to register in three states. Which parts of Form ADV must Triangle file? Part 1A Part 1B Part 2

I, II, III A firm must file Form ADV to register as an investment adviser. Generally, this is done electronically through the Investment Adviser Registration Depository (IARD), which forwards the forms to the appropriate regulator(s). The first section of the form consists of Parts 1A and 1B. Part 1A is completed by all firms regardless of their status as a state or federal adviser. Part 1B is completed only by firms that are registering at the state level. Form ADV also contains a separate section (Part 2), which is filed by both federal and state advisers. Part 2 contains the information about the investment adviser's business. Information contained in this section may be copied directly to, or used as, the basis for investor disclosure information found in the investment adviser's investor brochure.

According to NASAA's Statement of Policy on Unethical Business Practices, what information is required to be included in the renewal of an investment advisory contract? The investment advisory fee A statement that no assignment of the contract will be made by the investment adviser without the consent of the client A statement regarding the amount of prepaid fees that must be returned if the contract is terminated The educational background of each IAR

I, II, III NASAA's Statement of Policy on Unethical Business Practices states that the renewal of an investment advisory contract must include the disclosure of all fees and services provided, the method for computing the advisory fee, the amount of prepaid fees to be returned in the event of an early termination of the contract, and the fact that no assignment of the contract will be made without the client's consent. There is no requirement to disclose the educational experience of each investment adviser representative that works with clients.

Under the Uniform Securities Act, registration by coordination becomes effective: If no stop order is in effect At the same time that the SEC registration becomes effective provided the registration statement has been filed with the Administrator for at least 10 days Provided a prospectus has been filed with the Administrator

I, II, III When a security is in registration with the SEC under the Securities Act of 1933, that registration can be coordinated with the Administrator for state registration. A registration under coordination will automatically become effective at the same time the federal registration becomes effective, provided that no stop order is in effect, the registration statement has been on file with the Administrator for at least 10 days, and a statement of the minimum and maximum proposed offering prices and underwriting discounts has been on file for two business days. A prospectus meeting the requirements of the Securities Act of 1933 must also be filed with the Administrator.

Which of the following must be included in a solicitor disclosure document? The manner in which the solicitor will be paid by the registered investment adviser The amount of the client's fee that is related to soliciting activities The details of the agreement between the solicitor and the registered investment adviser The business history of the solicitor

I, II, III only In the solicitor disclosure document, investment advisers must disclose how they intend to pay the solicitor, how much of the client's fee will be paid to the solicitor, and a description of the relationship between the solicitor and the investment adviser. However, a full history of the solicitor is not required to be disclosed.

If an investor owns a portfolio of fixed-income securities, to which of the following risks would an investor be subject? Interest-rate risk Inflation risk Credit risk Opportunity risk

I, II, III, IV All of the risks listed apply to fixed-income securities. Interest-rate risk is the risk that a security's value will change due to a change in interest rates. Inflation risk is the chance that the cash flows from an investment will not be worth as much in the future because of changes in purchasing power due to inflation. Credit risk represents the risk that an investor will experience a loss of principal due to a borrower's failure to repay a loan or otherwise meet a contractual obligation. Opportunity risk is the risk that a better opportunity may present itself after an irreversible decision has been made.

When an investment adviser files Form ADV with the SEC, it will include: The past 10 years of business history and current affiliations of control persons An audited balance sheet The scope of authority over client funds The compensation to be received

I, II, III, IV An investment adviser must disclose on Form ADV the past 10 years of business history. An audited balance sheet is also required. In addition, the adviser must disclose the scope of authority over client funds: e.g., discretion, custody, substantial prepayment of fees. The manner in which the adviser will be compensated must also be disclosed.

The risk tolerance of the customer must be a consideration when providing advice to which of the following types of clients? High net worth investors Corporations Trusts Estates

I, II, III, IV The risk tolerance of the customer must be taken into account in all cases, even when the customer is an institution.

Under ERISA, which of the following transactions are prohibited between a fiduciary and a party in interest? The fiduciary is involved in the sale of property of the plan to a party in interest. The fiduciary extends credit between the plan and a party in interest. The fiduciary furnishes goods or services between the plan and a party in interest. The fiduciary transfers plan assets to a party in interest.

I, II, III, and IV All are prohibited transactions under ERISA

An investment adviser is also registered as a broker-dealer. According to the Investment Advisers Act of 1940, which of the following statements regarding contracts for impersonal advisory services is/are TRUE? These services may be provided orally. These services may contain statistical information as long as the IA expresses no opinions about the statistics. These services may be distributed in writing. These services cannot claim to meet any specific investment goals of any particular investor.

I, II, III, and IV If an investment adviser offers an impersonal advisory service, it may be offered verbally or in written form and may contain statistics. Impersonal advisory service is defined as service that does not purport to meet the objectives or needs of specific individuals or accounts. When an investment adviser also acts in the capacity of a broker-dealer, it must provide its advisory clients with written disclosure.

An advisory client has a portfolio that consists of a diversified group of domestic securities with different maturities. Diversification protects the client from which of the following risks? Business risk Financial risk Liquidity risk Market risk

I, II, and III only A diversified portfolio that focuses its investments domestically is subject to market risk. In order to reduce this systematic (market) risk, an investor must invest in assets that do not trade in the same marketplace (such as foreign markets). If the U.S. bond or stock market declines, all of the securities within that marketplace will be affected. This portfolio is protected from all of the other risks mentioned.

Which of the following items may not be bought on margin? Pink Sheets securities Nasdaq securities Options Futures Open-end investment company shares

I, III, V only Nonmarginable securities include those found in the Pink Sheets, on the OTCBB, as well as options, new issues, including IPOs, and mutual fund shares.

Ted set up a 529 plan for his daughter Nicky in 2004. Performance in the account has been mediocre. Which TWO of the following options does Ted have with respect to making changes in the account? Roll over the funds into a Coverdell Education Savings Account Set up a new and separate 529 plan Roll over the funds into another 529 plan Maintain the same 529 plan as required

II and III A rollover of a 529 plan is permitted every 12 months. In rolling over funds from this type of plan an investor would be moving the funds to another state's plan. Generally, there are no residency requirements for a 529 plan. A 529 plan may not be rolled over to a Coverdell Education Savings Account.

An investor purchases a mutual fund in a joint brokerage account and receives a dividend, which the investor decides to reinvest in the fund. Which TWO of the following statements are TRUE regarding the tax consequences of this transaction? Reinvested dividends are not subject to taxation. Reinvested dividends are subject to taxation. Any dividend received, whether reinvested or not, is subject to taxation. Qualified dividends are not subject to taxation.

II and III All dividends, interest, and capital gains distributed from a mutual fund are subject to taxation unless the investment is purchased within a tax-sheltered account such as a 401(k) or individual retirement account. Qualified dividends are subject to taxation.

A portfolio manager is interested in purchasing bonds, but is concerned about an increase in interest rates. In order to make the portfolio less price sensitive to yield changes, the manager does which TWO of the following? Buy bonds with a long duration Buy bonds with a short duration Buy bonds with a high coupon Buy bonds with a low coupon

II and III Bonds with short durations and high coupons are less price sensitive to changing interest rates. On the other hand, bonds with long durations and low coupons have more price volatility as interest rates fluctuate.

For an investment adviser, which TWO of the following situations are considered conflicts of interest? The adviser purchases the same securities for its own account that it currently holds in several client accounts The adviser sells securities from its own account to an advisory client The adviser is short a stock that it is currently recommending for its clients to purchase The adviser has a position in securities that it is not recommending to its clients

II and III If an investment adviser enters into a principal transaction (sells securities from its own account to an advisory client), it is considered a conflict of interest since the adviser is on the other side of the transaction. This conflict must be disclosed to the client and the client must consent to the principal transaction prior to each transaction. In addition, when an adviser takes a position that is inconsistent with the recommendations that it makes to its clients, this conflict must be disclosed to its clients.

Which TWO of the following clients may enter into an advisory contract that includes a performance-based fee? A joint account with a net worth of $1,000,000 A partner of the investment adviser with annual income of more than $200,000 An IRA account with $2,000,000 under management An individual account with $750,000 net worth

II and III Under the Investment Advisers Act of 1940, performance fees are generally prohibited. Exceptions include contracts for clients who have at least $1,000,000 under management with the adviser or who have a net worth in excess of $2,000,000. Performance-based fees may also be charged to a client who is an executive, a partner, or a knowledgeable employee of the adviser. The amount of funds under management is not a factor for these clients. The type of account (individual, joint, or IRA) is also not a factor.

When opening a margin account, which TWO of the following MUST be signed? A loan consent form A hypothecation agreement A margin account form Trading authorization

II and III When opening a margin account, a customer must sign both a hypothecation agreement and a margin account form. The loan consent form is used when a customer authorizes the member to lend his securities, and is not required. Trading authorization is also not required

According to NASAA's Statement of Policy on Unethical Business Practices, which TWO of the following statements are TRUE concerning information to be included in an investment advisory contract? The fee for managing equity securities may not be higher than for fixed-income securities. An assignment of the contract can be made only by the investment adviser with the consent of the client. There is disclosure explaining that no prepaid fees will be returned if the contract is terminated. There is disclosure as to whether the contract grants discretionary power to the adviser.

II and IV All fees and services provided The term of the contract A formula for computing the advisory fee The amount of prepaid fees to be returned in the event of an early termination of the contract The fact that no assignment of the contract will be made without the consent of the client Whether the contract grants discretionary power to the adviser The fee for managing equity securities may be higher than for fixed-income securities Choice (II) is true since the contract may be assigned with the consent of the client. Choice (III) is not true since some amount of prepaid fees should be returned if the contract is terminated.

An adviser is comparing two bonds of similar credit quality and duration for a client. The client is seeking a yield of 7.2%. After performing discounted cash flow analysis on each bond, the adviser has determined that Bond A is trading at a premium to its present value, while Bond B is trading at a discount to its present value. Which TWO of the following statements are TRUE? Bond A is priced attractively and should be purchased. Bond B is priced attractively and should be purchased. The investor will earn an annual interest rate greater than 7.2% with Bond A. The investor will earn an annual interest rate greater than 7.2% with Bond B.

II and IV Discounted cash flow (DCF) analysis evaluates the present value of all coupon payments and the repayment of a bond's principal at a present value, based on a rate of return. This makes it possible to evaluate a bond's value against the investor's desired rate of return. The sum of each of the discounted cash flows, plus the present value of the bond's principal, determine the total value of the bond. By comparing this value to the current price of the bond, the adviser will be able to determine if the bond is an attractive investment for a client. If a bond is trading at a discount to its present value, the investor will earn more than the interest rate that has been used to calculate the present value. Conversely, a bond that is trading at a premium to its total present value will be worth less than the price of the bond. (The investor would be overpaying for the bond.)

Which TWO of the following investments impose income limits on contributors? A Traditional IRA A Coverdell ESA A 529 plan A Roth IRA

II and IV Not all persons are eligible to make contributions to a Coverdell ESA and a Roth IRA. For both Roth IRAs and Coverdell ESAs, a person cannot contribute if her adjusted gross income is above a certain limit, which is adjusted for inflation. There's no income limit imposed on contributors to 529 plans. Also, a person can make contributions to a traditional IRA as long as she has earned income.

Under the Investment Company Act of 1940, which of the following situations requires shareholder approval? A mutual fund enters into a contract with an underwriter. A mutual fund adviser wants to deviate from its investment policy. A mutual fund wants to terminate the existing contract with its investment adviser. A mutual fund wants to use the fund's assets to pay for the cost of distributing shares.

II and IV The Investment Company Act of 1940 requires shareholder approval for a company to make changes to its investment policy statement or to have the cost of underwriting or distributing shares paid from the fund's assets (12b-1 fees). The other two choices involve decisions that are made by the fund's board of directors. To terminate its contract with an adviser, the board must approve of the action. On the other hand, to establish a contract with an underwriter, approval by a majority of the board's disinterested members is required

An IAR is analyzing various fixed-income securities for a client's portfolio. She notes that the yield curve is normal. This indicates that: Short-term bonds are yielding more than long-term bonds Short-term bonds are yielding less than long-term bonds The economy is expanding too rapidly Long-term bonds are yielding more than short-term bonds Short-term bonds and long-term bonds are moving toward parity

II and IV only A normal yield curve occurs when yields on long-term bonds are greater than yields on short-term bonds. Another way to understand normal yield curves is to remember that the longer the maturity of a bond, the more the yield investors demand for the additional time risk. Normal yield curves are generally understood to indicate a healthy economy. Alternate terms for a normal yield curve are positive, ascending, or upward sloping.

A corporate bond is purchased at its par value of $1,000 and later sold at a discount. This would be indicative of which of the following risks? Opportunity risk Credit risk Currency risk Interest-rate risk

II and IV only The most likely the reason for the bond's price decline is that interest rates have risen. The risk of interest rates moving against a bond investor is referred to as interest-rate risk. Another possible explanation for the bond losing value is that its credit rating fell (credit risk).

Writing covered call options provides a client with: The potential for unlimited gains The potential for unlimited losses More potential income The potential to increase return

III and IV only When an investor owns the underlying stock and sells a call option on the same stock, the position is referred to as a covered call. Covered call writing generates income for investors. They receive the proceeds from the sale of the option and increase their potential return. Covered call writing is considered a conservative option trading strategy that generates current income and hedges the stock position to the extent of the premium received.

Ken is an IAR who manages the portfolio of a prominent local oncologist. The doctor's last few investments have been winners. The doctor has just instructed Ken to buy 100,000 shares of Bio-Transmedia, a thinly traded biotechnology micro-cap stock specializing in cancer research. Based on the doctor's prior record, Ken decides to purchase a few hundred shares for his personal account. Which of the following statements is TRUE

If Ken buys the stock, he may be violating his obligation as an adviser Advisers have a fiduciary responsibility to act in the best interests of their clients. In this case, Ken may be guilty of trading ahead of his client. Trading ahead is the practice of buying or selling stock with the knowledge of a pending client order, which could move the market. Ken is hoping that he could purchase stock cheaply and the doctor's large order will subsequently drive up the stock's price. Ken's purchase would not be considered insider trading since he has no indication that the doctor is in possession of material, nonpublic information.

Ken is an IAR who manages the portfolio of a prominent local oncologist. The doctor's last few investments have been winners. The doctor has just instructed Ken to buy 100,000 shares of Bio-Transmedia, a thinly traded biotechnology micro-cap stock specializing in cancer research. Based on the doctor's prior record, Ken decides to purchase a few hundred shares for his personal account. Which of the following statements is TRUE?

If Ken buys the stock, he may be violating his obligation as an adviser Advisers have a fiduciary responsibility to act in the best interests of their clients. In this case, Ken may be guilty of trading ahead of his client. Trading ahead is the practice of buying or selling stock with the knowledge of a pending client order, which could move the market. Ken is hoping that he could purchase stock cheaply and the doctor's large order will subsequently drive up the stock's price. Ken's purchase would not be considered insider trading since he has no indication that the doctor is in possession of material, nonpublic information.

If an investment increases in value, which of the following statements would be TRUE?

If it was held for less than one year, the annualized rate of return would be greater than the holding period return The holding period rate of return states how much an investor earns over the period an investment is held. The annualized rate of return states how much an investor makes over a one-year period. If an investor had a 5% rate of return over six months, her holding period rate of return would be 5%; however, her annualized rate of return would be 10% (the 5% return earned over the six-month period multiplied by two). If the holding period had been more than one year, the opposite would be true--the holding period return would be larger than the annualized rate of return.

When is an IA or IAR permitted to publish a testimonial regarding the adviser?

If the testimonial appeared on an independent social media site over which the IA or IAR has no control An IA or IAR is permitted to publish testimonials if (1) they are shown as they originally appeared on an independent third party social media site, (2) are unedited, and (3) the IA or IAR has no direct or indirect influence or control over the independent site.

Regarding the taxation of an estate, which of the following statements is TRUE?

In a revocable trust, assets are included in the estate for federal estate tax purposes. In a revocable trust, assets are typically included in an estate for estate tax purposes. One of the advantages of an irrevocable trust is that its assets are excluded from the estate for estate tax purposes. Property that's left to a spouse is not subject to the estate tax. If an estate is subject to tax, taxes are paid by the estate itself, rather than the beneficiaries.

A client buys shares of a closed-end fund:

In the secondary market Although a closed-end fund has a calculated NAV, it is unlikely that its shares will trade at that price. Instead, the shares may be purchased either above or below the NAV. The shares of a closed-end fund trade in the secondary market. This is unlike an open-end management company (mutual fund) whose shares constantly remain in the primary market.

When investing in a 401(k), tax is deferred on which of the following?

Income Qualified accounts, such as 401(k) plans, provide for a deferral of tax on income only. Participants are still required to pay any other applicable taxes in the year their contribution is made. Earnings, such as capital gains, dividends, and interest, will also be deferred until they are withdrawn.

As the result of a legislative change, municipal bonds have increased in value. Which of the following BEST describes the reason for this occurrence?

Income tax rates have increased Legislative risk is the risk that changes in legislation will materially impact investment returns. When income tax rates increase, investors are required to pay taxes at a higher rate. If this happens, investors will seek investments that provide tax-free returns (e.g., municipal bonds). Therefore, as the demand for municipal bonds increases, so too will the value of these investments.

An investor is long a 3x Bearish Inverse Leveraged Nasdaq 100 Index ETF. If the index declines by 10%, the value of the ETF will:

Increase by 30% For inverse leveraged ETFs, their value should move in the opposite direction of the underlying index by the given leverage factor (e.g., 3x). In this question, if the index declines by 10%, the value of the ETF will increase by three times that amount, (i.e., 30%).

The TopDollar Mutual Fund group is forming a new international fund. The fund intends to mirror the composition of the MS Capital International Index (MSCI). The portfolio manager will attempt to match the performance of the MSCI. This strategy is called:

Indexing An indexing strategy attempts to match the annualized returns of a portfolio to a widely used market benchmark index. This is done by creating a portfolio that closely mirrors the composition of the index.

Which of the following asset management techniques would NOT be used to identify a security that is either undervalued or overvalued?

Indexing When a portfolio is indexed, the manager is attempting to match the performance of the market. An indexed portfolio is one with a composition that mirrors a benchmark index. Fundamental, technical, and tactical approaches all attempt to identify securities that are either overvalued or undervalued and use this information in an attempt to outperform the market.

A sector rotation strategy would include investing in which of the following?

Industrial stocks in an expanding economy A sector rotation strategy involves investing in businesses that will grow along with the economy. Industrial stocks are cyclical and will rise and fall with the economy. The best time to buy technology stocks is right before an expansion, not during a contraction. Commodities help protect against inflation, but inflation is generally not anticipated if the market is peaking. Consumer goods companies make staples (e.g., groceries and household products) and would generally perform well during a recession. (67731)

In order to better diversify a client's portfolio, an investment adviser recommends that its client invest in tangible assets, such as gold and silver. Which of the following risks is the adviser attempting to reduce?

Inflation risk When an adviser recommends that clients invest in tangible assets, such as precious metals (gold) and real estate, its general purpose is to help them hedge against inflation.

Mark is a registered representative and also an investment adviser representative for his firm. One of Mark's clients requests that he create a comprehensive financial plan for her. To implement the plan, Mark must:

Inform his client that he will be acting as a registered representative when implementing the plan and will be receiving compensation Investment advisers and investment adviser representatives must disclose all relevant facts so their clients can make informed decisions regarding potential conflicts of interest. If an adviser acts in more than one capacity when dealing with a client, this may present a conflict of interest. In this case, Mark should inform his client that he will be acting in more than one capacity (IAR when creating the plan and RR/agent when implementing the plan) and will be receiving additional compensation.

When determining the risk tolerance of a client, which of the following choices will an investment adviser representative consider to be the LEAST important?

Information about the client's life insurance policies When determining a client's risk tolerance, the least important factor (of the choices given) for an IAR to consider is information regarding the client's life insurance policies. However, all of the other choices could potentially influence a client's risk tolerance, such as the person's financial status, goals/objectives, and investment experience. For example, experienced clients who have invested in different, unique types of securities are better able to understand how the various types of risk will affect a security's value.

The purchase of which of the following securities by an investment adviser for its own account would MOST likely involve a conflict of interest?

Initial public offerings Initial public offerings (IPOs) are often available only in limited amounts. If such offerings are available to the adviser and are also suitable for accounts of certain customers, these customers should be given priority.

An insurance agent works in an office building down the hall from a broker-dealer. They are not affiliated. What compensation may the agent receive from the broker-dealer in exchange for referrals?

Insurance referrals Only individuals who are registered agents may receive monetary compensation from a broker-dealer based on the sale of a security. In this scenario, the insurance agent is not licensed and may not receive compensation in the form of commissions or fees, or soft-dollar compensation (no cash compensation) in the form of discounted commissions. If a broker-dealer wants to refer its clients to an insurance professional, this is permitted. Only licensed insurance professionals may be compensated for the sale of life insurance. (

An investor purchased a 6%, A-rated, corporate bond at par value. After one year, the bond's total return is actually 6.50%. The most likely reason for this is:

Interest rates have decreased The formula for calculating a bond's total return is interest received, plus appreciation, minus any depreciation, divided by the original cost of the investment. Since the 6% bond was originally purchased at par, but now has a total return of 6.5%, it is indicative of the bond's value having increased. Based on the inverse relationship between bond prices and interest rates, a bond's value will increase as market interest rates decrease.

A client deposits $20,000 and has expectations of an investment's future value including both income and expenses. What rate will the client's IAR use to calculate a net present value of zero?

Internal rate of returnThis question is asking about an investment for which the client knows the present value ($20,000) as well as the future value. Although the future value is not given directly, the question states that the investor "has expectations" about the investment's future income and expenses. Therefore, the only number that is not provided is the rate of return for the investment. The internal rate of return on an investment is the rate of return that makes the future value of an investment equal to its present value. Another way to think about the IRR is that it will make the net present value equal to zero (i.e., NPV = FV - PV). (89102)

In a down market, which of the following is a risk of using dollar cost averaging (DCA)?

Investments are not protected against losses Dollar cost averaging involves making periodic investments of a fixed dollar amount into stocks or mutual funds. When prices fall, more shares will be purchased at a lower cost, which ensures that an investor's average cost will be less than his average price. This will increase performance when the market once again rises. However, investors should not sell and must continue making purchases during down markets. When prices fall, investors are not protected against losing money.

What's a basic assumption of Modern Portfolio Theory?

Investors will minimize risk when possible Modern Portfolio Theory (MPT) is based on the following two assumptions: 1) Investors will try to maximize their returns, and 2) Investors generally seek to assume as little risk as possible. Although many investors will take less risk as they age, MPT doesn't specifically mention an investor's age. Also, MPT doesn't make any assumptions about the market or the economy.

According to the NASAA Custody Requirements for Investment Advisers Model Rule, an investment adviser that intends to send account statements directly to its clients:

Is permitted to do so if the IA is audited by an independent CPA According to the NASAA Custody Requirements for Investment Advisers Model Rule, an investment adviser that intends to send account statements directly to its clients is permitted to do so if the IA is audited by an independent public accountant. Advisory clients must be provided with account statements on a quarterly (not monthly) basis.

A complex trust:

Is permitted to retain some of its annual investment income A complex trust is permitted to retain some of its investment income. (In a simple trust, this income must be distributed to the beneficiaries in the year received.) Trustees of a complex trust are also empowered to distribute principal. The term complex has nothing to do with the number of beneficiaries of the trust, the use of derivatives within the trust, or the employment of an outside portfolio manager by the trustee.

Regarding the yield-to-maturity of a bond, all of the following are TRUE, EXCEPT:

It is always equal to the yield-to-call Callable bonds are able to be repurchased by the issuer earlier than anticipated (i.e., they will not reach maturity). Investors buying callable bonds must be aware of both the yield-to-maturity and the yield-to-call. If the bond is trading at its par value, the two yields will be the same. However, if the bond is trading at a discount or premium, the two yields will be different.

A, B, and C are the general partners of the ABC partnership, which they established to operate a commercial ostrich-raising farm. The farm will raise ostriches and sell their meat and eggs to stores and restaurants that specialize in exotic foodstuffs. Each partner contributed $50,000 in cash to the venture. Partner B also contributed a parcel of farmland, the fair market value of which is $100,000, and which has an outstanding mortgage of $50,000. What is the impact of B's contribution of the farmland on his cost basis in the partnership?

It is increased Investor B has increased his basis in the partnership by contributing property. Generally, an investor's basis in a partnership increases whenever he contributes additional cash or property (real or personal) to the partnership. Note, however, that his basis is increased $50,000, not $100,000. The partnership assumes responsibility for the property's mortgage. Investor B is relieved of that responsibility. Investor B's cost basis will be increased by the value of his equity in the property ($100,000 fair market value - $50,000 mortgage).

All of the following statements are TRUE of the death benefit of a variable life insurance policy, EXCEPT:

It may be reduced to zero by poor performance of the separate account Although the death benefit of a variable life policy may increase or decrease due to the performance of the separate account, it will not decrease below a minimum guaranteed amount (the face value of the policy).

Which of the following choices is NOT a benefit of discounted cash flow, fixed-income analysis?

It permits an adviser to minimize cash flow reinvestment risk Discounted cash flow (DCF) analysis does not offer relief from reinvestment risk when investing in fixed-income securities. A discounted cash flow evaluates the present value of each coupon payment and the repayment of a bond's principal at a present value, based on a rate of return. This makes it possible to evaluate a bond's value against current rates of return. The sum of each of the discounted cash flows, plus the present value of the bond's principal, determines a fair market value of a bond. By comparing the value calculated by the discounted cash flow formula to the current market price of the bond, the adviser will be able to determine if the bond is an attractive investment for a client. (

What's the benefit to creating a will?

It reduces estate taxes and speeds up the probate process Wills are legal documents that provide directions for managing an estate after a person's death. Although wills make the probate process quicker and easier, all estates must go through probate. By distributing assets, wills can minimize estate taxes; however, they may not always be able to be avoided altogether.

All of the following statements are TRUE about zero-coupon bonds, EXCEPT:

It would be a suitable investment for a customer who needs cash flow A zero-coupon bond is purchased at a discount and does not pay periodic interest. All interest is paid at maturity (the difference between cost and par value). The discount must be accreted each year and is treated as ordinary income even though it was not actually received. Since interest is not paid each year, a zero-coupon bond would not satisfy an investor's need for cash flow.

Which of the following is TRUE of a structured note?

It's a bond issued by a financial services company which offers a rate of return that's linked to other securities A structured note is typically a debt instrument (e.g., note or bond) which offers a rate of return that's linked to an underlying security, basket of securities, or derivative product (e.g., an option). Since the structured note is linked to another underlying security, the rate of return will fluctuate based on the performance of that security. Exchange traded notes (ETNs) are one of the most well known structured notes.

If a bond is currently selling for less than par value, then:

Its current yield is higher than its nominal yield Bond yields and prices have an inverse (opposite) relationship. This means that that as one increases, the other will decrease. Therefore, if a bond is selling at a discount (below par), its current yield will be higher than its nominal yield. For example, an investor owns an 8% bond that's trading at $850. Of course, the nominal yield is 8%. The bond's current yield is found by dividing the annual interest by the market price. An 8% bond pays $80 per year (assuming a par value of $1,000); therefore, the current yield is 9.41% ($80 ÷ $850).

What main characteristic qualifies an American-style equity option to be described as a derivative security?

Its value is based on the valuation of another asset Derivative contracts (e.g., options) obtain their value from the price/value of an underlying asset. Derivatives are often priced based on both intrinsic and time value. Futures and swaps are types of derivatives whose owners have no exercise rights. Simply being risky does not qualify a security as a derivative. American-style options are able to be exercised by the owner at any time prior to its expiration. However, European-style options may only be exercised by the owner on the last trading day prior to expiration.

Jake purchased a corn futures contract at $1.20 per bushel. At the end of the contract, if the price of corn has fallen to $1.10 per bushel, which of the following statements is TRUE?

Jake takes delivery As the purchaser of the contract, Jake has an obligation to take delivery. On the other hand, the party that initially sold the contract has an obligation to make delivery.

Perry is a 27-year-old paralegal who makes $35,000 annually. His law firm has a 401(k) plan and the firm matches employee contributions, but Perry does not participate in it, although he is eligible. He wants to save for retirement and is considering a variable annuity. Perry consults an investment adviser representative. The most appropriate advice that the IAR can give Perry is that he should:

Join his employer's 401(k) plan By joining the 401(k), Perry can save for retirement on a pretax basis. In addition, his employer matches his contributions, which will help his savings grow faster. Payments to a variable annuity would have to be made on an after-tax basis. Generally, people investing for retirement should exhaust their ability to contribute to vehicles (such as 401(k)s or IRAs) that allow them to save money on a pretax or tax-deductible basis before investing in variable annuities.

If required life insurance premiums are not paid on time, a policy will:

Lapse The failure to pay life insurance premiums will cause the policy to lapse. Some policies allow policyholders to borrow against their cash value to pay premiums. However, the policyholder is not allowed to skip a payment. Since cash value will not automatically be paid out, it is usually forfeited.

Which of the following business structures allows the pass-through of income and loss to its members on a Form K-1?

Limited liability company (LLC) Each of the answer choices provide for the pass-through of income for tax purposes to investors. However, only limited partnerships, LLCs, and S-corporations will report their income on a Form K-1. The reason that LLCs is the answer is because they refer to their owners as members. Owners of an S-corporation are referred to as shareholders, while owners of a limited partnership are referred to as partners.

Which form of business would offer limited liability and the flow-through of profits and losses?

Limited partnership The only choice that provides for the flow-through of income and the limited liability for some owners is a limited partnership. Unlike limited partners, the general partners in a limited partnership have unlimited personal liability. A characteristic of both general partnerships and sole proprietorships is that they create unlimited liability for all owners. Corporations do not have the benefit of the flow-through of income.

Market analysts are predicting a bearish trend in the market. Which of the following strategies could the portfolio manager of an aggressive growth mutual fund use to protect the fund's net asset value?

Liquidate current stock positions and reinvest the funds in short-term debt obligations A portfolio manager can protect against a bearish move in the market by transferring portfolio assets into cash, that is, money-market instruments. These short-term debt obligations are ideal for the objective of preserving capital. When the anticipated bearish move is over, the manager can move the assets in cash back into the securities the fund normally owns.

Which of the following is the BEST hedging strategy if a client is long 1,000 shares at $42?

Long 40 puts If the investor wants to hedge against downside moves in a stock, he should buy put options. Purchasing the out-of-the-money put options is cost-effective (lower premium) and therefore is an efficient hedging strategy. Buying calls on stock owned is considered a bullish strategy. Selling or shorting call options only provides downside protection to the extent of the premium received.

Jack purchased 100 shares of XTRO at $20. After nine years, he gave the shares to his nephew Sam when the fair market value of XTRO was $16 per share. Sam held the stock for seven months and then sold the shares for $23 per share. What is the tax consequence for Sam?

Long-term capital gain of $300 If securities are received as a gift, any tax implication is delayed until the securities are subsequently sold. For the recipient, the two details that should be determined are 1) the donor's cost basis and 2) the fair market value (FMV) at the time of the gift. One of these two will represent the recipient's basis for determining a capital gain or loss at the time of sale (referred to as dual basis), as described below: ****If the securities are later sold for a price that is *higher* than the donor's cost, the *seller's basis is the donor's cost* and the donor's holding period is included. If the securities are later sold for a price that is *lower* than the FMV at the time of receiving the gift, the *seller's basis* will represent the FMV and the holding period begins on the day after the gift is received. When securities are given as a gift after they had appreciated in value and the recipient subsequently sells them for a gain, the recipient's basis is the lesser of the donor's cost or the fair market value at the time of the gift (i.e., always the donor's cost). However, this is a tricky question since the securities were gifted at a time when the fair market value ($16) is less than the donor's cost ($20). Sam later sold the stock for a price that was higher than Jack's cost. For that reason, Sam will use the $20 cost as his basis against the proceeds of $23 on the sale, which results in a capital gain of $3 per share, or $300. In this situation, Sam is able to add Jack's holding period to his own (nine years plus seven months) to establish a long-term holding period and therefore he realizes a long-term gain.

As an employee of a publicly traded company, Jack has sold his company's stock short and then began spreading a rumor that his company is using inferior materials and will likely fail. After investors sell their stock, he covers his short position at a profit. This is an example of:

Manipulation Although this question doesn't specify Jack's position with the firm, he's an employee and his activities are manipulative. Due to the fact that he spread rumors and profited from them, he has violated the Securities Exchange Act of 1934 and is subject to prosecution.

The tax rate that applies to the last dollar that a person earns is called the:

Marginal tax rate The marginal rate is the rate at which the last dollar a person earns is taxed. In the U.S., a person's marginal tax rate (tax bracket) will rise as her total income increases.

Tony had two children, Susan and Mark, who each had three children of their own. Susan passed away at an early age. What's the result if Tony dies before Mark as well as his grandchildren and his will called for a per capita distribution?

Mark receives 25%, while Susan's surviving children will each receive 25%. With a per capita (by head) distribution, all surviving members are entitled to an equal share. However, only one generation of beneficiaries will be counted and is known more specifically as per capita, per generation. In this question, since Mark is still alive, his children are not counted as additional beneficiaries. Susan has passed away and therefore the next generation (her three children) will receive benefits. Ultimately, there are four heads (beneficiaries) being counted—Mark (Tony's surviving child) and Susan's three children. Mark receives 25% of the estate, while Susan's children will each receive 25%. If per stirpes distribution was being used, Mark would receive 50%, while the remaining 50% would be evenly split among Susan's children.

If a broker-dealer is publishing both bid and ask prices for securities in the secondary market, it's acting as a:

Market maker Market makers are firms that act as dealers in offering to buy and sell securities in the secondary market. When doing so, market makers assume their own risk.

After analyzing the financial sector, an analyst writes a report indicating that he expects a downturn in that sector. This could BEST be described as:

Market risk A downturn in the prices for an entire sector of the economy is a form of market risk. Business risk is the risk that one company, not an entire sector, will perform poorly.

An adviser is permitted to store records on:

Microfilm or Microfiche Books and records must be maintained in an easily accessible place for five years. During the first two years, the records must be maintained in an appropriate office of the investment adviser. Records may be preserved on microfilm, microfiche, or any similar media. They may, but are not required to, be kept on various electronic storage media such as CD-ROMs, provided the disks are tamper-evident, i.e., the information cannot be easily altered. This means that any attempt to alter the records would be easily determined upon examining them. These files do not need to be password-protected, but the adviser must be able to limit access to the records to authorized personnel and the regulators.

A mathematical technique that uses randomly generated scenarios, known as simulations, to determine the probability of possible returns, is known as the:

Monte carol theory Monte Carlo is a technique which uses randomly generated scenarios, called simulations, to attempt to determine the probability of possible returns. It is one of several computer programs that has been developed recently to give investors different tools to help them manage their portfolios.

An efficient market is one with:

Narrow or small spreads between bid and ask prices Liquid or efficient markets tend to have a large amount of trading activity and narrow spreads (bid and ask prices that are close to each other). While narrow spreads are a sign of an efficient market, wide spreads are a sign of an inefficient market.

When may an investment adviser representative project the future return of a variable annuity?

Never It is prohibited to project the future earnings in a variable annuity separate account.

A client purchased orange juice futures at $1.70 per pound. Two months later, if orange juice has increased to $1.95, the client has:

No realized gain or loss This is a tricky question. Since the question does not indicate that the contract has been closed out, there is no realized gain or loss. However, if the client sells, closing out the position, she would realize a profit if orange juice is trading above $1.70.

The interest rate that is stated on a bond's certificate is also referred to as the:

Nominal yield The nominal yield is the stated rate of interest found on the face of a bond, while a bond's internal rate of return is its yield to maturity. A bond's real rate of return is found by subtracting the rate of inflation from the bond's internal rate of return.

An individual represents an issuer in the sale of the issuer's securities to its employees, but does not earn commissions on the transactions. The individual is:

Not considered to be an agent of the issuer An agent is an individual who represents a broker-dealer or an issuer in effecting securities transactions. However, an individual who represents an issuer in a transaction with existing employees and does not receive commissions is NOT considered to be an agent. In this question, the individual does not fall under the definition of either a broker-dealer or an issuer.

Joshua is employed by JTR Investments. He holds Series 7 and 66 registrations. He also recently organized his own corporation through which he plans to sell promissory notes. Before he begins selling these notes to investors, Joshua must:

Notify JTR Investments in writing and receive its written permission Joshua must notify JTR about these proposed transactions in writing and receive its written permission before he begins selling promissory notes. (Some registered representatives have tried to argue that promissory notes are not truly securities and, therefore, do not require them to notify their firms and receive written permission). The regulators have rejected this argument warning registered representatives that financial instruments such as leasing arrangements or promissory notes count as securities for the purpose of this rule.

Dunn and Associates is a registered investment adviser that is organized as a partnership. Jon Dunn, the senior partner who owns 50% of the firm, has just announced that he is retiring next month. What must this RIA do?

Notify all its clients within a reasonable period The Investment Advisers Act states that if an investment adviser is formed as a partnership, its client contracts must contain a provision that it will notify clients of any changes in the partnership within a reasonable period. (

An agent solicits the purchase of MPH, Inc, a nonexempt, unregistered security. The agent requests the client sign a document, acknowledging the security's status. The document also includes an exculpatory provision absolving the agent and the broker-dealer from any liability or wrongdoing. The waiver the client signed is:

Null and void Agents must not solicit nonexempt, unregistered securities nor should they request a client sign documents absolving the agent or broker-dealer from wrongdoing. Such statements are sometimes called exculpatory clauses and are prohibited. These documents would be null and void under the Uniform Securities Act.

A broker-dealer advertises on a radio program that is broadcast from a bank. Which of the following would be prohibited by the Administrator?

Omitting the name of the broker-dealer in any 30-second ads during the show Broker-dealers and agents are not allowed to publish a blind ad unless it is a recruiting advertisement for a new hire. The name of the broker-dealer that approved the ad is generally required on all advertisements. Broker-dealers are allowed to mention that they are affiliated with or subsidiaries of banks. They must make the distinction that the products they offer are neither deposits nor are they guaranteed.

As of the close of business on Monday, a state-regulated IA has fallen below its minimum financial requirement. When must the IA file a statement of financial condition?

On the business day following the date that it reported the issue to the state Administrator. If an IA falls below the minimum financial requirement, it must provide notice to the state Administrator by the next business day. The IA must then file a statement of its financial condition by the next business day after providing notice. The statement of financial condition includes: A trial balance A statement which indicates all client funds or securities which are not segregated A computation of the aggregate amount of client ledger debit balances A statement as to the number of accounts

Which of the following would NOT be considered an income strategy?

One that always excludes covered call writing Eliminating covered call writing would not be appropriate for an income strategy. Writing covered calls is a conservative method of generating additional income in a portfolio. All of the other choices are consistent with an income strategy.

According to NASAA's Statement of Policy on Dishonest or Unethical Business Practices of Broker-Dealers and Agents, when may a broker-dealer exercise discretion in a customer account?

Only after the firm has received the customer's written consent Broker-dealers cannot exercise discretion without the customer's written consent. Not-held orders are the one exception for broker-dealers. With not-held orders, a brokerage firm may determine the time and/or price of execution without a client's written authority. Investment advisers (but not broker-dealers) may accept verbal authority from a client, but only for up to 10 days

An IA may charge a client an investment advisory fee for rendering investment advice while receiving compensation for effecting securities transactions related to such advice:

Only if disclosed to the client Under NASAA's Model Rule on Prohibited Conduct of IAs, IARs, and federal covered advisers, this practice is acceptable if it is disclosed to the client before effecting transactions pursuant to the advice.

May an investment adviser (IA) share in the profits and losses in a customer's account?

Only if the client agrees and signs a written contract While investment advisers generally cannot be paid from, or share in, capital gains in their customers' accounts, exceptions do exist. However, if investment advisers are sharing in their clients' profits and losses, they must have written agreements in place with the clients.

An investment adviser is both registered in and located in State A. One of the firm's IARs has three non-institutional clients and one institutional client in State B. A different IAR of the firm has four non-institutional clients in State B. If the investment adviser does NOT have an office in State B, who must register in State B?

Only the investment adviser An advisory firm that has more than five non-institutional clients residing in a state is required to register in that state. Since one IAR of the firm has three non-institutional clients and the another IAR of the firm has four non-institutional clients in State B, the investment adviser has a total of seven non-institutional clients in the state and must register there. However, since neither IAR has more than five non-institutional clients in the state, neither one needs to register in State B. Notice that all persons (the IA and both of the IARs) need to be registered in State A.

An agent of a broker-dealer is also employed as an investment adviser representative. A client asks the agent for assistance in placing the shares of his start-up company's initial public offering (IPO). Under what conditions may the agent accept the client's offer of commissions for completing the offering?

Only with the written permission and supervision of the broker-dealer This is an allowable action provided written permission is obtained from the supervising broker-dealer and the transactions are recorded on the broker-dealer's books and records. If the agent fails to notify her firm, it is considered selling away, which is an unethical and prohibited business practice.

All of the following securities prices are quoted based on the market price, rather than NAV, EXCEPT:

Open end management companies Open-end management companies (mutual funds) and unit investment trusts (UITs) are valued each day, at the close of market, based on their net asset value. Closed-end management companies and ETFs trade in the market at either a premium or discount to their NAV, or they are quoted at their market value. Since hedge funds do not trade, their value is not transparent.

An investment adviser representative manages a portfolio for a client on a discretionary basis. The client's objective is conservative growth. According to prudent investor standards, which of the following statements is TRUE regarding the inclusion of options in his portfolio?

Options strategies may be appropriate as part of a conservative portfolio Prudent investor standards are applied to the client's total portfolio, not on an investment-by-investment basis. Rather than restricting individual investments, many securities may be appropriate as part of a portfolio if the investment helps the portfolio achieve specific objectives. For example, writing covered calls or buying protective puts could be part of a conservative growth portfolio. Whenever a reference is made to a position being "uncovered," this means that the option has been sold and the investor either doesn't own the stock (if a call has been sold) or doesn't have the necessary cash available (if a put has been sold). There's no such thing as the "purchase of uncovered options."

Which of the following securities have no loan value?

Options that have nine months or less until expiration Options that expire in nine months or less may not be bought on margin. These contracts have no loan value and, therefore, must be paid for in full. On the other hand, options that have maturities of greater than nine months have a margin requirement of 75% (i.e., customers must pay 75% of the cost and the firm may loan the remaining 25%). (89141)

What's a benefit to establishing a business entity as an LLC rather than as a sole proprietorship?

Owners can protect their personal assets The primary difference between these two structures is that owners receive some liability protection by incorporating as an LLC. This is important for business owners who have personal assets — especially if they use those assets to conduct some of their business. With a sole proprietorship, if the owner defaults on his loans or is found liable for damages in excess of the business's assets, he may be forced to liquidate his personal assets. Both LLCs and sole proprietorships have flow-through of income for taxes purposes, which is not available to traditional corporations.

An IA receives rebates from a broker-dealer for placing client orders with the broker-dealer. Such rebates may be used by the IA for all the following purposes, EXCEPT:

Paying the salary of a new assistant to help in processing customer orders Rebates or soft dollars may be used to acquire or pay for services or other expenses that assist the adviser in rendering investment advice. Paying the salary of any employee does not benefit the adviser's clients and soft dollars may not be used for that purpose. The other choices are expenses for items that can be used in providing advice to clients.Keyword=EXCEPT!!!

Which of the following distribution methods will ensure that each branch of family receives an equal share of an estate?

Per stirpes Per stirpes distributions are done per branch of a family and, in some instances, members of the same generation of a family will receive different amounts from an estate. A per capita distribution of an estate ensures that each member of a generation will receive the same amount as any other. Inter vivos and testamentary are terms that are associated with trust accounts, but not estates.

Which of the following statements is FALSE about universal life insurance policies?

Premiums are always invested in a separate account of the insurance company In universal life insurance policies, all premiums are deposited in the insurance company's general account. Variable policies/contracts use a separate account. Universal policies also have flexible premiums that may be increased or decreased over the life of the policy. While universal policies may have a guaranteed minimum rate of return, the return may fluctuate above the minimum.

Net present value is best described as:

Present value above cost The net present value (NPV) of an investment is the difference between its present value and its costs (i.e., current market price). The present value of an investment is typically found by taking expected benefits (i.e., future cash flows) and discounting them by using the present value formula.

An investment adviser is constructing a diversified portfolio for a client. If the client requires liquidity, which of the following asset class would he least likely recommend?

Prime real estate Of the choices given, prime real estate is the least liquid asset. Commodities options are a type of investment that are included in the derivative asset class.

If a federal covered adviser makes a material change to its Form ADV, it must notify the SEC:

Promptly (generally within 30 days) If an investment adviser makes a material change to its Form ADV, it is required to file an amendment with the SEC promptly (generally within 30 days). For non-material changes that are made to the information found in Form ADV, an adviser must file an amended within 90 days of the end of the IA's fiscal year.

At the time of entering into a contract with a client, an IAR intends to give the client a glossy, laminated copy of the IA's Form ADV Part 2 before it has been filed. Why is this action prohibited?

Providing the client with Form ADV Part 2 that has not yet been filed is a violation. An IAR is prohibited from providing clients with a Form ADV Part 2 (a brochure) that has not yet been filed with a state or federal regulator.

An initial public offering (IPO) is being sold in one state only and is not being submitted for registration with the SEC under the Securities Act of 1933. According to the provisions of the Uniform Securities Act, what method of registration would be used for this offering?

Qualification The notification and coordination methods of state registration may only be used when the issuer also files a federal registration statement under the Securities Act of 1933. The qualification method of registration may be used in any state for any issuer that is not seeking federal registration.

If an advisory client is most concerned with minimizing her tax liability, common stocks may provide a greater benefit than corporate bonds because:

Qualified cash dividends are taxed at a maximum rate that is less than the rate at which the interest on corporate bonds is taxed If an investor owns stock of a domestic corporation (or shares of a fund that contains these types of stocks) and has satisfied a holding period, any dividends that are distributed by the company (or fund) are considered a qualified cash dividends and are taxed at a maximum rate of 20%. On the other hand, the interest on corporate bonds is taxed as ordinary income (possibly as high as 39.6%). Capital gains are only taxable if they are realized (i.e., the investor sells the asset at a price that is higher than what he originally paid).

A client is in the 35% tax bracket. She has three children, ages 8, 12, and 16 and would like to invest in a 529 plan for the two oldest children. The client has $20,000 that she can invest in each account. If she anticipates her children will enter college at age 19, and will need $75,000 each for their college expenses over four years, an adviser would determine the future value of each account by inputting all of the following factors, EXCEPT the:

Rate of inflation The inflation rate is not a factor in the calculation of the future value of an investment. Future value calculation: Pn = P0(1 + r)n

Weldon Shalls, an investor, has a portfolio that provides him with a steady stream of income and some capital appreciation. Weldon believes he should diversify his portfolio by investing in an instrument that can provide additional income and the ability to absorb a portion of the investment's expenses. What type of investment vehicle will Weldon most likely choose?

Real estate limited partnership Investing in a limited partnership is considered a passive activity. Investors must understand the tax treatment for partnerships. Income and losses generated by the partnership is absorbed by the investor. All tax information is required to be reported on Form K-1 by the partnership.

Walck Asset Management has $67.5 million in assets under management. Under the Uniform Securities Act (USA), if Walck transacts business with clients in State A, it is:

Required to pay an initial and renewal filing fee to state A Advisers with assets of $110 million or more must register with the federal government and are known as federal covered advisers. Advisers with assets of $100 million up to $110 million may register with the federal or state government. Those with fewer assets generally fall under state jurisdiction. Since the Advisory firm has assets under management of less than $100 million, it must register with State A and pay a registration fee and annual renewal fee, as well as in any state in which the firm is required to register.

An IAR of a state-registered adviser is registered in State X, but gives seminars to potential clients in State C. The IAR is:

Required to register in State C if he directs solicitations to more than five noninstitutional clients An IAR of a state-registered adviser is subject to registration in any state where he has a place of business and or directs communication to more than five noninstitutional clients.

Which of the following items is not included in an income statement?

Retained earnings The income statement for a business will disclose its sales (revenues) less its expenses. Interest and taxes are both expenses which are reflected on a company's income statement. However, retained earnings reflects a company's historical earnings and is found on the company's balance sheet, not its income statement.

What does the Sharpe Ratio measure?

Return of an asset based on the amount of risk being assumed The Sharpe Ratio is a risk-adjusted rate of return which measures how much an investor earned on an investment compared with the risk being taken to achieve the return.

Value investing is:

Risky because it's contrarian When using a value investing approach, and investor's goal is to find and invest in companies that are intrinsically undervalued. Value investors tend to focus on company stocks that have low share prices, high dividends, low P/E ratios, and low price-to-book ratios. Value investors try to find stocks that are trading at prices lower than their intrinsic value (i.e., the market undervalues them). Put another way, value investors are betting against the market as a contrarian strategy.

Mary had two children, Donald and Sarah, who each had three children of their own. Donald and his spouse died in an accident. Mary's will calls for a per stirpes distribution. If Mary predeceases the remaining family members, the result would be:

Sarah receives 50% of the estate, while Donald's children split the other 50% of the estate. In a per stirpes distribution, each branch of a family receives an equal share of an estate. In this question, Donald and Sarah represent the two branches of Mary's family. After Donald's death, his surviving children now represent his branch, with Sarah still representing the other branch. Due to Donald's death, his three children will split the 50% of the estate, that would have been inherited by him, while Sarah is entitled to the other 50% of the estate since she is still living.

Several economic indicators show that the economy is headed into a recession. The manager of a mutual fund decides to reallocate the fund's portfolio, shifting money out of cyclical stocks and into consumer staples. This is an example of:

Sector rotation The economy tends to follow a certain cycle--early recession, full recession, early recovery and full recovery. Investment managers who use sector rotation try to anticipate the next phase in the cycle and shift assets into industries that will benefit or outperform the market as a whole. Thus, a manager who believed that the economy was headed into a recession might shift assets out of cyclical stocks (businesses that tend to do well in good times) and instead invest in the securities of companies that make consumer staples, businesses that are recession-proof, and whose revenues tend to remain stable even when times are bad.

When considering implementing a buy-and-hold strategy for a bond portfolio, a manager should consider which of the following?

Selecting bonds based on certain criteria and determining their appropriateness by comparing their prices to similar bonds When implementing a buy-and-hold strategy with bonds, portfolio managers generally consider whether the bonds are valued fairly in comparison to other similar bonds. Also, since the portfolio is comprised of bonds, the manager would focus only on bonds, not other general investments. Selecting only bonds that are callable or bonds with the same maturity is not desirable. Instead, laddered (staggered) maturities would generally be a better approach for the manager.

An advisory client believes that the economy is heading into recession. Which of the following is the most appropriate strategy in anticipation of a market decline?

Selling index futures Selling stock index futures is a bearish strategy which may be used by a speculator or a hedger. The other strategies that are listed are bullish and are not appropriate for an investor who anticipates a market decline.

Which of the following actions will require an adviser to register under the USA?

Selling investment management services Under the Uniform Securities Act (USA), firms that sell securities are typically required to register as broker-dealers, not investment advisers. On the other hand, firms that sell management services are required to register as investment advisers under the USA.

Under the Uniform Securities Act, which of the following choices is a nonexempt transaction?

Selling unregistered nonexempt securities The only nonexempt transaction in this question is selling unregistered, nonexempt securities. This would actually be a violation of the Uniform Securities Act.

An advisory client owns a small business and inquires about whether he should set up his business as a partnership or as a C Corporation. Which of the following would be the BEST reason to set the business up as a partnership?

Setting up a partnership is easier than establishing a corporation A partnership is easier to establish and operate than a C Corporation, which requires more reporting and administration. Partnerships generally do not allow for the free transfer of shares and if a partner manages the business, he may be liable for the debts of the partnership.

If a 62-year-old married woman is about to start taking Social Security payments, which of the following is TRUE?

She locks in the payment she will receive An individual may start taking Social Security benefits once she reaches age 62. However, if she waits, she will receive larger payments. Once an individual begins taking benefits, she locks in the base amount that she will receive. The age at which a person begins taking benefits doesn't prevent her from receiving cost of living adjustments.

An firm has been hired to be the investment adviser of the Western Vistas family of funds. The fund family is the firm's only advisory client and it currently has $18 million under management. Which of the following statements concerning the adviser's registration is TRUE?

Since it is a federal covered adviser, it must register with the SEC. Investment advisers are ultimately required to be registered at either the federal level with the SEC or at the state level with the state Administrator; there is no requirement for them to register at both levels. Although there are exceptions, the typical basis for determining whether state or federal registration is required is the amount of assets under management (AUM) for the adviser. However, an important exception applies to firms that serve as advisers to registered investment companies. Regardless of the amount of assets under management for the adviser, any adviser of an investment company is considered a federal covered adviser and is only required to register with the SEC. In this question, the Western Vista family of funds is an investment company.

An investment adviser representative is supervising a large, diverse portfolio for an elderly client. Although the account is nondiscretionary, the client almost always accepts the recommendations of the IAR. The portfolio contains a significant position in bonds that are denominated in foreign currency, and the IAR has become concerned the increased volatility in the currency market could damage the value of the client's investments. The IAR is thinking about recommending the use of foreign currency futures to hedge the foreign currency risk of the portfolio. Considering prudent investor standards, which of the following statements is TRUE?

Since there are no categorical prohibitions on types of investments under prudent investor standards, the use of futures could be appropriate for a portfolio under certain conditions The standard of prudence under the Uniform Prudent Investor Act is applied to the client's total portfolio, rather than on an investment-by-investment basis. Since there are no categorical restrictions on types of investments under the Act, any investment might be appropriate as part of a portfolio designed to achieve specific aims. In this case, the use of futures to hedge currency risk could be appropriate as part of an overall strategy for this elderly investor.

The Uniform Securities Act prohibits

Soliciting orders for unregistered, non exempt securities Soliciting orders for unregistered, non-exempt securities is prohibited by the Uniform Securities Act. The Act permits broker-dealers to maintain discretionary accounts for customers, to charge fees for services performed in customer accounts, and to accept unsolicited orders.

A state securities Administrator has decided that a broker-dealer is in danger of becoming insolvent. The Administrator orders the broker-dealer to maintain a minimum net capital of 200% of the federal minimum. How would this be viewed under NSMIA?

State Administrators are not allowed to impose requirements that are more stringent than SEC regulations Under NSMIA, state securities Administrators are not allowed to impose requirements that are more stringent than SEC regulations.

A federal covered investment adviser has an office in State A. The adviser also has five clients in State B and five clients in State C. A client files a complaint against the adviser, accusing the adviser of unsuitability, excessive trading, and misappropriation of funds. The Administrator(s) of which of the following states have/has the authority to take action against the adviser?

States A, B, and C The Administrators of States A, B, and C would have the authority to take action against the adviser because no one is exempt from the antifraud provisions of the USA.

When creating a portfolio for a client, an investment adviser first determines the client's investment objectives and risk tolerance. Based on this information, the adviser then constructs a portfolio containing specific percentages of uncorrelated investments. On a periodic basis thereafter, the adviser readjusts the portfolio to maintain the original investment mix. This approach is best described as:

Strategic asset allocation *The allocation of assets into an optimal portfolio based on a client's risk tolerance and investment objectives is called strategic asset allocation. In theory, it is the best mix of assets, given the client's goals and level of risk aversion. Most strategic asset allocators also periodically rebalance the portfolio to restore the original asset mix. *

An insurance company is considering raising capital by issuing bonds. Under the Securities Act of 1933, the bonds are considered:

Subject to registration with the SEC Under the Securities Act of 1933, securities that are issued by insurance companies are subject to both the SEC's registration requirements and its prospectus delivery requirements. However, the bonds are exempt from registration with the state Administrator. Keep in mind, no securities are exempt from the antifraud provisions of the Securities Act of 1933.

An IAR is asked to create a financial plan for a client. Although implementing the plan may generate a significant tax liability for the client, the IAR believes the plan fully meets the client's investment objectives. What should the IAR do?

Suggest that the client seek the advice of a qualified tax professional Since the plan fits the client's investment objectives, it should not be changed. However, the IAR does not seem to have tax expertise, so he should recommend that his client seek outside tax help. It would violate the IAR's fiduciary duty to let the client handle the issue by herself.

All of the following are types of market efficiency under the Efficient Market Hypothesis, EXCEPT:

Systematic The Efficient Market Hypothesis (EMH) explains three different forms -- strong, semi-strong, and weak form efficiency. EMH proposes that information which moves share prices is distributed fairly and quickly. Investors that believe that markets are efficient should invest passively (e.g., indexing) rather than actively (e.g., using technical analysis). The term "systematic" is often used to describe a category of investment risk, but it's not a form of efficient market.

An investment adviser, concerned that the financial services sector is going to decline in value, rebalances his client's investment portfolios. This form of asset allocation is known as:

Tactical asset allocation Tactical asset allocation is an active allocation strategy based on the anticipation of future trends or changes in economic events. Changes are made in the allocation of assets to take advantage of perceived events. One example is sector rotation, in which assets may be reallocated from one industry sector to another, as the business cycle changes.

Dividends are:

Taxed as ordinary income Dividends are taxed as ordinary income at the investor's tax rate (even if reinvested to acquire more shares), usually at no more than a tax rate of 20%

If an adviser is using a sector rotation strategy, which of the following asset classes would they rotate into when the economy appears to be ready for recovery?

Technology During recovery, the sectors that perform the best are technology, industrial, and cyclicals. During a recession, the sectors that perform the best are consumer staples, utilities, service, and financial industries.

What's the best indicator of a successful mutual fund?

Tenure of the manager Tenure refers to the length of time that a manager has been running a mutual fund portfolio. Investment advisers with a longer tenure have most likely been successful over a long period. Previous year results are probably not a reliable indicator of success, since most mutual fund investors have a longer term outlook.

If a client's objective is long-term capital appreciation, all of the following insurance policies may be recommended by an adviser, EXCEPT:

Term life A term life policy would not provide future capital as it does not accumulate cash value. A whole life policy would accumulate cash value, though generally at a low rate. Universal life would also accumulate cash value that can be used to pay the premium, which reduces the cash value. With a variable life policy, a portion of the premium is invested in the separate account, which historically would provide a higher market based return.

A person has no existing life insurance policy and has little money available. Which policy should be recommended?

Term life The benefit of term life insurance is that it is the least expensive alternative. A person is able to buy a predetermined amount of coverage for a fixed period. Term life policies provide pure insurance, but do not build cash value. Each of the other options are more expensive and build cash value.

What type of insurance policies have level premiums and level death benefits?

Term life and whole life Both term life and whole life insurance policies have fixed premiums and fixed death benefits. On the other hand, the premiums on universal life policies can be changed and the death benefits of variable life policies can fluctuate.

According to the Uniform Securities Act, an Administrator may NOT impose which of the following requirements on an investment adviser?

That a federal covered adviser must keep books and records beyond the time period stated by the SEC Federal covered advisers must only comply with SEC requirements. State requirements cannot supersede those of the federal government for federal covered advisers.

All the following statements apply to a rule of the Administrator, EXCEPT:

The Administrator is not required to publish rules All rules of the Administrator must be published. The rules of the Administrator are not part of the law, but interpret the state's laws. To change a state's securities laws, approval from the state legislature (Congress) is required. (62963)

While a customer who resides in State A is traveling through State B, he is solicited to purchase a security by an agent who is registered in both State A and State B. The customer pays for the security while he is in State B, but then later, the agent sends the confirmation to the customer's home address in State A. Which Administrator(s) has/have jurisdiction over the transaction?

The Administrator of State B only Under the Uniform Securities Act, an Administrator has jurisdiction over any offer to buy or sell that is made and/or accepted in its state. In this question, the customer was solicited while he was in State B (i.e., the offer was made in State B) and the offer was also accepted while the customer was in State B. For those reasons, the Administrator in State B has jurisdiction over the transaction.

Under the Uniform Securities Act, all of the following statements are TRUE regarding the requirements for investment advisers that have a place of business in a state, EXCEPT:

The Administrator will inspect their books and records annually An Administrator will not inspect the books and records of an investment adviser on an annual basis. However, if an IA has custody of customer funds, an independent public accountant must perform an annual audit. All of the other choices are true.

Which of the following statements is TRUE regarding the details of nonpublic administrative investigations?

The Administrator will not provide the details of the investigation to any member of the public According to the Uniform Securities Act, no provision of the Act authorizes the Administrator or any of her officers or employees to disclose information except among themselves, or when necessary in a proceeding or investigation under the Act that was not made public

An investor who resides in State Y is visiting State X. Before returning to his home, he meets with a friend who is an agent and is registered in State X. While at a restaurant in State X, the agent convinces the client to immediately buy a specific security, rather than waiting until the client gets home. The client pays for the purchase and is told that the confirmation will be sent to his home in State Y. If the agent sold this client an unregistered, non-exempt security, which of the following statements is TRUE?

The Administrators of State X and State Y may take action against the agent. An Administrator has jurisdiction over every offer or offer to sell that is made or accepted in a state. In this question, an offer was made and accepted in state X however, the client resides in state Y. Though the Administrator in state X has jurisdiction, action can be taken by the administrator of both states

An investment adviser representative and a client have similar financial resources, investment goals, and risk tolerance. However, although the IAR recommends penny stocks as a small part of her client's portfolio, she would never consider investing in such securities herself. Which of the following statements is TRUE?

The IAR should tell her client that the recommendation is inconsistent with her own investment policy An investment adviser whose personal investing is inconsistent with recommendations made to clients generally has an obligation to disclose this to customers.

According to NASAA Model Rule 411(c), an investment adviser is required to keep a record of all the following documentation, EXCEPT:

The IARs personal e-mails All of the documentation listed is required to be maintained by investment advisers with the exception of the personal e-mails of their representatives.

What theory states that investors are basically risk-averse and that, if presented with two investments offering the same expected return, they would prefer the less risky one?

The Modern Portfolio Theory The Modern Portfolio Theory (MPT) assumes that investors are risk-averse. Essentially, this means that if an investor is given two portfolios that offer the same expected return, she will prefer the one presenting less risk. An investor may be willing to assume greater risk provided she is compensated with higher expected returns.

Which of the following is TRUE of the NAV for a closed-end fund?

The NAV is calculated at the end of every business day. As with open-end (mutual) funds, closed-end funds must calculate the NAV of their shares on a daily basis. However, the NAV doesn't represent the price of fund's shares; instead, closed-end fund shares trade on the exchange at a price that's determined by the forces of supply and demand.

Which of the following statements is TRUE regarding SEC Release IA-1092?

The Release noted that an individual who provides advice about securities in general may meet the definition of investment adviser SEC Release IA-1092 was created to provide guidance to the industry on how existing statutes (the Investment Advisers Act of 1940 and the Uniform Securities Act) applied to some of the newer types of advisory services being offered (e.g., wrap accounts, etc.). The Release was not intended to replace either the Investment Advisers Act or the USA. One of the issues that IA-1092 examines is how the ABC test is met by someone who provides general advice about securities investing but does not recommend specific investments. Under the Release, persons who give generalized advice about investing in securities, as a business, and for compensation, are deemed to be investment advisers.

Under the Investment Advisers Act of 1940, if an individual wants to create her own investment advisory firm, with which authority would she need to file the application?

The SEC An investment adviser registering under the Investment Advisers Act of 1940 would register with the SEC. In this question, if the advisory firm is required to register in a state, it would do so with an Administrator under the provisions of the Uniform Securities Act

Under the Uniform Securities Act, an investment adviser who has no place of business in a state is exempt from registration if:

The adviser has no more than five clients who are residents of the state Under the Uniform Securities Act, an adviser does not need to register in a state if it has no place of business in the state and does not have more than five clients who are residents of that state. This is referred to as the de minimis exemption.

Sharon Smith is a registered investment adviser who frequently recommends the purchase of limited partnership interests to her clients. Her personal financial condition and objectives are similar to her clients' but she does not purchase limited partnership interests for her own account. According to the Investment Advisers Act, which of the following statements is TRUE?

The adviser must disclose the fact that her personal securities transactions are inconsistent with the advice given to clients An investment adviser generally must disclose that her personal securities transactions are inconsistent with the advice given to clients, regardless of whether the adviser receives performance-based fees. (62956)

All of the following information would be permitted in an advertisement by an investment adviser, EXCEPT a list of:

The adviser's top-ten corporate pension fund clients along with the reasons they have accounts managed by the adviser Regarding specific standards, investment adviser advertising may not refer, directly or indirectly, to any testimonials about the adviser or its services. A testimonial is an advertisement in which an individual recommends the investment adviser's services based on personal experiences with the adviser. Testimonials are prohibited whether or not the clients consented to allowing their names to be disclosed. There is no prohibition against listing the name(s) of clients or employees of the adviser along with their professional designations.

Jack is going through a divorce. He has an individual account managed on a discretionary basis by Trust and Worthy Advisers, Inc. The advisory firm receives a subpoena from the attorney for Jack's wife demanding the records for his account. Which of the following statements is TRUE?

The advisory firm must provide the records demanded in the subpoena It is considered unethical for an investment adviser to disclose the identity, affairs, or investments of any client unless the client consents, or unless the law requires it. Trust and Worthy would be compelled by law to release the records of its client to the attorney.

An agent receives a letter from an irate client. The letter is the fifth in the last six months and the language is abusive. The agent decides against a reply and discards the letter. According to the Uniform Securities Act, which of the following statements is TRUE?

The agent must forward all written complaints to an immediate supervisor

Which of the following would NOT be an important consideration when conducting a capital needs assessment for a client?

The amount of anticipated volatility in the marketplace A capital needs assessment analyzes a client's future goals and needs. Retirement planning, college funding, and the risk of death before meeting a savings goal are all considered. A client's life expectancy, the rate of inflation, and her earnings will all affect the capital needs assessment. Market volatility may influence the securities on which recommendations are based, but not the capital needs assessment

Which of the following statements about variable annuities is FALSE?

The annuity feature protects investors from capital losses Variable annuity assets are directed into a separate account and invested in a portfolio that fluctuates with the market. Therefore, an investor's principal will fluctuate over time as it remains invested in a variable annuity. Mutual funds are often an investment choice within an annuity. If an investor is interested in principal protection and a guaranteed rate of return, he should consider a fixed annuity.

A customer buys a premium bond that is callable. Which of the following would be least beneficial for the customer?

The bond is called at its par value in five years If the bonds are called in five years at par, the premium paid for the bond would be amortized over the shortest time period. This would result in the investor realizing a lower yield than if the bond were called after a longer period of time. It is important to note that MSRB rules require a municipal firm to disclose to a customer the lowest possible yield that the customer can realize. On a premium bond (as in this example), the lowest yield will result from the bond being called at par in the shortest time period.

The limited registration provision available to Canadian broker-dealers conducting business in a state permits which of the following actions?

The broker-dealer transacts business with Canadian residents with whom the broker-dealer had an existing relationship The provisions allowing Canadian broker-dealers to transact business in a state are limited. A broker-dealer may effect transactions with a person from Canada who is temporarily in the state if there was an existing broker-dealer-client relationship before the person entered the United States.

If a client inherits $250,000 of GE stock, and that is his only asset, the adviser would be most concerned that:

The client lacks diversification If GE declines in value, the client is at risk of losing his entire investment. Therefore, the adviser is most concerned that the client's assets are not diversified among other asset classes.

A pension consultant receives compensation for working with a company's pension plan. Under the Investment Advisers Act of 1940, in which of the following situations is the consultant acting as an investment adviser and required to register?

The consultant recommends that the company use an asset allocation model that invests in securities and non-securities-related investments in order to better diversify risk The Investment Advisers Act of 1940 regulates pension consultants as investment advisers if they receive compensation and provide securities-related advice. The best choice is when the consultant recommends the use of an asset allocation model that invests in both securities and non-securities to diversify risk. The other choices are examples of incidental advice and don't require the pension consultant to be regulated as an investment adviser. Also, remember that real estate is not considered a security.

All of the following items should be included on an order ticket, EXCEPT:

The customer's home address A customer's home address is not required on an order ticket.

Which of the following is NOT a characteristic of whole life insurance policies?

The death benefit fluctuates or adjusts to the market In a whole life insurance policy, premium payments are invested in the general account of the insurance company. The insurance company guarantees the owner's cash value and provides a fixed, guaranteed death benefit.

Which of the following is FALSE regarding a Health Savings Account (HSA)?

The distributions that are used for non-qualified medical expenses are subject to a 50% penalty The distributions that are used for non-qualified medical expenses are subject to a 20% tax, not a 50% penalty. All of the other statements are true.

If a mutual fund shareholder decides to reinvest the dividends that are paid by the fund and purchase additional shares, what is the tax treatment?

The dividend is taxed even if it is reinvested When a mutual fund makes a dividend distribution, an investor is able to reinvest the distribution and use the funds to buy additional shares at their net asset value. However, the dividend is still taxed as ordinary income for the investor.

Which of the following statements about testamentary trusts is NOT TRUE?

The donor manages the assets in the trust. A testamentary trust goes into effect after the donor passes away. The assets are required to go through the probate process. Once the assets are put into the trust, the trustee (not the donor) manages the assets for the beneficiaries.

In what way does variable life insurance provide for a death benefit that can keep up with inflation?

The earnings in the subaccounts are added to the guaranteed minimum death benefit every year The death benefit will be increased each year by any increase in the value of the subaccounts. If the value of the subaccounts declines, the death benefit will decline but never below the minimum guaranteed amount. A variable life policy would be chosen because the owner feels that the growth in the subaccounts will provide a death benefit that is likely to keep up with inflation over the long run.

FGW Investment Advisers created an aggressive investment strategy that outperformed the market over the past four years. Recently, FGW's performance has been poor which has resulted in a loss of a few clients. In order to maintain the interest of the remaining clients, FGW holds an information session. What economic argument could FGW use to explain the reason for the poor performance?

The efficient market hypothesis The Efficient Market Hypothesis states that financial markets are efficient and that the prices of securities reflect all known information and will adjust instantly to reflect any new information. Therefore, outperforming the market over an extended period is unlikely.

Which of the following is a valuation model used to calculate the anticipated return for a portfolio of securities?

The expected rate of return The expected rate of return is used to estimate or anticipate the performance of a portfolio by averaging all of the possible returns and the probability that they will occur.

Which of the following statements is TRUE according to ERISA section 404(b)?

The fiduciary may maintain the indicia of ownership of assets of the plan outside the jurisdiction of the district courts of the U.S. if authorized by the Secretary of the Department of Labor The rules according to ERISA section 404(b) prohibits fiduciaries from maintaining the indicia or evidence, of ownership of plan assets outside the jurisdiction of U.S. courts, unless allowed by the Secretary of the Department of Labor.

An investment adviser is registered in 10 states. The firm wants to transact business with three clients in a state in which it is not registered. According to the Uniform Securities Act, which of the following statements is TRUE?

The firm would not be required to register as an investment adviser if it did not lease an office in the state in which it is not registered According to the Uniform Securities Act, if a firm has five or fewer clients in a state (during a 12-month period) in which it has no office, it is exempt from registration in that state. Also, if all the clients, regardless of their number, are banks, trust companies, insurance companies, or employee benefit plans, the firm is exempt from state registration.

An investment adviser manages the portfolio of a mutual fund and is considered to have custody of the assets. Based on this information, the adviser is subject to an annual audit and is required to maintain copies of which of the following records for a minimum of five years?

The fund's audited financial statements Generally, investment advisers are required to maintain records for five years. In this question, the adviser must retain the mutual fund's audited financial statements for five years. Since this adviser is maintaining custody of a mutual fund's assets, it is also required to schedule an annual audit to be conducted by an independent accountant

Which of the following would NOT be important when analyzing the issuer and price of a fixed-income security?

The investor's age An investor's age has no effect on the price of a bond. All of the other items listed would impact the value of a bond that trades in the market. For instance, the issuer's balance sheet gives insight into what it owns compared to what it owes at a specific point in time.

According to the Uniform Prudent Investor Act, what would be of LEAST concern to an investment adviser?

The length of the adviser's contract and amount of fees collected The length of time remaining in the client's contract is the least of the adviser's concerns. Factors that are of greater concern include taxes, inflation, and the direction of the economy. (67682)

Four municipal bonds each have a 3% coupon, the same face value, but different maturities. One bond matures in one year, another in five years, another in 10 years, and the last in 20 years. Duration is:

The longest for the 20-year bond and shortest for the one-year bond Duration is a measure of a bond's interest-rate risk. Long-term bonds will have a larger duration and more interest-rate risk. However, for a bond that pays interest, the duration will be less than its maturity.

After years of managing his own money, a client approaches an investment adviser representative seeking some financial advice regarding his IRA account. The IAR notices that the client's current investments include stocks, bonds, Treasury inflation-protected securities (TIPS), and a municipal bond fund. Also, the client is holding a small amount of cash that has not been invested. Which of the instruments in this client's IRA would be the biggest concern for the IAR?

The municipal bond fund In an IRA, the investment creating the greatest concern would be the municipal bond fund. Interest earned in an IRA is deferred, so, there is no need to purchase a tax-free security. Including municipal securities in an IRA is not advisable since the tax-free income received from the municipal bond fund becomes taxable if purchased in an IRA and later distributed.

What information is not included on the application for registration of a broker-dealer?

The names and addresses of the agents the broker-dealer intends to register The names and addresses of the agents the broker-dealer intends to register are not required. The qualifications and history of any partner, officer, director, or controlling person are required.

A company's market capitalization (size) can be measured by using which of the following calculations?

The number of common shares outstanding multiplied by the market value of the stock To calculate the market capital (market cap) of a company simply multiply the number of common shares outstanding by the current market price of the stock. Companies are referred to as either being small-cap, mid-cap, or large-cap.

A company's market capitalization or size can be measured by using which of the following calculations?

The number of common shares outstanding multiplied by the market value of the stock When calculating the market capital (market cap) of a company, simply multiply the number of common shares outstanding by the current market price of the stock. Companies are referred to as small-cap, mid-cap, or large-cap.

Which of the following is a characteristic of a Subchapter S corporation, but is not a characteristic of a limited partnership?

The number of potential investors is limited. Both Subchapter S corporations and limited partnerships pass through both earnings and losses to their investors, which eliminates double taxation. Neither organization is responsible for paying taxes on its earnings; instead, their earnings are distributed to the owners for tax reporting purposes. Additionally, both S Corporations and limited partnerships have centralized management. However, unlike limited partnerships, S Corporations are limited to 100 shareholders.

When considering estate planning needs, what can be said regarding Section 529 plans?

The plan participant maintains control of how the funds are distributed One of the advantages of a Section 529 plan is that the plan participant, the parent, etc. is the account owner and maintains control of how the funds in the plan are distributed and to whom. The beneficiary does not have to pay federal taxes on qualified withdrawals and the assets in the plan are generally not considered part of the participant's estate for federal estate tax purposes.

Two bonds each have a coupon rate of 3.5%, but one has a five-year maturity and the other has a 10-year maturity. If interest rates increase by 1%

The price of the 10-year bond depreciates more than the price of the five-year bond. When interest rates rise, bond prices will fall. However, the price of bonds with longer maturities will fall more than the prices of bonds with shorter maturities. Long-term bonds have a greater duration.

A company's PE ratio has historically been 30, but it has recently dropped to 15. In this case, what has most likely happened to the company?

The price of the company's stock has fallen. The Price-to-Earnings (PE) ratio is the price of a company's stock divided by its Earnings Per Share (EPS) (i.e., PE Ratio = Stock Price ÷ EPS). If the company's PE ratio has fallen, this means that either the stock price has fallen or the EPS of the company has risen. Paying common shareholders a dividend doesn't impact EPS and will most likely cause the stock's price to rise. (18614)

An annuitant has elected to annuitize using a 10-year period certain payout. His wife is the primary beneficiary and his brother is the contingent beneficiary. If the annuitant and his brother pass away three years after contract is annuitized, who will receive payments for the remaining seven years?

The primary beneficiary Annuitants can select more than one beneficiary on a period certain payout. The primary beneficiary is the first to receive payments, but only after the annuitant passes away. Contingent beneficiaries will only be paid after both the annuitant and primary beneficiary pass away. Since the annuitant's wife is the primary beneficiary and has not passed away, she will receive payments for the remainder of the period certain (seven years in this question).

Under the Uniform Securities Act, which of the following is TRUE regarding the antifraud provisions for exempt and non-exempt securities?

The provisions apply to both exempt and non-exempt securities Both exempt and non-exempt securities and both exempt and non-exempt transactions are subject to the antifraud provisions of the Uniform Securities Act. Exempt securities are not subject to the registration requirements of the Uniform Securities Act.

Which right is NOT granted to shareholders under the Investment Company Act of 1940?

The right to receive real-time updates of changes in the firm's portfolio Under the Investment Company Act of 1940, shareholders are not entitled to current information about the composition of the fund's portfolio. This information is considered to be proprietary and is not released in real-time. All of the other statements are true. On a semiannual basis, shareholders will receive reports which contain updated financial information about the fund as well as a list of the securities in the fund's portfolio.

If an investment adviser recommends that its clients diversify their investments by purchasing gold coins, gold certificates, or gold futures, which of the following risks is the adviser trying to avoid?

The risk of some investments losing value or performing poorly due to inflation Commodities investments, including futures, are a way to hedge against inflation risk. The risk that a single stock will perform poorly is referred to as business risk and may be diversified by purchasing stocks of multiple companies.

Registration of a security in a state is not required for ALL of the following reasons, EXCEPT:

The security has been registered with the Securities and Exchange Commission under the Securities Act of 1933 Under the Uniform Securities Act, a security is not required to be registered if: The security is exempt; or The security is non-exempt, but is being offered in an exempt transaction; or The security is a federal covered security; or The instrument does not meet the definition of a security Whether a security has been registered with the SEC (under the Securities Act of 1933) has no bearing on the state registration requirement. (88867)

A broker-dealer registered in Colorado sells a security listed on a national securities exchange. The transaction takes place in the secondary market and both clients are residents of Colorado. Under the USA, which of the following statements is TRUE?

The security is not required to be registered in Colorado This is an example of an exempt transaction since it is considered a nonissuer transaction executed by a registered broker-dealer where the security is listed on a national securities exchange. If the issuer were selling securities that were listed on a national securities exchange, this might qualify as an exempt security.

According to the Uniform Securities Act, which of the following statements best describes what it means for a security to be registered?

The security may be legally offered or sold in the state The Administrator does not rule on the accuracy or adequacy of any filing, nor does the government offer decisions on the investment merit or financial condition of an issuer. Essentially, when regulators grant a registration, it allows for the lawful offering and sale of securities within their jurisdiction.

Which of the following is TRUE if the price of a discount bond falls?

The spread between the yield-to-maturity and yield-to-call will increase and the yield-to-call will rise. When bond prices fall, both the yield-to-maturity and yield-to-call will rise. Short-term yields will always move more than long-term yields. Since the call date is always before maturity, the yield-to-call is a shorter-term than the yield-to-maturity on the same bond. To summarize, the yield-to-call will always move more than the yield-to-maturity. In this question, the yield-to-call is already higher than the yield-to-maturity since the bond is trading at a discount. When the price falls even further, the yield-to-call will increase more than the yield-to-maturity and therefore the spread between them will increase

Advisers whose advice is limited to U.S. government securities are defined as investment advisers and required to register with:

The state Administrator only According to the Investment Advisers Act of 1940, any adviser that limits its advice to U.S. government securities is excluded from the investment adviser definition. However, the Uniform Securities Act does not offer the same exclusion. For that reason, an adviser that provides advice only about U.S. government securities is defined as an investment adviser at the state level and is required to register with the appropriate state Administrator.

Under the Uniform Securities Act, which of the following BEST describes the term inspectorial power?

The state Administrator's power to subpoena records inside and outside of the state Inspectorial power refers to a state Administrator's ability to inspect or review any records that are located both inside and outside of the state in order to carry out the provisions of the Uniform Securities Act.

When making recommendations to senior investors, which of the following features is the LEAST important consideration?

The state in which the client has her primary residence FINRA published a notice discussing the suitability issues of senior investors. Some of the important questions advisers should ask prior to recommending a securities product are: What is the client's current employment status and what are her primary expenses? Does the client still make mortgage payments? How much income does she need and what are her sources of income? How important is liquidity, health care, and insurance? What are the client's investment goals? While the state in which the client has her primary residence may be important for tax and estate considerations, it should not be a primary concern of the adviser.

How does an Administrator determine whether excessive trading has occurred in a customer's account?

The suitability of solicited trades in the account Churning or excessive trading is a prohibited activity. To determine whether churning has occurred, regulators typically examine the suitability of solicited trades (i.e., those which were recommended by agents of a broker-dealer). Unsolicited trades are executed at the client's own volition, without advice from an agent of a broker-dealer, and are irrelevant for churning determination.

Under the Investment Advisers Act, all of the following statements are TRUE concerning agency cross transactions, EXCEPT:

The transaction must be conducted on an exchange An agency cross transaction is one in which the adviser acts as broker for both sides of the trade. All of the statements listed are true except the statement which indicates that they must be conducted on an exchange.

John and Chris are a married couple in their forties with two children. They have an annual income of $100,000. Their main assets are their house and John's 401(k) plan. They also have approximately $25,000 available for investment. Which of the following choices should the investment adviser representative recommend?

They should give the adviser a list of their investment objectives and goals The investment adviser needs more information about this couple's investment objectives before she can recommend a suitable strategy. All of the investment options could all be appropriate recommendations for this couple depending on their goals.

Broker-Dealer A is a publicly traded company listed on the New York Stock Exchange. Which of the following statements is TRUE regarding an agent of Broker-Dealer A who wants to sell securities of his company to a client?

This is acceptable if the agent discloses the relationship verbally prior to the transaction and in writing before the settlement date Failing to disclose that a broker-dealer is affiliated with or controlled by an issuer of securities is considered a dishonest and/or unethical business practice. The agent would need to disclose the affiliation before entering into any contract with a customer to buy or sell securities. The disclosure may be made verbally prior to the trade if written disclosure is made at or before the completion of the transaction (usually the settlement date). The disclosure would need to be made to any account of a broker-dealer.

A broker-dealer has an excellent research department with a highly rated team of analysts. An investment adviser manages several mutual funds as well as a hedge fund. The investment adviser wants to enter into an agreement with the broker-dealer to execute some of its transactions even though the broker-dealer charges higher commissions than some other brokerage firms. In return, the investment adviser would receive the broker-dealer's research reports and access to its analysts. Which of the following statements is TRUE?

This is an acceptable practice known as a soft-dollar arrangement In a soft-dollar arrangement, an investment adviser sends client transactions to a particular broker-dealer even though that broker's commissions may not be the lowest ones available in the market. In return, the investment adviser receives research or other services from the broker-dealer. In these arrangements, the clients of the investment adviser should be able to benefit by using the services of this broker-dealer as compared to a broker-dealer charging lower commissions. There is no requirement for the adviser to reduce its fee or obtain the client's prior written consent.

An investment adviser representative (IAR), who is also a member of a tennis club, offers club members discounted fees if they hire his firm to manage their money. The adviser makes full disclosure of this fact in both the brochure and in its ADV Part 2. Which of the following statements is TRUE?

This is an ethical practice since the details of the discounts have been properly disclosed in the brochure and ADV Part 2 It is not a violation of the Uniform Securities Act to offer discounted fees to advisory clients provided the nature of the discounts are fully disclosed to both the Administrator and clients.

Stan is an agent for Rocksolid Brokerage. He spends a lot of his free time fishing with his buddy, Oliver. Stan and Oliver heard of a franchising opportunity to provide vending machines that sell live bait. Stan has always wanted to start a business to supplement his income from selling securities, so he and Oliver formed a corporation to buy a vending machine franchise. Without telling Rocksolid, Stan sold small amounts of stock in the company to friends and neighbors to help get the enterprise started. Which of the following statements is TRUE?

This is an unacceptable practice since these securities transactions are not recorded on the books of Stan's broker-dealer It is considered an unacceptable practice for an agent to effect securities transactions that are not recorded on the books and records of the broker-dealer. Such transactions could involve violations of the Uniform Securities Act. Before engaging in such transactions, agents should notify their broker-dealers to obtain permission.

A client has terminated an investment advisory contract one week after signing the agreement. The adviser informs the client that a prorated portion of the advisory fee will be retained by the adviser, and the remainder will be sent to the client. Under the USA:

This practice is acceptable Investment advisers are allowed to charge a prorated fee for the time that a contract is in force.

An investment adviser is the manager of a mutual fund and several of its IARs are also registered representatives of an affiliated broker-dealer. The adviser executes portfolio trades through the affiliated broker-dealer and the broker-dealer receives commissions from these trades. According to the Investment Advisers Act, which of the following statements is TRUE?

This represents a conflict of interest, but is permitted if disclosure is made in the brochure There may be situations in which investment advisers will receive compensation in addition to the investment advisory fee that clients are being charged. Although this action may be deemed a conflict of interest, it is not prohibited. In order to alleviate any perceived unethical behavior, clients should be provided with disclosure regarding the additional compensation.

An asset management company would like to add a new method to calculate the fees it will charge to certain clients. The new fee would be performance-based and would be calculated over a period of time. Which of the following statements is TRUE?

This type of fee arrangement is permitted for qualified clients Under the Investment Advisers Act of 1940, performance fees are generally prohibited. Exceptions include contracts for clients who have at least $1,000,000 under management with the adviser or who have a net worth in excess of $2,100,000. They would be defined as qualified clients. Registered investment companies, clients that are not U.S. residents, and certain employees of the adviser may also be charged performance-based fees. The SEC does not need to approve the method used to calculate the fee, but it must be disclosed to clients. (79483)

A client requests that her agent display a quote in a thinly traded security. The client is the majority shareholder in this security and the broker-dealer honors the request and displays the quote. Which of the following statements is TRUE?

This would be permissible if the broker-dealer believed the quote was bona fide A broker-dealer is permitted to publish quotes (bid and ask prices) on behalf of its clients or for its own account. The broker-dealer must believe the quotes are bona fide and not intended to manipulate the market price of a security. If the quotes are not bona fide and the broker-dealer publishes them, they would have engaged in an unethical business practice.

Which of the following return calculations removes the distortions caused by the deposit and withdrawal of capital from an investment account over time?

Time-weighted return Time-weighted returns eliminate biases caused by the inflow or outflow of investor money. It is often used to compare the performance of money managers. On the other hand, dollar-weighted returns provide a better idea of how an individual investor has done over time by eliminating the biases caused by superior performance in one year or inferior performance in another.

An IAR walks into a coffee shop and sees a friend who invites him over to meet a group of his coworkers. One person admits that his sister is a client of the IAR. According to NASAA's model rules, which of the following is an acceptable response for the IAR?

To acknowledge that he knows the person's sister NASAA's model rules prohibit an IA or IAR from disclosing a client's identity, investments, or other confidential information without the client's specific written consent. Although acknowledging that the sister is a client and expressing that he can't discuss her confidential information seems like a reasonable answer, even if asked by a relative, the IAR is not permitted to disclose whether a person is a client. Remember, regulatory authorities have the ability to demand the disclosure of client information.

Which of the following is considered to be part of an investment adviser's fiduciary duty?

To ensure that all investment advice is impartial and disinterested Fiduciaries are obligated to put their clients' interests and needs ahead of their own and to offer impartial advice that is tailored to each client's specific goals. Since each client has her own objective, it is a violation of an IA's fiduciary duty to manage all accounts in the same way. Although many IAs separate client orders and require the clients to enter into arbitration, these are not part of their fiduciary duty.

What is the motivation behind setting up an UTMA account?

To provide gifts to the child/owner Due to the popularity of 529 plans, the effectiveness of custodian accounts has diminished. Any of the earnings that are generated in an UTMA are subject to taxation. The primary purpose for establishing a UTMA is to provide gifts of cash and/or securities for a child's future benefit.

In a retirement plan, if key employees own more than 60% of the plan's assets, it is considered a:

Top-heavy plan A top-heavy plan is one in which key employees own more than 60 percent of the plan's assets. If a plan is determined to be top heavy, there are certain steps that an employer can take to maintain the tax-advantaged status of the plan.

How do mutual funds report annual returns?

Total returns Mutual funds are required to disclose their returns to their shareholders using the Total Return formula. The NAV is the price of a mutual fund, not a measure of return. Arithmetic mean is simply an average, while beta measures the systematic risk of an investment.

A broker-dealer receives an order from a client to purchase 1,000 shares of ABC stock. Before executing the order, the broker-dealer purchases the same security for its own account at a price that would have satisfied the client's order. The broker-dealer's transaction is considered:

Trading ahead This question is an example of trading ahead of a client's order and is a prohibited practice. This violation occurs when a broker-dealer accepts an equity security order from a client and rather than immediately executing the order, the broker-dealer trades the security on the same side of the market for its own account at a price that would have satisfied the client's order.

A client of ABC Advisers has recently retired and is looking for an investment that will provide steady income and offer the ability to sell quickly if the need arises. Which of the following investments would be the BEST recommendation?

Treasury notes The best answer would be Treasury notes. Treasuries offer semiannual interest payments and the most liquid securities market. While some CDs and preferred stocks may be considered liquid, they are not as liquid as the Treasury market. Zero-coupon bonds do not provide semiannual interest payments

When determining a corporation's market capitalization, which of the following items is NOT included in the calculation?

Treasury stock Market capitalization is determined by multiplying all issued and outstanding shares by the current market price. Since treasury stock is not outstanding, but instead held by the corporation, it is not a factor in determining market capitalization.

As of the close of business on Monday, a state-regulated IA has fallen below its minimum financial requirement. When must the deficiency be reported to the state Administrator?

Tuesday If an IA falls below the minimum financial requirement, it must notify the state Administrator by the next business day.

All of the following are examples of derivatives, EXCEPT:

UIT municipal bond units Unit investment trusts (UITs) are a type of investment company. UIT units represent a direct investment in a fixed portfolio of securities and are not considered derivatives. All of the other choices are considered derivatives.

Maxman Investment Advisers has been in business for 17 years. The firm has not placed advertisements in the past to attract new clients but would like now to expand its business. The firm specializes in asset management and most of its clients are executives from video gaming companies. Maxman would like to include in the advertising quotes from some of the executives on how their returns have beaten the market over the last few years. This would be acceptable:

Under no circumstances Regarding specific standards, investment adviser advertising may not refer, directly or indirectly, to any testimonials about the adviser or its services. A testimonial is an advertisement in which an individual recommends the investment adviser's services based on personal experiences with the adviser. Testimonials are prohibited whether or not the client signs any document, the advertisement is filed with a regulator, or any disclosure is made concerning the persons quoted in the testimonial.

Which of the following insurance contracts have the elements of term insurance, a cash value that is not market-based, and a flexible death benefit?

Universal life Universal life is an insurance contract that allows the customer to select the amount of coverage and the size of the premium. Additionally, it has a cash value that grows at a minimum guaranteed rate. However, the performance of the cash value is not market-based.

What is the most suitable policy for an individual who wants to earn a higher return from an insurance policy, but does not want to assume market risk?

Universal life insurance In some cases, universal life insurance will either pay a minimum rate of return or slightly higher. Depending on the contract specifics, the additional rate of return could be pegged to a stock index or interest rate. An individual who purchases a variable annuity assumes the investment (i.e., market) risk of the security. If the separate account performs well, the value of the investment will increase. However, if the securities in the separate account perform poorly, the value of the investment will decline

When an accounting firm is auditing an issuer's balance sheet and income statement, which of the following auditor's opinions is the BEST?

Unqualified Unqualified opinions of auditors are typically considered the best, since they're issued without reservation. On the other hand, a qualified opinion is issued when an auditor questions at least one part of a company's finances (e.g., unusual expense). Adverse opinions are issued when an auditor doesn't believe that a company's finances represent the true results.

Under the Uniform Securities Act, for how long does an agent's registration remain in effect?

Until December 31 of the year her registration is granted Under the Uniform Securities Act, all registrations expire annually on December 31. Therefore, registrations must be renewed at the end of the calendar year.

A client had inherited an IRA from his mother. The client is able to

Use the 10-year rule for distributions An individual that inherits a non-spousal IRA must generally withdraw the funds in a 10-year period. The rules for a spousal IRA do permit withdraws to be taken over the surviving spouse's lifetime.

Kim Jones comes to your office and asks for advice concerning the funding of her retirement. She is considering using either her work-sponsored 401(k), a variable annuity offered by your firm, or a 529 plan. Your best advice to Kim is:

Use the 401k The best advice to give Kim is to fund her 401(k). The client will receive immediate tax relief from the reduction in her reportable income and tax-deferred growth. In a variable annuity, the earnings are taxed as ordinary income when withdrawn. 529 plans are primarily designed to help investors save for higher education expenses, not for retirement. Any withdrawals that are not used for qualified education expenses are subject to a 10% tax penalty. Opening an offshore account in the Cayman Islands to avoid all U.S. taxation is, of course, both unethical and illegal.

Monica comes to your office and asks for advice about funding her three-year-old son's college education. She is considering using a 529 plan, or a variable life insurance policy, or her employer-sponsored 401(k). Your best advice to Monica is:

Use the 529 plan since all qualified withdrawals will be free from federal taxes The best advice for Monica is to fund a 529 plan. Section 529 plans are specifically designed to help investors save for education expenses. All withdrawals that are used for qualified education expenses are free from federal taxation (and possibly state taxes as well). Withdrawals from 401(k) plans are taxed as ordinary income, not capital gains. There is nothing in the stem of the question to indicate that Monica needs life insurance. Her stated objective is saving for her son's college education.

A client calls an investment adviser representative to discuss a stock she is interested in purchasing. The stock has a low P/E ratio, a high dividend payout ratio and its issuer has a large amount of cash reserves. What type of stock is it?

Value Two of the characteristics of value stocks are a low P/E (price/earnings) ratio and a high dividend payout ratio (or high dividend yield). A company's dividend payout ratio is the percentage of its earnings that are paid to investors as a dividend. Value stocks are also characterized by low price-to-book ratios, which is consistent with a company that has a lot of cash on hand. In contrast, growth stocks, generally have a high P/E ratio and a low dividend payout ratio. These companies tend to keep most of their earnings in order to fund their continued expansion. Large-cap stocks are issued by companies that have market capitalizations of more than $10 billion. Many value stocks may also be large-cap stocks since they tend to be issued by mature companies with a history of regular dividend payments but they are NOT characterized by low P/E ratios.

An adviser was a fiduciary for a client who has recently died. What's the first thing that an IAR should do when she meets with the person who has power of attorney for the estate?

Verify the identity of the fiduciary When working with an estate, one of the first steps for an adviser is to identify and verify the person who will be managing the account (i.e., the executor or administrator). All of the other choices are lower priorities

An adviser was a fiduciary for a client who has recently died. What's the first thing that an IAR should do when she meets with the person who has power of attorney for the estate?

Verify the identity of the fiduciary When working with an estate, one of the first steps for an adviser is to identify and verify the person who will be managing the account (i.e., the executor or administrator). All of the other choices are lower priorities.

Which of the following statements is FALSE?

When securities are gifted, the donor may claim a deduction that is equal to his original cost basis. When securities are gifted, the deduction that a donor may claim is equal to the market value of the securities at the time of the gift. The donor will benefit if the stock price has risen, since he will avoid paying capital gains tax. If the shares are gifted to an individual, the recipient's cost basis is the original purchase price or the current market price, whichever is less. On the other hand, if shares are inherited by an individual, the beneficiary's cost basis is the market value of the shares on the date of the decedent's death

When is an investment adviser NOT required to provide a balance sheet to its clients?

When the adviser requires the prepayment of a $150 initial advisory fee NASAA's model rules require an adviser to file a balance sheet whenever the adviser requires the prepayment of fees in excess of $500, six or more months in advance of providing any services. The filing would also be required if an investment adviser maintains custody of customer funds and securities. Remember, an IA that has full discretion (i.e., the ability to withdraw funds or check-writing privileges) is considered to have custody.

According to the Investment Advisers Act of 1940, when is an investment adviser required to provide an audited balance sheet to its clients?

When the adviser requires the prepayment of a fee that is greater than $1,200, six months or more in advance of providing service Since state and federal laws overlap regarding the concept of providing an audited balance sheet, it is important to identify which regulator is asking the question. According to the Investment Advisers Act of 1940 (federal law) an adviser is required to provide clients with an audited balance sheet if it collects prepaid fees of more than $1,200, six months or more in advance of providing advisory services. However, according to the Uniform Securities Act (state law), an adviser is required to provide clients with an audited balance sheet if 1) the firm collects/solicits prepaid fees of more than $500, six months or more in advance of the service, or 2) the firm maintains custody or discretionary control of clients' assets.

When would a variable annuity be most suitable for a client?

When the client wants capital appreciation or growth over a long period Variable annuities are suitable for clients who are willing to invest for the long term and want to invest in the markets. The investment objective of variable annuities is capital appreciation (growth). Since a variable annuity's performance is tied to the market, its return is unpredictable and is not based on inflation. (

Which of the following is TRUE regarding the difference between owning property under community property compared to joint tenants with rights of survivorship?

With a community property agreement, when one owner dies the cost basis of all of the assets steps-up to the market value, thereby reducing potential capital gains Community property is a type of joint ownership that's only available to married couples and only in select states. On the other hand, joint tenants with rights of survivorship is available in all states and the joint owners don't need to be married. With both forms of ownership, assets will be transferred to the surviving owner(s) after the death of one owner. With community property, the cost basis of all of the assets is stepped-up after the death of one owner in a manner that's similar to inheriting securities. However, with joint tenants with rights of survivorship, the decedent's portion of the assets are stepped-up, but the surviving owner's assets basis remains the same.

Which of the following statements is TRUE regarding an individual who takes benefits from a retirement account that's been awarded under a QDRO?

Withdrawals can be made without penalty, even if the individual is under the age of 59 1/2. A qualified domestic relations order (QDRO) is created to divide a person's retirement account during a divorce. The spouse who receives benefits awarded under a QRDO is exempt from the early withdrawal penalties, but the withdrawals are taxable. The beneficiary can also roll her portion of the account into an IRA, thereby delaying withdrawals and taxes. QRDOs are used for any ERISA qualified retirement account, including pensions, 403(b) plans, and 401(k) plans.

An agent located in State W is employed by a broker-dealer that is owned by a bank. The bank is headquartered in State I. If the agent sells securities issued by the bank to clients in State I, the agent:

Would need to disclose the affiliation to the customer under any circumstances Failing to disclose that a broker-dealer is affiliated with or controlled by an issuer of securities is considered a dishonest and/or unethical business practice. The agent would need to disclose the affiliation before entering into any contract with a customer to buy or sell securities. The disclosure may be made verbally prior to the trade if written disclosure is made at or before the completion of the transaction (usually the settlement date). The disclosure would need to be made to any account of a broker-dealer.

The Smiths have little investment experience, but are interested in saving for retirement and their children's college education. They consult an investment adviser representative about purchasing mutual funds. The IAR recommends that they purchase variable life insurance instead, even though the Smiths already have large life insurance policies. The IAR discloses that he will earn a higher commission by selling a variable insurance policy instead of mutual funds. Given these circumstances, has the IAR violated his fiduciary responsibilities?

Yes, a variable life insurance policy is not a suitable recommendation for the Smiths A variable life insurance policy is not a suitable recommendation for the Smiths, given their expressed interest in an investment other than insurance. The primary purpose of life insurance is protection against premature death, not investment. Furthermore, the fact pattern in this question indicates that the Smiths have already satisfied their life insurance needs. There is nothing to indicate that their coverage is inadequate. The IAR's disclosure of the conflict of interest does not make up for the fact that the investment recommendation is not suitable.

An investment advisory client's holdings consists of: $ 6,000,000 -- Stock/bonds$ 1,000,000 -- Money-market funds$ 3,000,000 -- Real estate, commodities$10,000,000 -- Total assets Would these holdings be considered a securities portfolio?

Yes, because more than 50% of the assets are securities. When securities represent a majority of an investor's total assets, the investor is considered to have a securities portfolio. In this question, it is important to recognize that money-market funds are also considered securities. For this client, 70% of her total assets represent securities. When determining the amount of assets under management for an IA, the securities portfolios of its clients are included. The amount of assets under management are disclosed by the IA in Form ADV Part 1.

Janet has been married twice and earned more than each husband. Both marriages ended in divorce. Her first marriage lasted for 10 years, while her second marriage lasted for 12 years. Janet is now collecting Social Security. Are either of her ex-spouses eligible to collect Social Security based on Janet's work history?

Yes, but only if each ex-spouse does not remarry. If a person is divorced, but his marriage lasted 10 years or longer, he is entitled to receive benefits on his ex-spouse's work history (even if she has remarried) provided: He is unmarried; He is age 62 or older; His ex-spouse is entitled to Social Security or disability benefits; and The benefit he is entitled to receive based on his own work history is less than the benefit he would receive based on his ex-spouse's work history

An investment adviser's computer system was hacked and customer data was compromised. At the time of the breach, the firm was in the process of establishing procedures which required password and data protection. Is the firm at fault?

Yes, since a cyber preparedness policy should have been in place As with individuals and other businesses, investment advisers are potentially vulnerable to hackers. IAs are required to implement a policy that includes encryption, anti-virus, anti-malware programs, as well as employee training.

An attorney has been appointed as trustee for a trust created by Nancy Walker. The beneficiaries of the trust are Ms. Walker's three minor children. The attorney hired you to provide investment advice. You have interviewed the attorney and Ms. Walker extensively and are confident that you understand the trust's investment objectives and tolerance for risk. You are now ready to prepare a financial plan. Which of the following statements is TRUE?

Your fiduciary responsibility has been fulfilled, so you may proceed You may interview the children or their father, but it is not required. As adviser to a trust, your fiduciary responsibility is principally to the beneficiaries of the trust. The adviser must look through the trust and remember that the investments must be suitable based on the beneficiaries' objectives. The adviser would receive a copy of the trust to see which investments would be suitable.

Chuck, an IAR, is discussing an equity-indexed annuity with a client. In discussing the participation rates of the annuity, Chuck explains that if the S&P 500 Index gains more than 10%, the annuity will be credited with no more than 7%. Chuck is describing what type of annuity?

a capped equity-indexed annuity In a capped equity-indexed annuity, the participation rate is capped, regardless of how much the index increases, thus limiting any gain potential

Under the Uniform Securities Act, all of the following are considered to be investment adviser representatives, EXCEPT

a clerical employee of the advisory firm Employees who perform clerical or ministerial duties are excluded from the definition of an investment adviser representative. This term includes any employee, partner, officer, or director of the adviser, who sells advisory services, performs research, makes recommendations, or manages portfolios.

Under the USA, if information in a document that has been filed by an IA with the Administrator becomes inaccurate or incomplete, a correcting amendment must be filed:

promptly A prompt filing, typically within 30 days, is required by an IA if material changes are made to any document that has been filed with the Administrator (e.g., a change of address, a change in management, etc.).

Which of the following would NOT be considered a reason to suspend or revoke an investment adviser's registration under the Uniform Securities Act?

the advisor was sued by a client and lost the case Any violation of federal or state securities laws or any conviction in the last 10 years for a felony or securities-related misdemeanor is justification for the Administrator to take action against a registration. An investment adviser losing a lawsuit that was brought on by a client is not cause for the immediate revocation of registration, provided the registrant had not violated industry rules.

A mother creates a trust and deposits funds that will be used to pay for her children's college educations. The IAR managing the assets will NOT take into consideration:

the amount of money that the grantor makes The key variables that an IAR should consider are the timing of the outlays, the expected cost of higher education, and how market conditions will influence potential returns. The grantor is the person who created the trust and her income is not a factor that warrants consideration.

A client wants to make a payment in perpetuity of $3,000 per year to a beneficiary. Assuming a 3% annual return, how much principal would your client need to deposit?

$100,000 The client would need to deposit $100,000. To calculate the required principal, take the annual payment in perpetuity of $3,000 and divide it by the annual rate of return of 3% ($3,000 / .03 = $100,000). The phrase in perpetuity may also be referred to as a perpetual payment, meaning that payments will continue to be made forever.

A TIPS is issued at par and has a coupon of 4.0%. What's its principal value if the CPI increases 3%?

$1030 TIPS are bonds that are issued by the U.S. Treasury and are designed to protect investors from inflation. When inflation rises, as measured by the Consumer Price Index (CPI), the principal on a TIPS bond will increase by the same amount that the CPI rises. In this question, the bond started with a par (principal) amount of $1,000. If the CPI increases by 3%, the new principal amount is $1,030 ($1,000 x 3% = $30, and $1,000 + $30 = $1,030).

A client invested $100,000 in an Equity Indexed Annuity. The participation rate is 90% with a cap rate of 15%. In year one, the index increased by 20%. In year two, the index lost 5%. In year three, the index gained 10%. What is the value of the annuity after year three?

$125,350 An Equity Indexed Annuity is credited with the lesser of the participation rate or the cap rate, based on the performance of an index such as the S&P 500. In year one, the index increased by 20%. 90% of the gain is equal to 18%, but since the annuity can't be credited with more than the cap of 15%, the value of the annuity would be $115,000 (1.15 X $100,000). Many Equity Indexed Annuities have a floor of zero, thus a negative return in the index will not cause the value of the annuity to decline. Therefore, in year two, the value of the annuity would remain the same. In year three, the index increased by 10%. 90% of the gain is 9%, which is less than the cap rate, so the annuity would be credited with 9%, or $125,350 (1.09 X $115,000).

For a Coverdell ESA, what is the maximum allowable contribution and how is treated for tax purposes?

$2,000 per year and made on an after-tax basis up to the child's 18th birthday The maximum allowable contribution to a Coverdell Education Savings Account (CESA) is $2,000 per year up to the child's 18th birthday. The funds are contributed on an after-tax basis and may be used for any level of education.

On October 25, Mr. Smith purchased 5 listed XYZ Corporation July 50 calls and paid a $3 premium on each call. The current market price of XYZ Corporation is $48 per share. What would the breakeven point be for Mr. Smith per option?

$53 The strike price plus the premium equals the breakeven point for the buyer of a call. The breakeven point is $53 (the $50 strike price + the $3 premium = $53).

A client invests $10,000 into Company A, $10,000 into Company B, and $10,000 into Company C. — Company A's stock paid a $200 dividend and dropped 20%.— Company B's stock paid a $600 dividend and rose 5%.— Company C's stock paid no dividend and rose 7%. What is the client's total return?

0% Using the total beginning value of all investments, $30,000, the total ending value is $29,200, and the total annual earnings of $800, the client's total return may be calculated. ending value - beginning value + income ---------------------------------------------- beginning value 29200 - 30000 + 800 / 30000 = 0%

All of the following maturities are offered for TIPS, EXCEPT:

1 year Treasury Inflation-Protected Securities (TIPS) are offered in 5-year, 10-year, and 30-year maturities. Maturities of one year are not available for TIPS.

A customer invested $25,000. After 20 years the investment is now valued at $100,000. How many years did it take to double in value?

10 The $25,000 investment doubled twice, from $25,000 to $50,000, then to $100,000, over 20 years, or it doubled every 10 years.

The trustee is responsible for reporting all income, gains, and losses of a trust to the IRS on Form:

1041 On an annual basis, a trustee must report the trust's income, gains, and losses to the IRS on Form 1041. Regarding the other forms, Form 1040 is used for personal tax returns, Form 1065 is an informational return that is filed by partnerships, and Form 1040EZ is used by single person or those who are married, filing jointly with less than $100,000 of taxable income

A portfolio has a beta of 1.0 and an expected return of 12%. What would the alpha of the portfolio be if the beta was 1.4 and the actual return was 18.8%?

2.00% Alpha is the difference between the portfolio's actual return (which is given) and expected return. The expected return can be determined by using the Capital Asset Pricing Model (CAPM). Since this question doesn't provide a risk-free rate, the calculation of expected return is simply beta multiplied by the market return (Expected Return = Beta x Market Return). As this question begins, a portfolio with a beta of 1.0 will have the same expected return as the market return. In other words, both the expected and market returns are 12% (Beta of 1.0 x Market Return of 12% = 12%). However, if the beta changes to 1.4, the expected return will rise to 16.8% (Beta of 1.4 x Market Return of 12%). The question specifically states that the actual return of the portfolio was 18.8%. Therefore, the alpha can then by calculated by taking the actual return on the portfolio minus the expected return (actual return of 18.8% - expected return of 16.8% = +2.00% alpha).

An investor buys a security for $10,000 and the security is redeemable in one year for $10,000. When the investment is redeemed at the end of the year, the owner receives $300 in income. If the CPI over the holding period is 3%, what's the investor's holding period return?

3% Holding Period Return is also referred to as Total Return. The formula for Total Return is: (Ending Value - Beginning Value) + Investment Income/Beginning Value. In this question, the investor paid $10,000 and received $10,000 when it was redeemed; therefore, she has no gain or loss. Since she did receive $300 in investment income, her holding period return is 3% ($300/$10,000). Although the question provides the rate of inflation (i.e., CPI = 3%), Holding Period Return isn't adjusted for inflation and isn't used in calculating the answer to this question.

Which of the following allocations would be the LEAST suitable for an investor with a 30-year time horizon, moderate risk tolerance, and the goal of long-term growth?

50% money-market funds, 50% long-term government bond funds An investor with a long time horizon and a goal of growth should have some part of her portfolio in equity securities. An allocation with 50% money- market funds and 50% long-term government bonds is unlikely to meet this investor's goal.

A front-end load is the fee that is payable on the purchase of:

Mutual fund class A shares When an investor buys Class A mutual fund shares, he is assessed a load (sales charge) at the time of purchase (i.e., at the front-end). The other choices are in no way associated with a load (sales charge).

Which of the following statements is NOT TRUE regarding limited liability companies?

Only a limited number of states permit them Limited liability companies may now be established in any state by filing the appropriate documents with the secretary of state or another state authority. All the other statements are true.

When managing a portfolio of bonds, which of the following would be considered a passive investment strategy?

Buy and hold A buy-and-hold strategy is a passive one in which the portfolio generally remains fixed with very little turnover. Barbell, immunization, and interest-rate anticipation strategies all require a portfolio manager to actively rebalance the portfolio at various points in time or due to changing market conditions

What's required if an individual wants to give her spouse complete control over her investments?

A full power of attorney Full power of attorney is needed for complete control of an account. Limited power of attorney only gives the designated person the ability to buy and sell securities, but not to make withdrawals. Durable power of attorney can provide either full or limited in authority. Simply having durable power of attorney doesn't provide a person with full control of an account. The other choices will not provide the person with control of the assets until the original owner dies.

Which of the following may be advertised as a no-load fund?

A fund that assesses a 12b-1 fee of 0.25% of average net assets per year Regulators have created standards for advertising a fund as a "no-load." No-load funds cannot have a front-end sales charge, a back-end sales charge, or a 12b-1 fee that exceeds 0.25% of the fund's average net (not total) assets for the year.

How is a person's marginal tax bracket determined?

By reviewing the IRS tax table that's available on its website A taxpayer can research the tax tables on the IRS website. The tax bracket into which a person falls is determined by examining her income less her deductions.

For a person to be eligible for a Health Savings Account (HSA), he must be:

Covered under a high deductible plan (HDHP) To be eligible for a Health Savings Account (HSA), a person must be covered under a high deductible health plan (HDHP), not a qualified retirement account. Also, to be eligible for an HSA, individuals cannot be enrolled in Medicare or be claimed on another person's income taxes. Each eligible spouse must open a separate HSA, since joint HSAs are prohibited. Withdrawals taken from an HSA are tax-free if the funds are used to pay qualified medical expenses.

If an adviser wants to evaluate a publicly traded firm's ability to pay down its short-term debt, which ratio would be most appropriate?

Current Ratio The best measure of short-term liquidity is the current ratio, which is calculated by dividing current assets by current liabilities. Although the debt coverage ratio also measures liquidity, it actually includes all debt-both short-term and long-term.

Which of the following statements is TRUE regarding tenancy in common?

Due to the nature of the agreement, each individual's interest is generally more freely transferable In an account with a tenancy-in-common arrangement, the assets may or may not be equally divided. When an owner dies, his ownership interest in the account is included in his estate and is subject to probate. The benefit of a tenancy-in-common arrangement is that it makes it easier to transfer assets to other investors when one person dies. There is no requirement that the owners of a tenancy-in-common account be a married couple.

Which of the following terms relates to the graph of optimal portfolios resulting from a comparison of risk and return?

Efficient frontier According to modern portfolio theory, a graph of optimal portfolios may be created known as an efficient frontier

A customer opens an options trading account and purchases one STC call option with a strike price of 40. She pays a premium of 3.22. Which of the following statements is/are TRUE? The customer's breakeven point is 43.22 per share The customer is bullish on STC The customer is bearish on STC The customer's breakeven point is 36.78 per share

I and II only When buying a call option, the breakeven formula is the strike price + the premium. The customer's breakeven point is 43.22 (40 + 3.22). When an investor buys a call option, she is bullish. She wants the value of the underlying security (STC) to increase above her breakeven point. Customers who sell uncovered call options have the opposite strategy, bearish, but the breakeven calculation is the same for the individual who sells an uncovered call (the strike price + the premium)

Under SEC Release IA-1092, which TWO of the following statements are TRUE? An individual who provides tailored advice about securities but does not execute transactions is an investment adviser. An individual who writes an investment column for the Floor Street Journal is an investment adviser. An individual who provides advice about which fund manager a client should choose is an investment adviser. An individual who provides advice about which fund manager a client should choose is an investment adviser only if he is compensated by the manager he is recommending.

I and III According to SEC Release IA-1092, an individual that provides advice about a client's specific situation is still considered an investment adviser even if that individual does not implement the advice. Individuals selecting a particular investment manager would also be considered to be providing advice and fall under the definition of an investment adviser.

Which TWO of the following statements would most likely violate the USA's investment adviser antifraud rules? Our representatives are approved by the state Administrator. Each of our representatives has a minimum of 10 years' experience as a financial planner. Each of our investment adviser representatives is a qualified investment expert. Each of our representatives has earned either Certified Financial Planner or CPA certification.

I and III Advisers are not approved by the Administrator. The term expert may not be used to describe the registration status of an IAR or an investment adviser. Stating your employee's experience is a fact as long as you do not state that experience equals expertise. Earning the CFP designation is also a statement of fact.

A client purchases a TIPS with a 4% coupon at par. If the CPI indicates that the rate of inflation has increased by 5%, which TWO of the following statements are TRUE? The client will receive a 4% coupon rate The client will receive a 9% coupon rate The client will receive a 5% increase in his next payment The client will receive a 1% increase in his next payment

I and III Treasury Inflation-Protected Securities (TIPS) are U.S. government securities that have a principal amount that is adjusted for inflation (based on the Consumer Price Index or CPI). Choice (I) is correct since the coupon rate on a TIPS is fixed. The principal amount on which interest is based will change according to any increase in the rate of inflation, which has increased by 5% in this question; choice (III). The investor's coupon payment will represent 4% of the adjusted principal amount.

Which TWO of the following statements are TRUE of stop orders? A stop order may be described as a suspended market order A stop order may be executed only at the stop price or better A stop order, when triggered, guarantees an execution A stop order, when triggered, becomes a limit order and needs its limit price to be satisfied for execution

I and III A stop order becomes a market order (in turn receiving immediate execution) when a round-lot trades at or through its stop price. A stop-limit order becomes a limit order when a round-lot trades at or through its stop price, and requires that its limit price be satisfied to receive an execution. A stop order is sometimes described as a suspended market order since execution depends on the stop price being triggered first.

If a corporation's convertible bondholders convert their bonds into the common stock, the effect on the corporation's balance sheet does NOT include: An increase in current assets A decrease in total liabilities A decrease in stockholders' equity An increase in stockholders' equity

I and III only When a corporation's bonds are converted into common stock, there is no change to its current assets and its stockholders' equity is not decreased. Instead, the removal of the debt obligation will result in a decrease in the total liabilities and the additional shares will increase its stockholders' equity.

Sales of viatical investments can only be made to suitable investors. Which TWO of the following are considered suitable? An accredited investor under regulation D Anyone who has been specifically approved by the state Administrator Anyone who is in the highest marginal tax bracket and is in need of liquidity Anyone with a minimum net worth of $150,000 and gross income last year of at least $100,000, or a minimum net worth of $250,000

I and IV A viatical investment involves the purchase of an interest in an insurance policy covering the life of an individual. The purchase may be for a whole or fractional interest in the policy at a price above the cash value. The investors pay the premiums on the policy until the death of the insured at which time the death benefit is paid to the investors. Since it is unknown when the insured will die and the funds are not readily assessable on demand, NASAA has specific suitability requirements as found in choices (I) and (IV).

A client creates a limited liability company (LLC). Which TWO of the following statements are TRUE? Like a corporation, LLCs provide investors with limited liability. The owners who manage the LLC are personally responsible for all debts that are incurred. The income earned by the company is taxed like a corporation. The income flows through to investors and is taxed like a partnership.

I and IV Limited liability companies provide investors with limited risk. In other words, they may lose no more than their investment. Unlike corporations, LLCs are tax-efficient since they do not pay income taxes. Instead, the investors are required to pay taxes on the income earned by the company.

A broker-dealer has their principal office in Texas. They transact business with 8 banks in Arizona, 3 investment companies in Nevada and a client who is attending grad school in Ohio. Which state(s) must the broker-dealer be registered in? Texas Arizona Nevada Ohio

I only According to the Uniform Securities Act, a broker-dealer that has no place of business in a state and only transacts business there with issuers, other broker-dealers, financial institutions, or existing clients who are not residents of the state, are excluded from the definition of a broker-dealer and not subject to registration.

A broker-dealer would be required to register in Pennsylvania if the broker-dealer: Has an office in Pennsylvania and executes nonissuer transactions of securities listed on a national securities exchange Has no office in Pennsylvania and executes nonissuer transactions of securities listed on a national securities exchange with clients that are residents of Pennsylvania Has an office in Pennsylvania and executes transactions of municipal securities with clients that are not residents of Pennsylvania Has no office in Pennsylvania and executes transactions of municipal securities with clients that are residents of Pennsylvania

I, II, III, and IV In all four choices, the broker-dealer would be required to register in Pennsylvania. A nonissuer transaction of a security listed on a national securities exchange is an exempt transaction and a municipal security is an exempt security. A broker-dealer executing a transaction in an exempt security or exempt transaction is required to register, unless it is exempt from the definition of a broker-dealer in that state. Since the broker-dealer either has an office in Pennsylvania or is transacting business with residents of that state, it would be required to register.

An investment adviser wants to enter into an advisory relationship with a client in a state without being registered there. Legally, the relationship may be established if the advisory firm has no place of business in the state and all of its clients are: Insurance companies Banks and/or trust companies Investment advisers Broker-dealers

I, II, III, and IV The Uniform Securities Act does not require the registration of an IA if it has no place of business in the state and all of its clients are institutional investors. For purposes of this rule, broker-dealers, trusts, insurance companies, banks, and other investment advisers are all considered institutional investors

Under the Investment Advisers Act, records that MUST be maintained by an investment adviser include: All checkbooks, bank statements, and cancelled checks A record of the personal securities transactions of the adviser and its employees Copies of all circulars, advertisements, and newspaper articles sent to ten or more persons

I, II, and III The Investment Advisers Act specifies which records an adviser must keep and maintain. All of the choices listed are required. (

Limited partnerships are offered to the public or are sold through private placement. To avoid registration with the SEC, partnership interests can only be sold to which of the following persons? Any number of accredited investors No more than 35 nonaccredited investors No more than 35 accredited investors Both accredited and nonaccredited investors

I, II, and IV only In a private placement under Regulation D, the securities are exempt from registration with the SEC, provided all investors are accredited investors and/or, no more than 35 nonaccredited investors. To be accredited, the investor must be a financial institution, or an individual with a net worth of at least $1 million, or at least $200,000 of annual income ($300,000 if married).

When meeting with a potential customer for the very first time, which of the following would be a reasonable course of action? Prior to the meeting, informing the client to bring additional financial information. If necessary, an IAR may inform the client that he may not know the answer to one of her questions, but will find out and respond within a reasonable period. Inform the client that she needs to increase her risk tolerance to obtain her goals. Discuss the customer's current financial situation and her goals for retirement.

I, II, and IV only When meeting with a potential customer, it would not be reasonable to tell her that she needs to change her risk tolerance. A client may increase the future value of a portfolio by saving more money or by allowing her money to compound over a longer period. It is often considered inappropriate to encourage a client to assume risk beyond her comfort level.

Under the Uniform Securities Act, which of the following individuals are required to register as an agent? The CEO of a company who sells commercial paper with an 11-month maturity to the general public A life insurance salesperson who only sells general account products A New York-based salesperson who sells debt obligations for the city of Dresden, Germany An administrative assistant of a brokerage firm who only accepts unsolicited orders when his boss is unavailable

I, III, IV An employee of an issuer who sells exempt securities to the public is NOT considered an agent. Commercial paper may be an exempt security, but only if its maturity doesn't exceed nine months. Since the commercial paper in Choice (I) has an 11-month maturity, it's a non-exempt security and the CEO is required to register as an agent. According to the USA, only the securities of national foreign governments (with the exception of those issued by Canadian political subdivisions) are exempt. In Choice (III), the city of Dresden, Germany is issuing the debt; therefore, the salesperson is required to register. If the securities had been issued by the national German government, they would be exempt and so too would the salesperson. Lastly, since the administrative assistant is representing a broker-dealer and has been authorized to accept unsolicited orders, registration as an agent is required.

According to NASAA Model Rule 502(c), regarding contents of investment advisory contracts, an advisory contract would include which of the following choices? The term of the contract An exculpatory clause waving compliance with the Investment Advisers Act of 1940 A nonassignment clause A clause requiring the adviser to notify clients of any change in membership if the firm is a partnership

I, III, IV only It is unlawful for an investment adviser, investment adviser representative, or federal covered investment adviser to enter into, extend, or renew any investment advisory contract unless it provides, in writing, the term of the contract, how the fees will be calculated, the fact that the adviser may not be compensated on gains only, and a nonassignment clause. It is unlawful to include any condition, stipulation, or provision binding any person to waive compliance with any provision of this Rule or of the Investment Advisers Act of 1940. This prohibited practice is known as an exculpatory clause

Which TWO of the following meet the definition of a broker-dealer under the Uniform Securities Act? An agent of a broker-dealer who effects securities transactions for the firm A person that effects securities transactions for the accounts of others A person that effects securities transactions for its own account A representative of an IA who provides securities related advice to customers

II and III Under the Uniform Securities Act, a broker-dealer is defined as a person that is in the business of effecting securities transactions either for the accounts of others or for its own account. For that reason, choices (II) (acting as a broker) and (III) (acting as a dealer) are correct. Choice (I) is the individual agent who represents the broker-dealer in effecting securities transactions. Remember, agents are NOT considered to be broker-dealers. Choice (IV) is an individual investment adviser representative who provides advice to the advisory firm's clients and manages accounts/portfolios. Investment adviser representatives are NOT considered to be broker-dealers.

A registered representative planning to supplement her income by selling securities away from her firm must first: Notify her supervisor either orally or in writing Notify her firm in writing Receive her firm's written permission Notify the state securities Administrator or the SEC

II and III only A registered representative who wants to sell securities outside of her normal course of employment must first notify her firm in writing about the proposed transactions and must receive her firm's written permission. The notice must describe the proposed private securities transactions in detail. If the firm approves the representative's activities, then it is responsible for supervising the transactions and must record them in its books and records.

According to the Uniform Securities Act, persons providing investment advice are NOT required to register as investment advisers if the adviser: Deals only with insurance or investment companies and has only one office in the state Is a savings institution, bank, or trust company Is in a profession such as accounting or law, where the performance of this service is incidental Conducts business exclusively with broker-dealers and other investment advisers and maintains branch offices only in the state

II and III only Choices (II) and (III) are specifically excluded from the definition of investment adviser under both the Uniform Securities Act and the Investment Advisers Act. The advisers in choices (I) and (IV) would not need to register as investment advisers if they did not have a place of business in the state.

Kristine has created a business of providing advice regarding the pricing of Nasdaq-listed stocks, but does not effect any of her clients' trades. According to the Uniform Securities Act, she must comply with which of the following? Rules related to the broker-dealer's registration Antifraud provisions Rules related to investment adviser registration

II and III only If a person gives advice about listed securities but does not effect any transactions, she is required to register as an IA, but not as a broker-dealer. Since Kristine is not working for a registered investment adviser, she would need to satisfy IA registration requirements in order to provide this type of advisory service. It is important to remember that if an IAR works for a registered investment adviser, she is exempt from registration as an investment adviser. An IAR (employee of the advisory firm) is not held to the same registration requirement as the adviser (firm) itself. Also, the antifraud provisions apply to all securities and registered personnel.

How is alimony treated for tax purposes? Alimony payments are deductible to the payer. Alimony payments are taxable to the payer. Alimony payments are taxable to the payee. Alimony payments are not taxable to the payee.

II and IV Alimony is taxable to the spouse who makes the payments (i.e., the payer), but is not taxable to the recipient (i.e., payee). Alimony is taxed at ordinary income rates.

Which TWO of the following are characteristics of hedge funds? They are mutual funds. They are typically sold through a private placement. They are registered and heavily regulated. They use leverage, derivatives, short positions, and invest in illiquid asset classes.

II and IV Hedge funds are generally sold through a private placement and are exempt from most regulations. In many cases, hedge funds leverage their portfolios and attempt to earn higher-than-average returns by investing in derivatives and illiquid securities or assets.

Which TWO of the following statements are TRUE? A customer will sell at the ask. A customer will sell at the bid. A customer will buy at the bid. A customer will buy at the ask.

II and IV When trading securities in the secondary market, investors sell at the bid price and buy at the ask (offering) price. (67489)

Bill is an investment adviser representative for an advisory firm that has satellite offices in Florida and California, but its principal office is in New York City. The adviser has assets under management of $63,000,000 and its largest client is the Aquarius SmallCap Growth Mutual Fund. Bill works in the New Jersey office and has clients that reside in Florida, New York, and New Jersey. In which of the following states must Bill register as an investment adviser representative? California New Jersey New York Florida

II only Although Bill's advisory firm has less than $100 million under management, it is an adviser to a registered investment company. For that reason, the firm is considered a federal covered investment adviser and is only required to register with the SEC (i.e., it is exempt from registration at the state level). The IARs of federal covered advisers are required to register with the Administrator in any state in which they maintain an office. In this question, since Bill only maintains a place of business in New Jersey, he is required to register as an IAR in New Jersey. (89124)

Which of the following statements are TRUE regarding the administrative proceedings under the Uniform Securities Act? Denial orders issued by the Administrator are final, binding, and may not be appealed Administrators are not permitted to retroactively enforce orders and revocations Administrators are permitted to issue summary orders denying certain exemptions The burden of proof for qualifying for an exemption is on the party claiming the exemption

II, III, IV Choice (I) is not true since orders issued by the Administrator may be appealed within 60 days. All of the other choices are true statements

A mutual fund is planning to issue 10 million Class B shares. Five hundred thousand shares will be offered in the state of Rhode Island. Under the Uniform Securities Act, the Administrator of Rhode Island will require the fund to: Register the shares in Rhode Island Include a prospectus with its registration Pay a filing fee Sign a Consent to Service of Process

III and IV only Mutual funds are federal covered securities and, therefore, a state may not require registration or regulate any offering document. Except for securities that are listed on one of the exchanges (such as the NYSE or Nasdaq), the state may charge a filing fee. The state may also require issuers to submit a filing notice and sign a Consent to Service of Process.

Manny is an active septuagenarian who would like to create a living trust. This trust is referred to as a(n):

Inter vivos trust A living (inter vivos) trust is established during the donor's lifetime. A testamentary trust is established through instructions left in the donor's will. In vitro and prewilled are not types of trusts

Which of the following types of risk would have the greatest impact on a 20-year corporate bond during the first year of the investment?

Interest-rate risk A change in interest rates would most likely have the greatest impact on the bond in the first year of the investment. If interest rates were to go up or down, that could dramatically change the bond's price. The risk presented by inflation would be significant over a longer term. Liquidity risk is the risk that a security cannot be traded quickly enough in the market to prevent a loss (or make the required profit). Market (systematic) risk is the risk that a security's value may decline over a given period due to economic changes or other events that impact large portions of the market.

A dollar-weighted return is also referred to as:

Internal rate of return A dollar-weighted returns is the same as the internal rate of return (IRR). It represents the discount rate that will make the present value of future cash flows (i.e., interest payments and principal) equal to the current market price (i.e., current value).

A pension fund manager wants to protect the fund's diversified stock portfolio against a market downturn. To best meet this objective, she should purchase:

Puts on a comparable index Index options would move in the same direction as the market as a whole and, therefore, provide a better hedge for a diversified portfolio than individual stock options.

Which of the following is an example of an exempt transaction under the Securities Act of 1933?

Reg. D Since this question is asking about an exempt transaction at the federal level (Act of 1933), the only appropriate answer is Regulation D. A Regulation D offering is also considered private placement and represents a federal exempt transaction. Under the Securities Act of 1933, U.S. Treasuries, municipal securities, and railroad equipment trusts are all exempt securities (not transactions).

According to the Securities Act of 1933, a pooled investment fund is considered a federal covered security when it:

Registers with the SEC under the Investment company act of 1940

An Administrator has determined that a broker-dealer is selling unregistered securities in the Administrator's state. Which of the following statements is TRUE if the broker-dealer does not comply with the cease-and-desist order issued by the Administrator?

The Administrator may bring action in a court of competent jurisdiction to request an injunction The Administrator may take more severe action if a broker-dealer does not comply with a cease-and-desist order to stop selling unregistered securities in the Administrator's state. This would entail the Administrator going to state court to ask for an injunction.

All of the following would be reasons for a state Administrator to revoke an IA's registration, EXCEPT:

The adviser filed for personal bankruptcy four years ago Personal bankruptcy is not an event that leads to the revocation of a registration. However, any violation of securities laws or industry regulations, or conviction within the last 10 years for any felony or securities-related misdemeanor, is grounds for revocation of a registration. Although the Commodities Exchange Act and the CFTC govern futures trading, a commodities violation would be grounds for the loss of a securities registration

Mary is a 32-year-old investor with an aggressive risk tolerance. She has trading authorization for the account of her 87-year-old aunt who's in ill health. The account was opened with the aunt's money and for the aunt's benefit. Mary is the sole heir of her aunt's estate. Whose objectives and needs should an investment adviser representative consider when recommending investments for the account?

The aunt's only Any investments that the adviser recommends for the account must be suitable for the aunt, since the account was opened with her funds and for her benefit. The fact that Mary expects to inherit the money is not relevant, even if this is likely to occur fairly soon given her aunt's age and condition. (15671)

Who typically does not supply the property to create a trust?

The beneficiary The terms settlor, grantor, and maker are synonymous. The person who supplies the assets to place in a trust may also be referred to as the donor or trustor. When taking your examination, you may see these terms used interchangeably. The term beneficiary of the trust refers to whom the trust will benefit.

Which of the following is a benefit of a transfer on death (TOD) account designation?

The beneficiary avoids probate A transfer on death (TOD) account will pass to the beneficiary (or beneficiaries) upon the death of the account owner. The benefit of a TOD designation is that the transfer of ownership of the account's assets avoids probate. However, assets in a TOD account are still subject to the estate tax. This form of account is not considered a joint account since the beneficiary only receives control of the account after the death of the original owner.

The ABC Growth Fund charges a 12b-1 fee. This fee is based on:

The fund's average annual NAV A 12b-1 fee is assessed against the average annual NAV of a mutual fund and is used to cover the costs associated with promotion, distribution, and the trailing commissions (trailers) that are paid to registered personnel. The 12b-1 fee is a part of the operating expense ratio of a mutual fund and the fee is ultimately paid by the shareholders.

ABC Inc., a financial services company, is registered as both a broker-dealer and an investment adviser. On a regular basis, ABC is required to provide its clients with disclosures and obtain written agreements from them regarding acting in both a broker-dealer and investment adviser capacity. In which of the following situations is ABC not required to obtain a written agreement from the client prior to effecting the transaction?

The investment adviser side of ABC makes no recommendation to the client, but the client decides to effect a securities transaction through the broker-dealer If the investment adviser side of ABC makes no recommendation and the client decides to effect a securities trade through the broker-dealer, the investment adviser disclosure rules don't apply since the client has not used the adviser's services. However, broker-dealers are required to disclose on trade confirmations whether they have acted in a principal or agency capacity.

The Department of Labor requires that employer-sponsored plans adhere to the following guidelines, EXCEPT:

The plan must offer a fixed number of investment options Under Section 404(c) of ERISA, an employee sponsored plan has no maximum limitation as to the number of investment options that are available to the participants, but must have a minimum of three. The plan must maintain a separate account for each participant and participants must have the right to forward any requests to the fiduciary of the plan

Which of the following is a characteristic of hedge funds?

Their managers usually receive performance-based fees Hedge fund managers generally receive a fee that is tied to the fund's performance as well as a management fee. For example, the manager may receive a 2% management fee plus a 20% performance fee (two and twenty fee). There is no active market for hedge fund shares; therefore, they are usually illiquid investments. Hedge fund managers may invest in all types of assets, including precious metals and commodities. Hedge funds may be sold or marketed to any investor as long as the recommendation is suitable. However, certain exemptions do apply if they are only sold to accredited retail investors and/or institutional investors

An agent has been given the login and password for a client's account. The client has provided written authorization for the agent to login and place trades in the client's account. This activity is considered:

Unacceptable, since it likely violates the terms of service of the broker-dealer's online account access system. Since the agent is logging into the website using the client's credentials, it's impossible to determine which transactions were entered by the agent and which were entered by the customer. Since broker-dealers cannot identify whether trades were fraudulent, they prohibit the sharing of usernames and passwords in their terms of service.

Which of the following best describes Class A shares of a mutual fund?

They have up-front sales charges Class A shares typically have front-end sales charges, but no deferred sales charges (i.e., back-end loads). In some instances, Class A shares have 12b-1 fees (an annual charge), but they're typically lower than the 12b-1 fees associated with Class B and C shares.

George and Barbara are a wealthy couple in their 70s with five grandchildren. The couple wants to set aside the maximum amount possible for each grandchild's college education, while minimizing their estate tax exposure. Based on these investment objectives, which of the following recommendations is the most appropriate for them?

They should establish a 529 plan for each grandchild. The couple could only contribute $2,000 per year to a Coverdell Education Savings Account for each grandchild. However, they're able to contribute much more to a 529 plan ($30,000 per year or $150,000 every five years per designated beneficiary) without tax consequences, and the amount contributed will also be removed from their estate. For a married couple, the annual amount that can be given and avoid gift tax consequences is $30,000. Therefore, the five-year accelerated contribution is $150,000

Melinda, an agent for Broker-Dealer X, finds a handwritten note on her desk left by a client who frequently stops by her office. In the note, the client states that she is unhappy with the recommendations that Melinda has made recently. Melinda has recommended only the stocks that appear on the firm's Buy List. The client has lost money on some of the recommendations that she accepted. Which of the following statements is TRUE regarding this note?

This is considered a written complaint and should be forwarded to a supervisor It is considered a fraudulent practice for agents to fail to notify a supervisor of written customer complaints. Such complaints do not need to conform to a specific format--any written grievance from a customer should be considered a complaint. The decision regarding how to respond to a complaint is a supervisory or management issue. Such decisions should not be made by agents who have not notified their supervisor of the existence of the complaint.

Nick is a client of Nora, who represents Whiteglove Securities. Nora's recommendations have made Nick a lot of money over the last two years. Nick is so pleased that he decides Nora can keep 5% of any gains in his account from now on, in addition to the commissions that he usually pays. Which of the following statements is TRUE?

This is not acceptable as it is considered sharing in the client's profits Sharing in the profits and/or losses in a client's account is generally not permitted. There is an exception if the client and the agent have a joint account, the client and the broker-dealer consent to the arrangement in writing, and profits and losses are shared in proportion to the capital that each contributed to the account.

An investor has allocated 95% of her portfolio in the stock of one company, with the remaining 5% in cash. Although the client tells her IAR that the stock has increased 40% over the last year, she is interested in his advice regarding her portfolio. Which of the following recommendations is MOST appropriate?

To diversify the porftolio in order to lower risk Most investment professionals would not recommend for investors to focus 95% of their portfolio on one stock. Although the upside could be very rewarding (as suggested in this question), the client is exposed to significant losses if the stock performs poorly. By investing in a variety of securities and using asset allocation strategies, an investor's returns may fall, but so too will the risk. If one particular stock or investment performs poorly, it will not be as potentially devastating due to diversification. Essentially, diversification lowers both the potential risk as well as the potential returns.

Which of the following is a leverage ratio?

Total Debt/Total Equity Leverage is a measure of the amount that a firm has borrowed. One of the more common ratios used to measure a company's leverage is the Debt-to-Equity Ratio, which is calculated by taking total debt (i.e., liabilities) and dividing by total equity

Which of the following is NOT an option exercise style?

Uncovered Uncovered is a term that relates to an option position or strategy that investors may employ and it's in no way connected to an option exercise style. American, European, and Capped are option exercise styles. American style options allow the owners to exercise their contracts any time prior to expiration. European style options allow the owners to exercise their contracts only during a specified period, typically on the business day of expiration. Capped style options will be exercised automatically if the value of the underlying security hits or exceeds a specified capped price.

Howie is both a registered investment adviser and a licensed real estate agent. He recently prepared a financial plan for Bob. In this plan, Howie recommended that Bob increase his life insurance coverage and also consider putting a portion of his portfolio in hard assets, such as real estate. Howie knew of a property called Blackacre that was currently for sale and told Bob about it. Bob eventually purchased Blackacre using Howie as his real estate agent and Howie received a commission for the purchase. Would Howie's commission for selling Blackacre be considered compensation under the Investment Advisers Act?

Yes, Howie's commission for Blackacre would be considered part of his compensation as an investment adviser SEC Release 1092 states that investment adviser compensation includes "any economic benefit." It can include commissions generated by the sale of nonsecurities products such as insurance or real estate. Thus, Howie's commission for Blackacre would be considered investment adviser compensation within the meaning of the Investment Advisers Act.

The Dividend Discount Model is BEST described as:

a model that determines a stocks price based on future dividends The Dividend Discount Model indicates that the current value of a stock should be based on its future dividends which are discounted to present value. If the current market value of a stock is less than the discounted value of future dividends, it is considered a bargain. From a practical standpoint, the model has value for utility stocks, REITs, and companies that pay steady dividends. (88618)

A client is interested in finding an investment that will provide growth potential and future income. He does not want to pay any taxes on his investment until he withdraws the funds at retirement. Of the following investments, which one is the best choice?

a variable annuity Variable annuities allow investors to defer the tax liability on their investment and buy into a portfolio of securities that will satisfy the growth objective. When the investor is ready to retire, the annuity may provide payments to the investor. The payments may represent both a return of capital as well as taxable investment income.

Generally, the identity of an estate's executor can be determined from which of the following documents?

a will Normally, the deceased has named an executor (executrix) in her will.

If an IA has recently changed the number of partners, but not control of the firm, the IA registration must be amended within 30 days. Since there is no change in control or ownership of the firm, a Form ADV-W is not filed. This type of change, however, is known as succession by:

amendment This is an example of succession by amendment, as the ADV is simply amended to reflect the new control structure. If the advisory firm were sold, then a change in ownership occurs and the new firm must submit a new application or ADV. Once effective, a Form ADV-W is filed to withdraw the registration of the acquired adviser

While meeting with a new client, an IAR is attempting to gauge the client's interest in investing funds in an IRA or buying a new car. This is the IAR's way of determining the client's:

attitude towards money ]Identifying whether a client would use available funds to invest for her future or buy a big ticket item is effective when determining her attitude toward money. For an IAR, this determination can be a complicating process. To make appropriate recommendations, an IAR must understand a person's fundamental approach to dealing with her finances. Family dynamics, personal beliefs, past experiences, and philosophical convictions can all influence a person's attitudes about money.

a client buys shares of a closed-end fund

in the secondary market Although a closed-end fund has a calculated NAV, it is unlikely that its shares will trade at that price. Instead, the shares may be purchased either above or below the NAV. The shares of a closed-end fund trade in the secondary market. This is unlike an open-end management company (mutual fund) whose shares constantly remain in the primary market.

Jack purchased 100 shares of XTRO at $20. After nine years, he gave the shares to his nephew Sam when the fair market value of XTRO was $16 per share. Sam held the stock for seven months and then sold the shares for $23 per share. What is the tax consequence for Sam?

long term capital gain of $300 If securities are received as a gift, any tax implication is delayed until the securities are subsequently sold. For the recipient, the two details that should be determined are 1) the donor's cost basis and 2) the fair market value (FMV) at the time of the gift. One of these two will represent the recipient's basis for determining a capital gain or loss at the time of sale (referred to as dual basis), as described below: If the securities are later sold for a price that is higher than the donor's cost, the seller's basis is the donor's cost and the donor's holding period is included. If the securities are later sold for a price that is lower than the FMV at the time of receiving the gift, the seller's basis will represent the FMV and the holding period begins on the day after the gift is received. When securities are given as a gift after they had appreciated in value and the recipient subsequently sells them for a gain, the recipient's basis is the lesser of the donor's cost or the fair market value at the time of the gift (i.e., always the donor's cost). However, this is a tricky question since the securities were gifted at a time when the fair market value ($16) is less than the donor's cost ($20). Sam later sold the stock for a price that was higher than Jack's cost. For that reason, Sam will use the $20 cost as his basis against the proceeds of $23 on the sale, which results in a capital gain of $3 per share, or $300. In this situation, Sam is able to add Jack's holding period to his own (nine years plus seven months) to establish a long-term holding period and therefore he realizes a long-term gain

A father buys stock on July 1 and gifts it to his son on December 1 of the same year. If the son sells the stock on July 2 of the following year, the son will have a:

long-term capital event When securities are gifted, the recipient's holding period typically begins when the donor purchased the securities. In this question, the holding period begins when the father purchased the securities. Since the son sold the shares more than one year after his father purchased them, the result is a long-term capital event

Which of the following types of business is often listed on an exchange?

master limited partnership Master limited partnerships can be established for businesses operating in commodities or real estate industries. As with limited partnerships, they are owned by limited and general partners and provide for the flow through tax treatment of income. However, master limited partnerships are often listed on national stock exchanges, which offers the partners more liquidity than traditional limited partnerships.

With a simple trust, the trust:

must distribute its annual investment earnings A simple trust is required to distribute all its income to beneficiaries in the year received. The body (corpus or principal) of the trust may not be distributed by the trustee. The term simple has nothing to do with the number of beneficiaries in the trust or the investment profile of the assets contained therein

An investor owns a TIPS bond that has a coupon rate of 5%. What happens to her TIPS if the CPI increases 3%?

principal increases by 3% TIPS are bonds that are issued by the U.S. Treasury and are designed to protect investors from inflation. When inflation rises, as measured by the Consumer Price Index (CPI), the principal on a TIPS bond will increase by the same amount that the CPI rises. However, the interest rate (i.e., coupon) on a TIPS bond will not change, regardless of what happens to the CPI

The FDA has announced that it is extending the trial period for a new drug of a pharmaceutical company. As a result, the company's stock price declines. This is an example of:

regulatory risk The situation being described is an example of regulatory risk. The Food and Drug Administration (FDA) is a federal agency and is responsible for promoting and protecting the public health of the United States. The FDA was empowered by Congress to enforce federal regulations that are based on both the Federal Food, Drug, and Cosmetic Act and the Public Health Service Act.

The prices of which of the following bonds would change the LEAST if interest rates rose?

short-term municipal notes The prices of short-term bonds tend to decline less when interest rates rise than bonds with longer maturities. Zero-coupon bonds also tend to be particularly vulnerable to increases in interest rates.

Which of the following metrics represents the basic measure of risk in Modern Portfolio Theory (MPT)?

standard deviation In MPT, the standard deviation of an investment's expected return is used as the basic measure of risk. Standard deviation measures the degree of dispersion or variability of returns from the mean return. The greater the standard deviation a portfolio has, the greater the risk it will have.

A client, age 61, has invested $200,000 in after-tax dollars in a variable annuity. His annuity is currently worth $380,000. The client decides to draw down $50,000 from the contract. How will the distribution be taxed?

the entire distribution will be taxable at ordinary income rates This question discusses a nonqualified annuity. In a nonqualified annuity, the investment is made with after-tax dollars. When a client makes a single (irregular) withdrawal from a contract, the IRS requires that a last in, first out (LIFO) method be used when calculating tax liability. This means earnings (the last in) come out first. In this case, the $50,000 is taken out of the $180,000 of earnings and would be fully taxable as ordinary income. Annuities never generate long-term capital gains.

A broker-dealer is subject to federal registration requirements and registers with the SEC. All of the following statements regarding its net capital are false, EXCEPT:

the net capital requirement is set by the SEC and the administrator may not set a higher amountThe Securities and Exchange Commission (SEC) regulates the net capital requirements of federally registered broker-dealers. The Administrator is prohibited from creating or enforcing a requirement that is more stringent than the amount set federally (by the SEC).


Kaugnay na mga set ng pag-aaral

Ch. 31 Medication Administration

View Set

2nd 9 Weeks WORLD GEOGRAPHY Exam

View Set

Reading 2 - Organizing, Visualizing, and Describing data

View Set

Nursing care during labor and delivery (test 2)

View Set

"Night To Remember" Study guide (Blazek)

View Set